Download as pdf or txt
Download as pdf or txt
You are on page 1of 258

Item: lof38 ~. , .

M k <:] t> al ~· ~
QIO: 1443 .l. ar Previous Next lab 'lifllues Notes Calculator

•1 •
A 25-year-old man with a 7-yea r history of epilepsy experiences the sudden onset of a generalized seizure . ~~AI
•2
The seizure involves movement of all four of his extremities and is associated with loss of consciousness .
•3 When emergency personnel arrive at the home 5-10 minutes later, the patient continues to show alternating
•4 stiffen ing and jerking of the limbs and ir regula r breathing and has not returned to fu ll consciousness.
•5
•6 What is the mechan ism of action of the medication that shou ld be given at this point to rapidly control the patient's
seizures?
•7
•8 :
A. Aminobutyric acid analog
•9
• 10 B. Increases duration of chloride channel opening duration
· 11 C. Increases frequency of ch loride channel open ing
• 12
D. Sod ium channel blocker
• 13
• 14
E. T-type ca lcium channel blocker
• 15
• 16
• 17

• 18
• 19
• 20
• 21

6
lock
s
Suspend
0
End Block
Item: lof38 ~. , . M k <:] t> al ~· ~
QIO: 1443 .l. ar Previous Next lab 'lifllues Notes Calculator

1 •

•2
The correct answer is C. 52°/o chose this.
•3
A person is said to be in status epilepticus when seizu re activity has continued fo r more than 5-10 minutes
•4 without regaining consciousness between episodes. The drug of choice for the t reatment of status epilepticus is
•5 the benzodiazepine, diazepam, because of its rapid onset of action . I ntravenous lorazepam is the drug of choice
in the hospita l, whereas diazepam is most often used as a home rescue medication. Benzodiazepines act by
•6
increasing the opening frequency of the ch loride channe l-associated y-aminobutyr ic acid (GABA) receptor, wh ich
•7 inhibits further neu rona l firing. The effects of both lo razepam and diazepam are virtually immediate and can
•8 contro l seizu re activity within seconds or minutes. Note that the definition of status epilepticus va ries between
•9 groups . Although some organizations define status epi lepticus as a single seizure of >30 minutes du ration, th is
is not practical clinica lly. As such, the accepted operational definition consists of> 5 minutes of continuous
• 10
seizure activity or two or more discrete seizures between which there is an incomplete recovery of
· 11 consciousness.
Benzodiazepine Diazepam lorazepam Status epilepticus Gamma-Aminobutyric acid Intravenous therapy Epileptic seizure Pharmaceutical drug
• 12
• 13 Receptor (biochemistry)

• 14 A is not correct. 7°/o chose this .


• 15 Gabapent in, an analog of the neurotransmitter GABA, is used fo r the treatment of simple or complex partial
• 16 seizures but is not used to t reat status epi lepticus. Gabapentin blocks high-voltage activated Ca 2+ channe ls .
Gabapentin Neurotransmitter Status epilepticus Gamma-Aminobutyric acid Seizure types Epileptic seizure Partial seizure Structural analog
• 17

• 18 B is not correct. 15% chose this .


• 19 Phenobar bita l is used as a second- line agent to t reat simple partial and gene ral ized tonic-clonic seizures and can
• 20
also be used for feb rile seizures in ch ild ren . Phenoba rbital is a very effective anticonvu lsant, but is not fi rst-l ine
treatment for status epilepticus because it requi res a longer t ime to administer and is associated with a highe r
• 21 incidence of respiratory depression than benzodiazepines. Phenobarbital increases the duration of chlor ide •

6
lock
s
Suspend
0
End Block
Item: lof38 ~. , . M k <:] t> al ~· ~
QIO: 1443 .l. ar Previous Next lab 'lifllues Notes Calculator
; I I - - "' • I "'
1
•2 Phenobar bita l is used as a second- line agent to t reat simple partial and gene ral ized tonic-clonic seizures and can
also be used for feb rile seizures in ch ild ren . Phenoba rbital is a very effective anticonvu lsant, but is not fi rst-l ine
•3
treatment for status epilepticus because it requi res a longer t ime to administer and is associated with a highe r
•4 incidence of respiratory depression than benzodiazepines. Phenobarbital increases the duration of chlor ide
•5 channel opening .
Anticonvulsant Phenobarbital Hypoventilation Status epilepticus Chloride channel Partial seizure Benzodiazepine Epileptic seizure Febrile seizure
•6
Generalised tonic-clonic seizure Fever Chloride Seizure types Major depressive disorder Depression (mood)
•7
•8 D is not correct. 19% chose this .
•9 Phenytoin acts on sodium channels. A loading dose of phenytoin or phosphenytoin is often given in status
epilepticus because it helps to control seizures and prevent seizu re recu rrence. However, benzod iazepines are
• 10
the first-line medication because they control seizures most effective ly and fastest .
· 11 Phenytoin Status epilepticus Benzodiazepine Sodium channel Sodium Epileptic seizure Pharmaceutical drug

• 12
E is not correct. 7°/o chose this .
• 13
Ethosuximide is primarily used in the treatment of absence seizu res and is not effective in the treatment of
• 14 status epi lepticus. Ethosuximide blocks low-th resho ld T-type ca lcium channels.
• 15 Ethosuximide Status epilepticus Absence seizure Voltage-dependent calcium channel Epileptic seizure Calcium channel Calcium T-type calcium channel

• 16
• 17
Bottom Line:
• 18
Cl inical ly, status epilepticus is defined as >5 minutes of seizure activity with no return to consciousness
• 19
between episodes. Initial t reatment is with a benzodiazepine such as diazepam or lorazepam. Diazepam is a
• 20 long-acting benzod iazepine that can be used fo r status epilepticus because of its rapid onset of action .
Benzodiazepine Diazepam lorazepam Status epilepticus Epileptic seizure
• 21

6
lock
s
Suspend
0
End Block
Item: 1 of 3 8 ~ 1 • M k -<:J 1>- Jil ~· !:';-~
QIO: 1443 ..L ar Pre v ious Next Lab fli!ltues Not es Calcula t o r
& &
1
•2 FA17 p514.1
•3 Epilepsy drugs
·4 GENERALIZED

•5 ~
<
'-' ~

•6
C>
~
:z
C>
~ ... "'
:::>
'-'
.,., ;=
..."':z...
~
'-' '-'
""
;= v :::>::1:;
,_
•7
...""
"" :z
,_
C>
<-
,_
"'
......
~

MECHANISM SIDE EFFECTS NOTES


""
·8 Ethosuximide • Blocks thalamic T-type Ca~· l•:J GI IIJ- Fthosuximide Sucks to have Silent
.9 .I channels causes Fatigue. Gl distress. (absence) Seizures
lleadache. Itching (and
• 10 urticaria). and Stevens-
• 11 Johnson syndrome
Benzodiazepines •• t Ci\BA,\ action Sedation, tolerance, Also for eclampsia sci7urcs (1st
• 12
(eg, diazepam, .I dependence, respiratory line is Mg$04)
• 13 lorazepam, dcprc~~ion
midazolam)
• 14
Phenobarbital .I .I t CABAA action Sedation, tolerance, Ist line in neonates
• 15 depe ndence. induction
• 16 of cytochrome P--!50,
• 17
Phenytoin, .I •
.I
...
.I
Blocks Na+ channels; 7ero-
order kinetics
cardiorespiratory depression
\lcurologic: nystagmus, di plopia, ataxia, sedation, peripheral
neuropathy. Dermatologic: hirsutism, Stc1·ens-)ohnson
• 18 fosphenytoin
S) ndrome, gingival hyperplasia, DRESS syndrome.
• 19
l\lusculoskeletal: osleopenia, SLE-Iike syndrome. llematologic:
• 20 megaloblastic anemia. Reproductil'e: teratogenesis (fetal
h)dantoin S) ndrome). Other: C)tochrome P-450 induction
• 21
• r:.. rh :a. ,..,~ .. onino ./ Rltv-lo• ~+ ,.},..,nnPI" 1""\inlnni~ o:thl\, i~ 1-.1~...1 le-t linP fnr trinomnin.,J nPuf'"'\lhi'l

a
Lock
s
Suspend
8
End Bl ock
Item: 1 of 3 8 ~ 1 • M k -<:J 1>- Jil ~· !:';-~
QIO: 1443 ..L ar Pre v ious Next Lab fli!ltues Not es Calcula t o r
& &
1
Carbamazepine ./ Blocks Na+ channels Diplopia, ata>.ia, blood 1st line for trigeminal neuralgia
•2 ./ d) scra~ia~ (agranulocytosis,
aplastic anemia), li\'er
•3
toxicit). teratogenesis.
·4 induction of C) tochrome
P-450, SIADII, Ste1 ens-
•5 johnson 51 nd rome
•6 Valproic acid ./ • ./ t '\a+ channel inactil·ation, C l distress. rare but futal Also used for myoclonic seizures,
•7 ./ t CABA concentration hepatotoxicity (measure bipolar disorder, migraine
by inhibiting CABA LF'ls), pancreatitis, neural prophylaxis
·8 transaminase tube defects, tremor, weight
gain, contraindicated in
.9
pregnancy
• 10 Vigabatrin t CABA by irrc\'ersibl) Perm.mcnt l'iSllalloss (black
• 11 inhibiting C.\ B. \ bo' liMning)
transaminase
• 12
Gabapentin ./ Primaril) inhi bits high-voltage- Sedation, ata,ia Also used for peripheral
• 13 acti\'a ted Ca 2+channels; neuropathy, postherpetic
designed as CABA analog neuralgia
• 14
Topiramate ./ ./ Blocks a+ channels, t C BA Sedation, mental dulling, Also used for migraine
• 15 action kidney stones, weight loss, prevention
• 16 glaucoma
Lamotrigine ./ ./ ./ Blocks voltage-gated a' Stevens-Johnson S)'ndrome
• 17
channels, inhibits the release (mmt be titra ted sloll'ly)
• 18 of glutamate
• 19 levetiracetam ./ ./ Unknoll'n; may modulate J•:1tigue, clro11siness.
CABA and glutamate release headache, neuropsychiatric
• 20 S\ mptoms {eg, personality

• 21 changes)
• . - .- ..
a
Lock
s
Suspend
8
End Bl ock
Item: 1 of 3 8 ~ 1 • M k -<:J 1>- Jil ~· !:';-~
QIO: 1443 ..L ar Pre v ious Next Lab fli!ltues Not es Calcula t o r

1 chnnges) •
.2 Tiagabine t Cr\BA by inhibit ing rcuplake

•3 = 1st line; = 1st line for acute; ••• = 1st line for J>rophylaxis.

·4
•5 FA17 p 515.2

•6 Benzodiazepines Diazepam, lorazepam, triazolam, tcmazcpam, oxazepam, m idazolam, chlordiazepoxide,


.7 alprazolam .
·8 MECHANISM Facilitate CABA,\ action by t frequency of "Frcnzodiazepincs" t frcqucncr.
.9 CJ- channel opening. l R E~ I sleep. Most Benzos, barbs, and alcohol all bind the
ha,·e long half-)i,·es and active metabolites C BA,\ receptor. ,,·hich is a ligand-gated Cl-
• 10
(exceptions [ATO!\ I]: Alprazolam, Triazolam, channel.
• 11
O xazepam, and ~ l idazolam are short ac ting O xazepam, Temazepam, and Lorazepam are
• 12 -+ higher addictive potential). metabolized O utside T he Liver
• 13 CLINICAL USE Anxiety, spas ticity, status epilcpticus (lorazepam
• 14 and diazepam), eclampsia, detoxification
• 15 (especially alcohol withdnl\vai- DTs), night
• 16 terrors, sleepwalking, general <~n est he tic
(amnesia, muscle relaxation), hypnotic
• 17
(insomn ia).
. 18
• 19
ADVERSE EFFECTS Dependence, additi,·e C ·s
depression effects
with alcohol. Less risk of respiratory depression
• 20 and coma than'' ith barbiturates.
• 21 Treat overdose with Aumazenil (compel it i' e

a
Lock
s
Suspend
8
End Bl ock
Item: 1 of 3 8 ~ 1 • M k -<:J 1>- Jil ~· !:';-~
QIO: 1443 ..L ar Pre v ious Next Lab fli!ltues Not es Calcula t o r

1 chnnges) •
.2 Tiagabine t Cr\BA by inhibit ing rcuplake

•3 = 1st line; = 1st line for acute; ••• = 1st line for J>rophylaxis.

·4
•5 FA17 p 515.2

•6 Benzodiazepines Diazepam, lorazepam, triazolam, tcmazcpam, oxazepam, m idazolam, chlordiazepoxide,


.7 alprazolam .
·8 MECHANISM Facilitate CABA,\ action by t frequency of "Frcnzodiazepincs" t frcqucncr.
.9 CJ- channel opening. l R E~ I sleep. Most Benzos, barbs, and alcohol all bind the
ha,·e long half-)i,·es and active metabolites C BA,\ receptor. ,,·hich is a ligand-gated Cl-
• 10
(exceptions [ATO!\ I]: Alprazolam, Triazolam, channel.
• 11
O xazepam, and ~ l idazolam are short ac ting O xazepam, Temazepam, and Lorazepam are
• 12 -+ higher addictive potential). metabolized O utside T he Liver
• 13 CLINICAL USE Anxiety, spas ticity, status epilcpticus (lorazepam
• 14 and diazepam), eclampsia, detoxification
• 15 (especially alcohol withdnl\vai- DTs), night
• 16 terrors, sleepwalking, general <~n est he tic
(amnesia, muscle relaxation), hypnotic
• 17
(insomn ia).
. 18
• 19
ADVERSE EFFECTS Dependence, additi,·e C ·s
depression effects
with alcohol. Less risk of respiratory depression
• 20 and coma than'' ith barbiturates.
• 21 Treat overdose with Aumazenil (compel it i' e

a
Lock
s
Suspend
8
End Bl ock
Item: 1 of 3 8 ~ 1 • M k -<:J 1>- Jil ~· !:';-~
QIO: 1443 ..L ar Pre v ious Next Lab fli!ltues Not es Calcula t o r
., , ~ t ~ •• •
1
and diazepam}, eclampsia, detoxification
•2
(especially alcohol withdrawal-DTs}, night
•3 terrors, sleepwalking, general anesthetic
·4 (amnesia, muscle relaxation), hypnotic
•5 (insomnia).
•6 ADVERSE EFFECTS Dependence, additi,·e C ·s
depression effects
•7 with alcohol. Less risk of respiratory depression
and coma than" ith barbiturates.
·8
Treat o,·erdose with Aumazenil (competiti'e
.9
antagonist at CABA benzodiazepine receptor).
• 10 Can precipitate seizures b) causing acute
• 11 benzodiazepine withdrawal.
• 12
• 13 FA17 p 515.1
• 14 Barbiturates Phenobarbital, pentobarbital, thiopental, secobarbital.
• 15 MECHANISM Facil itate CABAA action by f duration orct- channel openi ng, thus t neuron firing (barbidunltCs
• 16 f duration). Contraindicated in porph)'ria .

• 17 CLINICAL USE Sedative for anxiety, seizures, insomnia, induction of anesthesia (thiopental).
• 18 ADVERSE EFFECTS Respiratory and cardio,·ascular depression (can be fatal ); Cl S depression (can be exacerbated by
• 19 alcohol use}; dependence; drug interactions (induces cytochrome P-450).
Overdose treatment is supportive (assist respiration and maintain BP).
• 20
• 21

a
Lock
s
Suspend
8
End Bl ock
Item: 2 of 3 8 ~ 1 • M k -<:J 1>- Jil ~· !:';-~
QIO: 3550 ..L ar Pre v ious Next Lab fli!ltues Not es Calcula t o r

IAA]
& &
1
A 75-year-old white woman presents w ith a fever, altered menta l status, seizures, and nucha l rigidity . A
•2
lumbar puncture is performed, and the cerebrospinal fluid shows gram-positive bacilli that are 13-hemolytic
•3 and have a tumbl ing moti lity.
·4
•5 Which of the following medications is the treatment of choice for this patient's meningitis?
•6
:
•7 A. Acyclovir
·8
B. Amphotericin
.9
C. Ampicillin
• 10
• 11 D. Ceftriaxone
• 12 E. Vancomycin
• 13
• 14
• 15
• 16
• 17
• 18
• 19
• 20
• 21

a
Lock
s
Suspend
8
End Bl ock
Item: 2 of 38 ~. I • M k <:] t> al ~· ~
QIO: 3550 .l. ar Previous Next lab 'lifllues Notes Calculator

1 •

2 The correct answer is C. 66°/o chose this.


•3 The finding of gram-positive bacil li in the patient's cerebrospinal flu id (CS F) is diagnostic of men ingitis caused
by Listeria monocytogenes, an organism that commonly infects very young and very old patients. Listeria
•4
characteristical ly grows at low te mperatures . Ampicillin is the most appropriate antibiotic for Listeria men ingitis,
•5 and empiric coverage in these patients, even wh ile waiting for CSF analysis, is appropriate.
•6 listeria monocytogenes Ampicillin Cerebrospinal fluid Gram-positive bacteria Meningitis Antibiotics listeria Bacilli Bacillus (shape) Organism

•7 A is not correct. 2°/o chose this.


•8 Acyclovir is the appropriate treatment for herpes simplex virus meningitis. The cerebrospina l fluid resu lts are
•9 consistent with bacterial mening itis, and, thus, acyclovir is not the most appropriate treatment for this patient .
Aciclovir Cerebrospinal fluid Herpes simplex virus Meningitis Bacterial meningitis Herpes simplex Virus
• 10

· 11 B is not correct. 3°/o chose this.


• 12 Amphotericin is used for cryptococca l fungal meningitis, more commonly found in the immunocompromised . The
• 13
Gram sta in ind icates a bacterial cause .
Gram staining Meningitis Immunodeficiency New England Compounding Center meningitis outbreak Fungal meningitis Cryptococcus neoformans Fungus
• 14
Amphotericin B
• 15
• 16 Dis not correct. 21% chose this .
• 17 Ceftriaxone is an appropriate antibiotic for the broad-spectrum coverage of bacterial meningitis while
cerebrospinal fluid cultures and bacterial sensitivities are pend ing. However, when the cultures and sensitivities
• 18
of the bacteria become ava ilable, the antibiotic coverage options should be narrowed down to an antibiotic that
• 19 is specific for the responsible bacterial agent .
Ceftriaxone Cerebrospinal fluid Antibiotics Meningitis Bacterial meningitis Bacteria Broad-spectrum antibiotic
• 20
• 21

E is not correct. 8°/o chose this .
6
lock
s
Suspend
0
End Block
Item: 2 of 38 ~. I • M k <:] t> al ~· ~
QIO: 3550 .l. ar Previous Next lab 'lifllues Notes Calculator


- .. -- -- - . - ~

1
Amphotericin is used for cryptococca l fungal meningitis, more commonly found in the immunocompromised . The
2 Gram sta in indicates a bacterial cause.
•3 Gram staining Meningitis Immunodeficiency New England Compounding Center meningitis outbreak Fungal meningitis Cryptococcus neoformans Fungus

•4 Amphotericin B

•5
Dis not correct. 21% chose this .
•6
Ceftriaxone is an appropriate antibiotic for the broad-spectr um coverage of bacterial meningitis while
•7 cerebrospinal fluid cultures and bacterial sensitivities are pend ing. However, when the cultures and sensitivities
•8 of the bacteria become ava ilable, the antibiotic coverage options should be narrowed down to an antibiotic that
is specific for the responsible bacterial agent .
•9
Ceftriaxone Cerebrospinal fluid Antibiotics Meningitis Bacterial meningitis Bacteria Broad-spectrum antibiotic
• 10
E is not correct. 8°/o chose this.
· 11
Vancomycin is an appropr iate antibiotic for the broad -spectrum coverage of bacterial mening itis whi le
• 12
cerebrospinal fluid cultures and bacterial sensitivities are pend ing. However, when the cultures and sensitivities
• 13 of the bacteria become ava ilable, the antibiotic coverage options should be narrowed down to an antibiotic that
• 14 is specific for the responsible bacterial agent .
Vancomycin Cerebrospinal fluid Antibiotics Meningitis Bacterial meningitis Bacteria Broad-spectrum antibiotic
• 15
• 16
• 17 Bottom Line:
• 18 Listeria monocytogenes men ingitis is common in very young and very old patients and is treated with
• 19 ampicil lin .
listeria monocytogenes Ampicillin Meningitis listeria
• 20
• 21

6
lock
s
Suspend
0
End Block
Item: 2 of 3 8 ~ 1 • M k -<:J 1>- Jil ~· !:';-~
QIO: 3550 ..L ar Pre v ious Next Lab fli!ltues Not es Calcula t o r
& &
1

2 FA17 p 184.1
•3
Penicillinase-sensitive Amoxicillin, ampicillin; ami nopcnicillins.
·4 penicillins
•5 MECHANISM Same as penicillin. W ider spectrum; \ \l inoPenicillins are Ai\ IPed-up penicillin.
•6 penicillinase sensitive. Also combine" ith AmO ,icillin has greater O ral bioa,·ailability
.7 clavulanic acid to protect against destruction than ampicillin.
br ~-lactamase.
·8
.9 CLINICAL USE Extended-spectrum penicillin-11 in{luen;:ae, CO\ crage: ampicillin/amoxicillin III IELPSS
H pylori, E coli, Listeria monocytogenes, kill enterococci.
• 10
Proteus mirabilis, Salmonella, Shigella,
• 11
e nterococci.
• 12
ADVERSE EFFECTS Hypersensitivi tr reactions; rash;
• 13 pseudomembranous col it is.
• 14
MECHANISMOF RESISTANCE Pe nicillinase in bacteria (a type of ~-l ac tama sc)
• 15 clem·es ~-laetam ring .
• 16
• 17 FA17 p 131 .2
. 18 ~-hemolytic bacteria Gram (±) cocci. Complete lysis of RBCs - clear area surrounding colony on blood agar fJ. Include
• 19 the following organisms:
• 20 Staphylococcus aureus (catalase and coagulase (±))
Streptococcus pyogenes-group A strep (catalase 8 and bacitracin sensiti,·e)
• 21
• - t"'o • 4 I '• n 1 I ,......
, 1 I •• . . . \

a
Lock
s
Suspend
8
End Bl ock
Item: 2 of 3 8 ~ 1 • M k -<:J 1>- Jil ~· !:';-~
QIO: 3550 ..L ar Pre v ious Next Lab fli!ltues Not es Calcula t o r
& &
1
2 FA17 p 131 .2
•3 ~-hemolytic bacteria Gra m ~ cocci. Complete lysis of RBCs - clear area surrounding colony on blood agar fJ. Include
·4 the following organisms:
•5 Staphylococcus aureus (catalase and coagul ase ~)
•6 Streptococcus pyogenes-group A stre p (catalase 8 and bacit racin sensitive)
Streptococcus agalactiae-group B stre p (catalase 8 and bacitracin resistant)
•7

·8
.9
• 10
• 11
FA17 p 135.2
• 12
• 13 Listeria G ra m ~. fac ultative intracellular rod; <lcqu ired b} ingestion of unpaste uri zed dairy products and

• 14
monocytogenes cold deli meats, via transplacental transm ission, or by vaginal transmission during birth. Grows
well at refrigeration te mperat ures (4°- l0°C; "cold enrichme nt").
• 15
Forms "rocket tails" (red in fJ) via ac tin poly111erization that allow intracellular movement and cell-
• 16
to-cell spread across cell membranes, the re by avoiding antibody. Characteristic tumbl ing motility
• 17 in broth .
• 18 Can cause anmionitis, septicemia, and spontaneous abortion in pregnant women; granulom atosis
• 19 infantiseptica; neonatal me ningitis; me ningitis in immunocompromised patients; mild, self-
lim ited gastroente ritis in healthy individuals .
• 20
Treatme nt: ampicillin .
• 21

a
Lock
s
Suspend
8
End Bl ock
Item: 3 of 38 ~ 1 • M k -<:J 1>- Jil ~· !:';-~
QIO: 1939 ..L ar Pre v ious Next Labfli!llues Notes Calcula t o r
& &
1
A 5-year-old boy is brought t o the emergency department by his mother after she found him having
2
convulsions on the kitchen f loor. On the way to the hospital, he experienced another seizure . On admission,
•3 t he patient appears weak and disoriented and has no recollection of the event. He is given a dose of
·4 phenytoin for seizure prophylaxis; the plasma concentration after drug administration is 70 1-Jg/ ml. One hour after
•5
the drug is administered, his plasma concentration is 50 1-Jg/ ml; 2 hours after drug administration, it is 30 1-Jg/ ml.
•6
What will the plasma concentration be 3 hours after the drug's administration?
•7

·8 :

.9
A. 10 1-Jg/ ml
• 10 B. 15 1-Jg/ ml
• 11 C. 20 1-Jg/ml
• 12
D. 25 1-Jg/ml
• 13
• 14
E. 5 1-Jg/ml
• 15
• 16
• 17
• 18
• 19
• 20
• 21

a
Lock
s
Suspend
8
End Bl ock
Item:3of38 ~. , . M k <:] t> al ~· ~
QIO: 1939 .l. ar Previous Next Lab 'lifllues Notes Calculator

1 •

2 The correct answer is A. 78°/o chose this.


Based on the information given, t his dr ug is exhibiting zero-order elimination . I n zero- order elimination, the
3
concentrat ion of a drug decreases linearly wit h t ime, at a constant rate; the rate of the reaction is independent
•4 of the concentrat ion of the reactant . I n first-order eliminat ion, a constant percentage of the reactant is
•5 metabolized. Zero-order kinetics usually occu rs when enzymes are satu rated by the react ants and cannot
•6 increase the rate any further. In this scena r io, the rate of elim inat ion can be calcu lated as follows: Rate of
eliminat ion = (change in plasma concentrat ion) I (time) = (70 iJglmL- 50 iJglmL) I ( 1 h) = 20 iJglm Lih .
•7
Therefore, 3 h after drug admin istration, there wou ld be 10 iJglm L rema ining . The most common examples of
•8 drugs that have zero-order kinet ics are Phenyt oin, Ethanol, and A spi rin, wh ich can be remembered by this
•9 menmonic: zero PEAs for me .
Biological half-life Rate equation Blood plasma Reagent Chemical kinetics Metabolism Enzyme
• 10

· 11 B is not correct. 6°/o chose this.


• 12 This is a calculat ion error. If the drug were undergoing fi rst order kinet ics, its concent ration would be ha lved to
• 13 15 iJglm L from 30 iJglm L.
Rate equation Kinetics (physics) Chemical kinetics
• 14
• 15 C is not correct. 11% chose this .
• 16
This is a calculat ion error. After deter mining the rate of eliminat ion, t he concentration wou ld be reduced by
20 iJglm L; 30 iJglmL - 20 iJglmL resu lts in 10 iJglm L.
• 17
D is not correct. 4°/o chose this .
• 18
This is a calculat ion error. The rat e of reduct ion of the serum concentration decreases substant ia lly in the
• 19
question by 20 iJglmL at each t ime interval; a mere 5 iJglm L reduction is not enough .
• 20 Blood plasma Serum (blood)

• 21
• E is not correct. 1°/o chose this .
6
lock
s
Suspend
0
End Block
Item:3of38 ~. , . M k <:] t> al ~· ~
QIO: 1939 .l. ar Previous Next Lab 'lifllues Notes Calculator
- - • - • - • •• -• • • - - n · w• • .- • •• - - • - • • • - - •- •
1
drugs that have zero-order kinet ics are Phenyt oin, Ethanol, and A spi rin, wh ich can be remembered by this
2 menmonic: zero PEAs for me.
3 Biological half-life Rate equation Blood plasma Reagent Chemical kinetics Metabolism Enzyme

•4
B is not correct. 6°/o chose this.
•5 Th is is a calculat ion error. If the drug were undergoing first order kinet ics, its concent ration would be ha lved to
•6 15 1-Jg/mL from 30 1-Jg/mL.
Rate equation Kinetics (physics) Chemical kinetics
•7
•8 C is not correct . 11% chose this .
•9 Th is is a calculat ion error. After determ ining the rate of eliminat ion, t he concentration wou ld be reduced by
• 10 20 1-Jg/mL; 30 1-Jg/mL - 20 1-Jg/mL resu lts in 10 1-Jg/mL.
· 11 D is not correct. 4°/o chose this.
• 12 Th is is a calculat ion error. The rat e of reduct ion of the serum concentration decreases substant ia lly in the
question by 20 1-Jg/mL at each t ime interval; a mere 5 1-Jg/mL reduction is not enough .
• 13 Blood plasma Serum (blood)
• 14
E is not correct. 1°/o chose this .
• 15
Th is is a calculat ion error. 5 1-Jg/mL is less than 10 1-Jg/mL, which is the serum concentration after a 20 1-Jg/mL
• 16
reduction in zero-order kinetics .
• 17 Rate equation Chemical kinetics Blood plasma Kinetics (physics)

• 18
• 19
Bottom Line :
• 20
In zero -order el imination, the concent ration of a drug decreases linearly with t ime .
• 21

6
lock
s
Suspend
0
End Block
Item: 3 of 38 ~ 1 • M k -<:J 1>- Jil ~· !:';-~
QIO: 1939 ..L ar Pre v ious Next Labfli!llues Notes Calcula t o r
& &
1
2 FA17 p226.1

3 Elimination of drugs

·4 Zero-order Rate of elimination is constant regardless of CP Capacity-! imited elimination.


•5 elimination (ie, constant amount of drug eliminated per PK \ . (A pea is round, shaped like the "0" in
unit time). CP l linearly" ith time. Examples /.ero-order.)
•6
of drugs-Phenytoin, Ethanol, and \ spirin (at
•7
high or toxic concentrations).
·8
First-order elimination Rate of elimination is directly proportional Flow-dependent elimination.
.9 to the drug concentration (ic, constant
• 10 fraction of drug eliminated per unit time).
• 11 CP l exponentially with time. Applies to most
• 12
drugs.

• 13 Zero-order elimination First-order elimination


[ m1nat1on ;te sop. I ic
• 14
• 15 .g 2U/h
~ Time of t112 isconstant
• 16 ~ Time of t11z.J, as as concentration .j.
g
v
concentration .j.
• 17
"'~ '-----::----ll 2 Ulh ' =_._2U/h
• 18 .!!! FI'SI t112 > Fintt112 • 1
C>.
en 1
• 19 2 1
0
• 20
• 21
• Time thl Time (hi

a
Lock
s
Suspend
8
End Bl ock
Item: 4 of 3 8 ~ 1 • M k -<:J 1>- Jil ~· !:';-~
QIO: 1471 ..L ar Pre v ious Next Lab fli!ltues Not es Calcula t o r
& &
1
A 5-year-old boy is brought to the emergency department by his mother after telling her he felt ill from
2
consuming "a lot of the little black berries from the bush in the back yard." Soon after eating the berries, he
3 began to feel very thirsty and felt his heart "beating fast." His mother noticed that he was f lushed and his
•4 pupils were enlarged. Upon arrival, the boy is mu m bling incoherent ly and is very agitated . His temperature is 38°C
•5
( 100.4°F) and blood pressure is 105/70 mm Hg. Physical examination shows dry, f lushed skin. ECG shows sinus
tachycardia with a rate of 180/min .
•6
•7
Which of the following is the best treatment for this boy's condition?
·8
:
.9
A. Atropine
• 10
B. Clonidine
• 11
• 12 C. Metoprolol
• 13 D. Neostigm ine
• 14
E. Physostigmine
• 15
• 16
• 17
• 18
• 19
• 20
• 21

a
Lock
s
Suspend
8
End Bl ock
Item:4of38 ~. , . M k <:] t> al ~· ~
QIO: 1471 .l. ar Previous Next lab 'lifllues Notes Calculator

2
The correct answer is E. 51 °/o chose this.
The boy is exhibiting signs and symptoms of atropine overdose. There are a number of poisonous plants that
3
contain anticholinerg ic alkaloids such as atropine and scopolamine. Atropa bel/adona, also known as dead ly
4 nightshade, has bell-shaped purp le flowers and sh ining black ber ries that can be mistaken for edible
•5 blackberries. The plant produces the toxin atropine, which acts as an antagonist at chol inergic receptors and
•6 causes a variety of systemic effects that are best remembered with the mnemonic: Blind as a bat (myd r iasis),
Red as a beet (flushed skin), Mad as a hatter (de lirium and disorientation), Hot as a hare (fever by inhibition
.7
of sweat glands), and Dry as a bone (decreased sal iva and tear production) . Other system ic effects of atropine
•8 overdose include constipation and urinary retention .
•9 Physostigmine acts by reversibly inhibiting acetylchol inesterase, thus increasing the amount of acetylcho line
• 10 avai lable at the cholinergic receptor. It is used in the acute t reatment of atropine overdose in the setting of
· 11 severe central nervous system toxicity (eg, ag itation and seizure) and hemodynamic instability. However, its
administration can easi ly induce cholinerg ic toxicity (eg, bradycard ia and diarr hea), so its use is limited to
• 12
severe cases of atropine toxicity. Many cases can be t reated with supportive care, the use of activated charcoa l,
• 13 and benzod iazepines, but this patient's instabil ity necessitates the use of physostigmine .
• 14 Atropa belladonna Anticholinergic Hyoscine hydrobromide Mydriasis Physostigmine Atropine Activated carbon Bradycardia Urinary retention
• 15 Central nervous system Cholinergic Acetylcholine Constipation Acetylcholine receptor Acetylcholinesterase Benzodiazepine Acetylcholinesterase inhibitor Diarrhea
• 16 Receptor antagonist Alkaloid Solanaceae Saliva Mnemonic Delirium Nervous system Solanum Toxin Erythema Flushing (physiology) Tears
• 17
Receptor (biochemistry) Fever Drug overdose Epileptic seizure Toxicity Atropa Sweat gland Neurotoxin Charcoal Beetroot
• 18
A is not correct. 18% chose this .
• 19
Atropine is an antichol inergic that is responsible for the symptoms seen in this patient. As a resu lt, further
• 20
administration of this agent wou ld not be indicated .
• 21 Anticholinergic Atropine

6
lock
s
Suspend
0
End Block
Item:4of38 ~. , . M k <:] t> al ~· ~
QIO: 1471 .l. ar Previous Next lab 'lifllues Notes Calculator

1 •
A is not correct. 18% chose this.
2
Atropine is an antichol inergic that is responsible for the symptoms seen in this patient. As a resu lt, further
3 administration of this agent would not be indicated.
4 Anticholinergic Atropine

•5
B is not correct. 7°/o chose this .
•6
Clonidine, an orreceptor agon ist, acts to diminish central adrenergic outflow and is used to treat hypertension.
•7 It does not act at cholinergic receptors and has no role in the treatment of atropine toxicity.
Clonidine Atropine Cholinergic Hypertension Agonist Toxicity Adrenergic
•8
•9 C is not correct. 9°/o chose this .
• 10 Metoprolol is a cardioselective ~-b l ocker that is used to treat hypertension . Th is drug does not act at the
· 11 cholinergic receptor and is not indicated for treating atropine overdose .
Metoprolol Atropine Cholinergic Hypertension Acetylcholine receptor Receptor (biochemistry)
• 12
• 13 D is not correct. 15% chose this .
• 14 Neostigmine is a reversible acetylcholinesterase inhibitor similar to physostigmine; however, it has poor
penetration of the central nervous system and therefore is not indicated for treating cases of atropine overdose
• 15
with central nervous system toxicity .
• 16 Acetylcholinesterase inhibitor Physostigmine Atropine Neostigmine Acetylcholinesterase Central nervous system Nervous system Toxicity Enzyme inhibitor

• 17 Neurotoxin

• 18
• 19
Bottom Line:
• 20
Atropine overdose can lead to symptoms of antichol inergic excess, including tachyca rdia, delirium, mydriasis,
• 21
• fli 1c:hinn nf l'h~'> c:k-in ::~nri rlrv mn1 11'h c:;vmnl'nmc: r::~n h~'> ri'>VI'> r<:~'>ri wil'h inhihil'nrc: nf l'hl'> ::ll"'l'>i'v l rhnlinl'><:i'l'> r::l<:~'>

6
lock
s
Suspend
0
End Block
Item:4of38 ~. , . M k <:] t> al ~· ~
QIO: 1471 .l. ar Previous Next lab 'lifllues Notes Calculator

1 •
- - - . ..
Clonidine, an orreceptor agon ist, acts to diminish central adrenergic outflow and is used to treat hypertension .
2
It does not act at cho linergic receptors and has no role in the treatment of atropine toxicity.
3 Clonidine Atropine Cholinergic Hypertension Agonist Toxicity Adrenergic

4
C is not correct. 9°/o chose this.
•5
Metoprolol is a cardioselective ~-b l ocker that is used to treat hypertension . Th is drug does not act at the
•6 chol inergic receptor and is not indicated for treating atropine overdose .
•7 Metoprolol Atropine Cholinergic Hypertension Acetylcholine receptor Receptor (biochemistry)

•8 D is not correct. 15% chose this .


•9 Neostigmine is a reversib le acetylcho linesterase inhibitor simi lar to physostigmine; however, it has poor
• 10 penetration of the centra l nervous system and therefore is not indicated for treating cases of atropine overdose
with central nervous system toxicity.
· 11
Acetylcholinesterase inhibitor Physostigmine Atropine Neostigmine Acetylcholinesterase Central nervous system Nervous system Toxicity Enzyme inhibitor
• 12
Neurotoxin
• 13
• 14
• 15 Bottom Line:
• 16 Atropine overdose can lead to symptoms of antichol inergic excess, including tachyca rdia, delirium, mydriasis,
• 17 flushing of the skin, and dry mouth. Symptoms can be reversed with inhibitors of the acetylcho linesterase
enzyme, such as physostigmine, so as to increase the amount of acetylcholine available for bind ing at the
• 18
acetylchol ine receptors .
• 19 Anticholinergic Mydriasis Physostigmine Atropine Acetylcholine Enzyme Tachycardia Acetylcholinesterase Xerostomia Drug overdose Delirium
Receptor (biochemistry) Flushing (physiology) Acetylcholine receptor
• 20
• 21

6
lock
s
Suspend
0
End Block
Item: 4 of 3 8 ~ 1 • M k -<:J 1>- Jil ~· !:';-~
QIO: 1471 ..L ar Pre v ious Next Lab fli!ltues Not es Calcula t o r
& &
1

2 FA17 p232.1
3 Cholinomimetic agents
4 DRUG ACTION APPLICATIONS

•5 Direct agonists
•6 Bethanechol Activates bowel and bladder smooth muscle; Postoperative ileus, neurogenic ileus, urinar~
.7 resistant to AChE, no nicotinic acti,·ity. retention
"Bethany, call (bethanechol) me to activate
·8
rour bowels and bladder."

.9
Carbachol Carbon copy of acetylcholine (but resistant to Constricts pupil and relie\'es intraocular
• 10
AChE). pressure in open-angle glaucoma
• 11
Methacholine Stimulates m uscarinic receptors in airway when Challenge test for diagnosis of asthma
• 12 inhaled.
• 13 Pilocarpine Contracts ciliary muscle of eye (open-:mgle Potent stimulator of sweat, tears, and saliva
• 14 glaucoma), pupillary sphincter (closed-a ngle Open-angle and closed-angle glaucoma,
• 15 glaucoma); resistant to AChE, can cross blood- xerostomia (Sjogren syndrome)
• 16
brain barrier (tertiary amine). "You cry, drool,
an d swea t on your ' p•., ow.'"
• 17
Indirect agonists (anticholinesterases)
. 18
Galantamine, t ACh. Al:rheimer disease (AI7heimer patients gallantly
• 19
donepezil, swim down the ri,er)
• 20
rivastigmine
• 21
• Frlrnnhnnium t Ar.h

a
Lock
s
Suspend
8
End Bl ock
Item: 4 of 3 8 ~ 1 • M k -<:J 1>- Jil ~· !:';-~
QIO: 1471 ..L ar Pre v ious Next Lab fli!ltues Not es Calcula t o r
& &
1 Indirect agonists (anticholinesterases)
2 Galantamine, t ACh. Al:rhei mer disease (AI7heimer patients gallantly
3 d onepezil, swim down the ri' er)
4 rivastigmine
•5 Edrophonium t Ch. Historicalh- used to diagnose myasthenia gra,·is;
•6 replaced by anti-.\ ChR Ab (anti-acetylcholine
.7
receptor antibody) test.
Neostigmine t ACh. Postoperative and neurogenic ileus and
·8
r\eo CKS = :\o CN'S penetration (quaternary urinary retention, myasthenia gravis,
.9
amine). reversal of neuromuscular junction blockade
• 10 (postoperative).
• 11
Physostigmine t ACh. Ph~ sostigmine "ph) \Cs" atropine Antidote for anticholinergic toxicity; phrcely
• 12 overdose. (freely) crosses blood-brain barrier - Cl S
• 13 (tertiary ami ne).
• 14 Pyridostigmine t ACh; t muscle strength. Pyridostigm inc gets Myasthenia gravis (long acting); does not
• 15 rid of myasthenia gravis. penetrate CNS (quaternary ami ne).
• 16 Note: With all cholinom imetic agents, watch for exacerbat ion of COPD, asthma, and peptic ulcers when giving to susceptible
• 17 patients.
. 18
• 19 FA17 p 233.2
Atropine luscarinic antagonist. Used to treat brad) cardia and for ophthalmic applications.
• 20
• 21 ORGAN SYSTlM ACTION NOTES

a
Lock
s
Suspend
8
End Bl ock
Item:4of38 ~ ., . M k <:] t> al ~· ~
QIO: 1471 .l. ar Previous Next lab 'lifllues Notes Calculator

1 • •

2 FA17 p 233.2
Atropine luscarinic antagonist. Used to treat bradycardia and for ophthalmic applications.
3
4 ORGAN SYSTEM ACTION NOTES

•5 Eye t pupil dilation, cycloplegia Blocks DU\1BBeLSS in cholinesterase


•6 Airway ' secretions inhibitor poisoning. Does not block excitation
of skeletal muscle and C S (mcdie~ted by
•7 Stomach ' acid secretion nicotinic receptors).
•8 Gut ' motility
•9 Bladder ' urgency in cystitis
• 10 Side effects:
ADVERSE EFFECTS t body temperature (clue to ' sweating);
· 11 rapid pulse; dry mouth; dry, fl ushed skin; Ilot as a hare
• 12 cycloplegia; constipation; disorientation Dry as a bone
• 13 Can cause acute angle-closure glaucoma in Red as a beet
elderly (clue to mydriasis), urinary retention Blind as a bat
• 14
in men with prostatic hyperplasia, and lad as a hatter
• 15 hyperthermia in infants. Jimson weed (Datura) -+ gardener's pupil
• 16 (mydriasis due to plant alkaloids)
• 17

• 18 FA17p233.1
• 19 Muscarinic antagonists
• 20 DRUGS ORGAN SYSTEMS APPLICATIONS

• 21 Atropine, Eye Produce mydriasis and cycloplegia.


• •

6
lock
s
Suspend
0
End Block
Item: 4 of 3 8 ~ 1 • M k -<:J 1>- Jil ~· !:';-~
QIO: 1471 ..L ar Pre v ious Next Lab fli!ltues Not es Calcula t o r

1 ( y p )
2
3 FA17 p233.1

4 Muscarinic antagonists
DRUGS ORGAN SYSTEMS APPLICATIONS
•5
•6
Atropine, Eve

Produce mydriasis and cycloplegia.
homatropine,
•7
tropicamide
·8
Benztropine, CK Parkinson disease ("park my Bcn1").
.9 trihexyphenidyl Acute dystonia.
• 10
Glycopyrrolate C l, respiratory Parenteral: preoperative use to reduce airwar
• 11 secretions.
• 12 Oral: drooling, peptic ulcer.
• 13 Hyoscyamine, Cl Antispasmodics for irritable bowel syndrome.
• 14 dicyclomine
• 15 lpratropium, Respiratory COPD, asthma ("' pray I can breathe soon!").
• 16
tiotropium

• 17 Oxybutynin, Genitourinary Reduce bladder spasms and urge urinary


solifenacin, incontinence (O\'Cractive bladder).
• 18
tolterodine
• 19
Scopolamine CNS lotion sickness.
• 20
• 21

a
Lock
s
Suspend
8
End Bl ock
Item:5of38 ~. , . M k <:] t> al ~· ~
QIO: 3478 .l. ar Previous Next lab 'lifllues Notes Calculator


1
2
A 29-year-old woman complaining of chest pa in is brought to the emergency department via ambu lance. She I"' Aj
was at a night club when the pa in began . During the interview, she is obviously agitated and breath less when A
3 answer ing questions. Upon examination, the patient is diaphoretic, and her pupils are mydriatic. Pu lse is
4 130/min, blood pressure is 150/110 mm Hg, and respiratory rate is 25/min. An ECG is questionable for ischem ic
changes, and cardiac enzymes are elevated . Metopro lol, aspirin, mor phine, and nitroglycerin are adm inistered to
0 5
the patient. She subsequently develops a blood pressure of 180/145 mm Hg and loses consciousness.
0 6
7
Wh ich of the fo llowing illicit substances is most likely responsible for th is patient's presentation?
0

0 8
:
0 9
A. Lyserg ic acid diethylamide
0 10
B. Cocaine
· 11
0
12 C. Heroin
0
13 D . Ketamine
0 14
E. Phencyclidine
0 15
0
16
0 17
0
18
0 19
0 20
0
21

6
lock
s
Suspend
0
End Block
Item:5of38 ~. , . M k <:] t> al ~· ~
QIO: 3478 .l. ar Previous Next lab 'lifllues Notes Calculator

1 •
The correct answer is B. 77°/o chose this.
2
This patient presents with chest pain and like ly recreational drug ingestion. In this scenario, cocaine use shou ld
3 be high ly suspected . Cocaine intoxication man ifests with impai red judgment, pupillary dilation, hallucinations
4 ( including tactile), paranoid ideations, and may cause angina. Notably, treatment with ~-b l ocke rs of a patient
5 with cocaine intoxication can preceiptate a hypertensive event seconda ry to unopposed a-ad renergic
stimu lation, causing vasoconstriction and increased blood pressu re. Corona ry vasospasm also occu rs by this
•6
mechanism. The best t reatment fo r a patient with cocaine intoxication is a benzodiazepine, with a
•7 nondihydropyr idine calcium-channel blocker such as verapamil or di ltiazem used to control tachyca rdia.
•8 Benzodiazepine Cocaine intoxication Cocaine Beta blocker Hallucination Mydriasis Pupillary response Angina pectoris Alcohol intoxication Paranoia

•9 Substance intoxication Vasodilation

• 10
A is not correct. 6°/o chose this.
· 11 Lysergic acid diethylamide (LSD) intoxication causes perceptual distortion (visual, auditory), depersonal ization,
• 12 anxiety, pa ranoia, and psychosis .
lysergic acid diethylamide Depersonalization Psychosis Paranoia Anxiety Alcohol intoxication Substance intoxication Sound
• 13
• 14 C is not correct. 5°/o chose this .
• 15 Heroin is an opioid med ication that causes euphoria, respirato ry and CNS depression, gag reflex, pupillary
• 16 constr iction (pinpo int pupils), and seizu res (overdose) . The classic triad of opioid overdose is coma, pinpoint
pupils, and respiratory depression .
• 17
Pupillary reflex Pharyngeal reflex Central nervous system Miosis Heroin Major depressive disorder Depression (mood) Reflex Pharmaceutical drug
• 18
Epileptic seizure
• 19
D is not correct. 8°/o chose this .
• 20
Ketamine is an N-methyl-o-aspartate (NMDA) receptor antagonist that is often illicitly used for its hal lucinogen ic
• 21
• ;::~nri P 11nhnr ir nrnnPrtipc; I'PrmPri " riic;;::~c;c;nr i ;o~l'htP " TnYiril'v r;o~n inr l11rlP n;o~r;o~nnirl rlP I11c;innc; ;::~n i l';::~l'i nn

6
lock
s
Suspend
0
End Block
Item:5of38 ~. , . M k <:] t> al ~· ~
QIO: 3478 .l. ar Previous Next lab 'lifllues Notes Calculator

1 p p ' e p y I p
Pupillary reflex Pharyngeal reflex Central nervous system Miosis Heroin Major depressive disorder Depression (mood) Reflex Pharmaceutical drug
2
Epileptic seizure
3
4 D is not correct. 8°/o chose this.
5 Ketamine is an N-methyl- o-aspartate (NMDA) receptor antagonist that is often illicitly used for its hal lucinogen ic
•6 and euphoric properties, termed "disassociative." Toxicity can include paranoid de lusions, agitation,
hypertension, and ocu lar disturbances like nystagmus. Administration of metopro lol, aspirin, nitroglycerin, or
•7
morphine in ketamine use would not cause hypertension as seen in this patient .
•8 MDMA Euphoria Attention deficit hyperactivity disorder

•9
E is not correct. 4°/o chose this .
• 10
Phencyclidine ( PCP) intoxication causes violence, impulsivity, psychomotor agitation, nystagmus,
· 11 tachycardia, hypertension, analgesia, psychosis, delirium, and seizures .
Psychomotor agitation Nystagmus Tachycardia Psychosis Analgesic Hypertension Epileptic seizure Phencyclidine Delirium Substance intoxication Impulsivity
• 12
• 13 Alcohol intoxication Psychomotor retardation Psychomotor learning

• 14
• 15
Bottom Line:
• 16
Cocaine blocks reuptake of seroton in and norepinephrine. Common presenting symptoms are dilated pupils,
• 17 tachycardia, and elevated blood pressure . Giving a ~-bloc ker can cause unrestricted a-activity, causing
• 18 hypertension. A nondihydropyridine agent such as verapamil is preferred to control heart rate and blood
• 19
pressure in cocaine overdose .
Paroxetine Serotonin Norepinephrine Cocaine Reuptake
• 20
• 21

6
lock
s
Suspend
0
End Block
Item: 5 of 3 8 ~ 1 • M k -<:J 1>- Jil ~· !:';-~
QIO: 3478 ..L ar Pre v ious Next Lab fli!ltues Not es Calcula t o r
& &
1

2 FA17 p 540.1

3
Psychoactive drug intoxication and withdrawal
DRUG INTOXICATION WITHDRAWAL
4
Depressants
5
'onspecific: mood elevation, l am.iety, l\onspecific: anxiet~, tremor, seizures,
•6 . .
sedation, behavioral disinhibition, respiratory 1nsomma.
.7
depression.
·8
Alcohol Emotionallabilit}. slurred speech, ataxia, Time from last drink:
.9 coma, blackouts. Serum Y·glutam} ltransferase 3- 36 hr: minor symptoms similar to other
• 10 (GGT)-sensitive indicator of alcohol use. depressants
• 11 AST value is twice LT value. 6-48 hr: withdrawal seizures
• 12 12-48 hr: alcoholic hallucinosis (usually visual)
48-96 hr: delirium tremens (DTs) in 5% of
• 13
cases
• 14
Treatment: benzodiazepines .
• 15
Opioids Euphoria, respiratory and C S depression, Sweating, dilated pupils, piloerection ("cold
• 16 l gag reAex, pupillar>' constriction (pinpoint turkey"), fever, rhinorrhea, yawning, nausea,
• 17 pupils), seizures (overdose). t-.lost common stomach cramps, diarrhea ("Au-like" symptoms).
. 18 cause of drug overdose death. Treatment: Treatment: long-term support, methadone,
• 19
naloxone. buprenorphine.
• 20
Barbiturates Low safety margin, marked respiratory Delirium, life-threatening cardiovascular
depression. Treatment: symptom management collapse.
• 21
• (pu ::.~~i~t rPmir::.tinn t RP\

a
Lock
s
Suspend
8
End Bl ock
Item: 5 of 3 8 ~ 1 • M k -<:J 1>- Jil ~· !:';-~
QIO: 3478 ..L ar Pre v ious Next Lab fli!ltues Not es Calcula t o r

1 p p
2 Barbiturates Low safety margin, marked respiratory Delirium, life-threatening cardiovascular
3 depression. Treatment: symptom management collapse.
(eg, assist respiration, t BP).
4
5
Benzodiazepines Greater safety margin. taxia, minor Jeep disturbance, depression, rebound anxiety,
respiratory depression. Treatment : Oumazenil setzure.
•6
(benzodiazepine receptor antagonist, but
.7 rarely used as it can precipitate seizures).
·8 Stimulants
.9
Nonspecific: mood elen1tion, psychomotor 'onspecific: post-use "crash," including
• 10 agitation, insomnia, cardiac arrh) thmias, depression, lethargy, t appetite, sleep
• 11 taclwcardia, anxietr. disturbance. ,·ivid nightmares.
' '
• 12 Amphetamines Euphoria, grandiosity, pupillary dilation,
• 13 prolonged wakefulness and attention,
• 14 hypertension, tachycardia, anorexi<1, par<Jnoia,
fever. Severe: cardiac arrest, seizures .
• 15
Treatment: benzodiazepines for agitation and
• 16
SetZtlfeS.
• 17
Cocaine lmpaired judgment, pupillary dilation,
. 18 hallucinations (including tactile), paranoid
• 19 ideations, angina, sudden cardiac death .
• 20 Treatment: a -blockers, benzodiazepincs.
P-blockers not recommended.
• 21
• . . ..,.,. .. .... ...
a
Lock
s
Suspend
8
End Bl ock
Item: 5 of 3 8 ~ 1 • M k -<:J 1>- Jil ~· !:';-~
QIO: 3478 ..L ar Pre v ious Next Lab fli!ltues Not es Calcula t o r
& &
1
Caffeine Restlessness, t diuresis, muscle twitching. lleadache, difficulty concentrating, Au-like
2 symptoms.
3 Nicotine Restlessness. Irritability, anxiety, restlessness, difficulty
4 concentrating. Treatment: nicotine patch,
5 gum, or lozenges; bupropion/varcnicl inc.
•6 Phencyclidine iolence, impulsi,i ty, psychomotor agitation,
.7 nystagmus, tachycardia, h) pertension,
·8
analgesia, psychosis, delirium, sci ~ures.
Trauma is most common complication.
.9
Treatment: benzodiazcpi ncs, rapid-acting
• 10 antipsychotic.
• 11
Lysergic acid Perceptual distortion (visual, auditory),
• 12 diethylamide (LSD) depersonalization, anxiety, paranoia,
• 13 psychosis, possible flashbacks.
• 14 Marijuana Euphoria, anxiety, paranoid delusions, lrriLabiliLy, anxiety, depression, insomnia,
• 15 (cannabinoid) perception of slowed time, impaired judgment, restlessness, ~ appetite.
• 16
social withdrawal, t appetite, dry mouth,
conjunctival injection, hallucinations.
• 17
Pharmaceutical form is dronabinol: used
. 18 as antiemetic (chemotherapy) and appetite
• 19 stimulant (in AIDS).
• 20 MDMA (ecstasy) Hallucinogenic stimulant: euphoria, Depression, fatigue, change in appetite, difficulty
• 21 disinhibition, hyperacti' it), distorted sensory concentrating, anxiety.

a
Lock
s
Suspend
8
End Bl ock
Item: 5 of 3 8 ~ 1 • M k -<:J 1>- Jil ~· !:';-~
QIO: 3478 ..L ar Pre v ious Next Lab fli!ltues Not es Calcula t o r

1 ) y
2 Midodrine Autonomic insufficiency and postural
3 hypotension. May exacerbate supine
hrpertension.
4
Mirabegron Urinary urge incontinence or O\eracti ve bladder.
5
•6 Norepinephrine l lypotension, septic shock.

.7
Phenylephrine Hypotension (\·asoconstrictor), ocular procedures
·8
(mydriatic), rhinitis (decongestant).
.9
Indirect sympathomimetics
• 10
Amphetamine Indirect general agonist, reuptake inhibitor, also Narcolepsy, obesity, ADHD.
• 11
releases stored catecholami nes
• 12
Cocaine Indirect general agonist, reuptake inhibitor Causes \'asoconstriction and local anesthesia.
• 13
l c,·er give ~-bl ockers if coca ine intoxication is
• 14 suspected (can lead to unopposed a 1 activation
• 15 and extreme hypertension).
• 16 Ephedrine Indirect general agonist, releases stored 1asal
decongestion (pseudoephedrine), urinary

I
• 17 catecholamines incontinence, hypotension.
. 18
• 19 FA17 p 544.3
• 20
Antidepressants
• 21

a
Lock
s
Suspend
8
End Bl ock
Item: 5 of 3 8 ~ 1 • M k -<:J 1>- Jil ~· !:';-~
QIO: 3478 ..L ar Pre v ious Next Lab fli!ltues Not es Calcula t o r
& &
1
2 FA17 p 544.3

3 Antidepressants
4
NORADRENERGIC SEROTONERGIC
5
•6 AXON AXON
~ MAOinhlbltors
•7
MAO ~ ~
-o- -f MAO Metabolites
MelabolltM - - -
·8
.9 I Buproplon \
• 10 NE - - 0 00 ¢-----5-HT
• 11 (/o )
• 12
• 13
• 14
/
0
I 0
1

• 15
• 16
• 17
• 18
• 19
• 20
• 21

a
Lock
s
Suspend
8
End Bl ock
Item: 6 of 3 8 ~ 1 • M k -<:J 1>- Jil ~· !:';-~
QIO: 420 1 ..L ar Pre v ious Next Lab fli!ltues Notes Calcula t o r

IAA]
& &
1
A 20-year-old woman sees her physician, reporting that she has been experiencing severe, uni lateral,
2
throbbing headaches several t imes each month. The headaches are associated wit h nausea and occasional
3 vomiting. She has used ibuprofen in the past but finds it does not control the headaches well. The physician
4 recommends another drug that the patient shou ld take at the immediate onset of her symptoms.
5
•6 What is the mechanism of action of her new medication?
•7 :

·8 A. ~ 2 -Agonist
.9 B. Dopamine receptor blocker
• 10
C. Monoamine oxidase inhibitor
• 11
D. Serotonin receptor agonist
• 12
• 13 E. Serotonin receptor ant agonist
• 14
• 15
• 16
• 17
• 18
• 19
• 20
• 21

a
Lock
s
Suspend
8
End Bl ock
Item: 6 of 38 ~. I • M k <:] t> al ~· ~
QIO: 4201 .l. ar Previous Next lab 'lifllues Notes Calculator

1 •

2 The correct answer is D. 54°/o chose this.


3 Sumatriptan is an agonist of the SHT16; 10 receptor that is used in the treatment of acute migraine and cluster
headaches. Action on these receptors causes vasoconstriction of cerebral and meningeal vessels. Other
4
"triptans" work on the SHT16; 10 receptor and include almotriptan, eletriptan, rizatriptan, frovatriptan (frequently
5 prescribed given its long duration of action), and zolmitriptan. A rare adverse effect of SHT16; 10 agon ism is
6 coronary vasoconstriction.
Zolmitriptan Frovatriptan Eletriptan Sumatriptan Rizatriptan Almotriptan Migraine Vasoconstriction Agonist Triptan Cluster headache Serotonin Adverse effect
•7
•8 Coronary vasospasm Receptor (biochemistry)

•9 A is not correct. 8°/o chose this .


• 10 ~rAgonists such as albuterol cause bronchodilation and are used in the treatment of asthma .
Asthma Bronchodilator Salbutamol
· 11
• 12 B is not correct. 7°/o chose this .
• 13 Haloperidol is an antipsychotic drug that blocks dopamine receptors . It is used in the treatment of sch izophrenia
• 14 and psychosis .
Haloperidol Dopamine Antipsychotic Schizophrenia Psychosis Dopamine receptor
• 15
• 16 C is not correct. 8°/o chose this .
• 17 Seleg iline is an antiparkinsonian drug that selectively inhibits MAO-B and increases the avai lability of dopamine .
Selegiline Antiparkinson medication Monoamine oxidase B Dopamine
• 18
• 19 E is not correct. 23% chose this .
• 20 Ondansetron is a SHT3 receptor antagonist and is used in the treatment of nausea and vomiting associated with
chemotherapy .
• 21
• Ondansetron Chemotheraov Nausea Serotonin Vomitina Receotor antaaonist Antaaonist Receotor lbiochemistrv)

6
lock
s
Suspend
0
End Block
Item: 6 of 38 ~. I • M k <:] t> al ~· ~
QIO: 4201 .l. ar Previous Next lab 'lifllues Notes Calculator

1 •

2 A is not correct. 8°/o chose this.


3 ~rAgonists such as albuterol cause bronchodilation and are used in the treatment of asthma.
Asthma Bronchodilator Salbutamol
4
5 B is not correct. 7°/o chose this.
6 Haloperidol is an antipsychotic drug that blocks dopamine receptors . It is used in the treatment of sch izophrenia
and psychosis .
•7
Haloperidol Dopamine Antipsychotic Schizophrenia Psychosis Dopamine receptor
•8
•9
C is not correct. 8°/o chose this .
Seleg iline is an antiparkinsonian drug that selectively inhibits MA0- 8 and increases the avai lability of dopamine . t
• 10
Selegiline Antiparkinson medication Monoamine oxidase B Dopamine
· 11
E is not correct. 23% chose this .
• 12
Ondansetron is a SHT3 receptor antagonist and is used in the treatment of nausea and vomiting associated with
• 13
chemotherapy .
• 14 Ondansetron Chemotherapy Nausea Serotonin Vomiting Receptor antagonist Antagonist Receptor (biochemistry)

• 15
• 16
Bottom Line:
• 17
Sumatr iptan can be used as abortive treatment for migraine headaches. It is a serotonin receptor agon ist that
• 18
must be used with caution in coronary artery disease patients because it can cause vasoconstriction .
• 19 Sumatriptan Migraine Serotonin 5-HT receptor Vasoconstriction Coronary artery disease Serotonin receptor agonist Agonist Coronary circulation

• 20
• 21

6
lock
s
Suspend
0
End Block
Item: 6 of 3 8 ~ 1 • M k -<:J 1>- Jil ~· !:';-~
QIO: 420 1 ..L ar Pre v ious Next Lab fli!ltues Notes Calcula t o r
& &
1
FA17 p 516.3
2
Triptans Sumatriptan
3
MECHANISM 5-HT18110 agonists. Inhibit trigeminal nene A SlJ\ to" res tier TRIPs AJ\d falls on rom
4
acti\'ation; pre\'ent ,·asoactive peptide release; head.
5 induce vasoconstriction.
6 CLINICAL USE Acute migraine, cluster he<Jdache attacks.
7
Coronary ,·asospasm (contraindicated in
0

ADVERSE EFFECTS
o8 patients with CAD or Prinzmetal angina),
.9 mild paresthesia, serotonin S) ndrome (in
• 10 combination with other 5-HT agonists).
• 11

• 12 FA17 p 488.1
• 13 Headaches Pain due to irritation of structmes such as the dura, cranial nerves, or extracranial structures. More
0 14 common in females, except cluster headaches.
• 15 CLASSIFICATION LOCALIZATION DURATION DESCRIPTION TREATMENT
0
16 Clustera Unilateral 15 min-3 hr; Repetitive brief headaches. Acute: sumalriplan, 100% 0 2
0
17
repetitive Excrucialing periorbita I Prophylaxis: verapamil
pain with lacrimation and
. 18
rhinorrhea. l\ lay present with
• 19 Horner syndrome.
• 20
Tension Bilateral > 30 min Steady pain. o photophobia Analgesics, 1 SAIDs,
• 21 (trpically 4-6 or phonophobia. 1 o aura. acetaminophen;

a
Lock
s
Suspend
8
End Bl ock
Item: 6 of 38 ~. I • M k <:] t> al ~· ~
QIO: 4201 .l. ar Previous Next lab 'lifllues Notes Calculator

1 • •

2 FA17 p488.1

3 Headaches Pain due to irritation of structures such as the dura, cranial nerves, or extracranial structures. More
4 common in females, except cluster headaches.
CLASSIFICATION LOCALIZATION DURATION DESCRIPTION TREATMENT
5
Cluster'~ Unilateral 15 min-3 hr; Repetitive brief headaches. Acute: sumatriplan, 100% 0 2
6
repetitive Excruciating periorbital Prophylaxis: verapamil
•7
pain with lacrimation and
•8 rhinorrhea. May present with
•9 Horne r syndrome.
0 10 Tension Bilateral > 30 min Steady pain. o photophobia Analgesics, NSAIDs,
· 11 (typically 4-6 or phonophobia. No aura. acetaminophen;
hr); constant amit riptyline for chronic
• 12
pam
• 13
Migraine Unilateral 4-72 hr Pulsating pain with Acute: NSAIDs, triptans,
• 14
nausea, photophobia, or dihydroergotamine
0 15 phonophobia. May have Prophylaxis: lifestyle changes
• 16 "aura." Due to irritation of (eg, sleep, exercise, diet),
• 17 C1 V, meninges, or blood ~-blockers, calcium channel

• 18 vessels (release of substance blockers, amitriptyline,


P, calcitonin gene-related topiramate, valproate.
• 19
peptide, vasoactive peptides). POUND - Pulsatile, O ne-day
20
0
duration, Unilateral, Nausea,
• 21 D isabling •

6
lock
s
Suspend
0
End Block
Item: 6 of 3 8 ~ 1 • M k -<:J 1>- Jil ~· !:';-~
QIO: 420 1 ..L ar Pre v ious Next Lab fli!ltues Notes Calcula t o r
I " W , t t I I t , II •
1
repetitive Excruciating periorbita I Prophylaxis: verapamil
2
pain with lacrimation and
3 rhinorrhea. ~ lay present with
4 Horner syndrome.
5 Tension Bilateral > 30 min Steady pain. o photophobia .\ nalgesics, NSAIDs,
6 (trpically 4-6 or phonophobia. ' o aura. acetaminophen;
0 7 hr); constant amitriptyline for chronic
pam
o8
Migraine Unilateral 4-72 hr Pulsating pain with Acute: l\SAIDs, triptans,
.9
nausea, photophobia, or dihydroergotamine
• 10
phonophobia. Mar ha,·e Prophylaxis: lifestyle changes
0
11 "aura." Due to irritation of (eg, sleep, exercise, diet),
0
12 CN V, meninges, or blood P-blockers, calcium channel
0
13 ,·essels (release of substance blockers, amitriptyline,
0 14
P, ca lcitonin gene-related topiramate, valproate.
peptide, vasoactive peptides). POUND- Pulsatile, O ne-day
• 15
duration, Unilateral, Nausea,
0
16 D isabling
0
17 Other causes of headache include subarachnoid hemorrhage ("worst headache of my li fe"), meningitis, hydrocepha lus,
0
18 neoplasia, giant cell (temporal) arteritis.
a compare'' ith trigeminal neuralgia, which produces repetitive, unilateral, ~hootin g pain in the distribution ofC V that
1
0 19
• 20 lasts (typicallr) for< l minute (note: first-line therapy is carbamazepine).
0
21

a
Lock
s
Suspend
8
End Bl ock
Item: 7 of 3 8 ~ 1 • M k -<:J 1>- Jil ~· !:';-~
QIO: 4 0 5 0 ..L ar Pre v ious Next Lab fli!ltues Not es Calcula t o r

IAA]
& &
1
A 36-year-old man with AIDS is brought to the emergency department with altered mental status and
2
personal ity changes. He has been wandering the streets and is not oriented to place or t ime. Lumbar
3 puncture reveals bloody cerebrospinal fluid in the absence of a traumatic tap, w ith 80 WBCs/mm 3 consisting
4 mostly of lymphocytes; a protein level of 35 mg/dl; and glucose level of 70 mg/dl. Wh ile in the emergency
department, he also experiences a self-limited seizure.
5
6
Empiric treatment with which of the following medications should be started immediately?
0 7
o8 :
A. Acyclovir
.9
• 10 B. Amphotericin
• 11 C. Penicillin G
• 12
D. Vancomycin
• 13
0 14
E. Zidovudine
• 15
0
16
0
17
• 18
• 19
• 20
• 21

a
Lock
s
Suspend
8
End Bl ock
Item: 7 of 38 ~. I • M k <:] t> al ~· ~
QIO: 4050 .l. ar Previous Next lab 'lifllues Notes Calculator

1 •

2 The correct answer is A. 60°/o chose this.


3 The cause of this patient's illness is herpes simplex vi r us (HSV). Cerebrospinal fluid (CSF) findings in viral
4 causes of mening it is and encephal itis include high WBC counts with lymphocytic or monocytic predominance and
5
norma l protein, glucose, and opening pressure. The absence of norma l brain function (a ltered mental status,
sensory or motor deficits) makes the diagnosis more likely to be encepha litis than mening it is. HSV-1
6 encepha litis tends to produce a bloody CSF without a traumatic tap. HSV- 1 may manifest with temporal lobe
7 involvement on imaging. Empiric treatment for HSV-1 infection is acyclovi r, which ha lts vira l repl ication;
•8 treatment should be initiated as soon as meningitis or encepha litis is suspected, without an obvious cause,
because early t reatment is associated with a significant decrease in morbidity and morta lity .
•9
Aciclovir Cerebrospinal fluid Meningitis Temporal lobe Encephalitis Herpes simplex virus Herpes simplex Protein Morbidity Altered level of consciousness Glucose
• 10
Monocyte Empiric therapy Brain Virus Viral replication
· 11
• 12
B is not correct. 21% chose this .
Amphotericin wou ld be used intratheca lly for fungal meningitis, as it does not cross blood-brain barrier. Fungal
• 13
mening it is is seen more in immunocompromised ind ividuals, and CSF find ings would show increased opening
• 14 pressure, increased lymphocytes, increased proteins, and decreased blood g lucose.
• 15 Meningitis Immunodeficiency Fungal meningitis New England Compounding Center meningitis outbreak Blood sugar Blood-brain barrier Fungus Glucose

• 16 Intrathecal administration Amphotericin B

• 17
C is not correct. 5°/o chose this .
• 18 Treatment of Neisseria meningitides consists of penici llin G or ceftriaxone.
• 19 Ceftriaxone Penicillin Benzylpenicillin Neisseria meningitidis Neisseria

• 20
D is not correct. 6°/o chose this .
• 21 Empiric coverage for bacterial meningitis consists of vancomycin and a th ird-generation cephalosporin such as •

6
lock
s
Suspend
0
End Block
Item: 7 of 38 ~. I • M k <:] t> al ~· ~
QIO: 4050 .l. ar Previous Next lab 'lifllues Notes Calculator

1 • mening it is is seen more in immunocompromised individuals, and CSF findings would show increased opening
2
pressure, increased lymphocytes, increased proteins, and decreased blood glucose.
Meningitis Immunodeficiency Fungal meningitis New England Compounding Center meningitis outbreak Blood sugar Blood-brain barrier Fungus Glucose
3
Intrathecal administration Amphotericin B
4
5 C is not correct. 5°/o chose this.
6 Treatment of Neisseria meningitides consists of penicillin G or ceftriaxone.
Ceftriaxone Penicillin Benzylpenicillin Neisseria meningitidis Neisseria
7
•8 D is not correct. 6°/o chose this .
•9 Empiric coverage for bacterial meningitis consists of vancomycin and a th ird-generation cephalosporin such as
ceftriaxone .
• 10
Cephalosporin Ceftriaxone Meningitis Vancomycin Bacterial meningitis
· 11
E is not correct. 8°/o chose this .
• 12
Zidovud ine is a reverse transcriptase inhibitor, specifically a nucleoside ana log reverse transcriptase inhibitor,
• 13
that is used to treat HIV infection; it wor ks by stopping the reverse transcriptase enzyme that HIV uses to
• 14 reproduce .
• 15 Zidovudine Reverse-transcriptase inhibitor Reverse transcriptase Enzyme HIV Nucleoside analogue Nucleoside Structural analog

• 16
• 17
Bottom Line:
• 18
HSV is the most likely causative organism of viral encephalitis in a patient with AIDS. It typical ly affects the
• 19 temporal lobe, causes bloody CSF on lumbar puncture, and is treated with acyclovir.
• 20 lumbar puncture Aciclovir Temporal lobe Encephalitis Herpes simplex virus HIV/AIDS Virus Viral encephalitis Cerebrospinal fluid

• 21

6
lock
s
Suspend
0
End Block
Item: 7 of 38 ~. I • M k <:] t> al ~· ~
QIO: 4050 .l. ar Previous Next lab 'lifllues Notes Calculator

1 • •
FA17 p 176.2
2
CSF find ings in me ningit is
3
OPEN ING PRESSURE CEll TYPE PROTEIN GlUCOSE
4
Bacterial t t PMNs t ~
5
Fungai/TB t t lymphocytes t ~
6
Viral ormal/t t lymphocytes ormal/t Normal
7
•8
•9 FA17 p 160.2

• 10 He rpesviruses Enveloped, OS, and linear viruses


· 11 VIRUS ROUTE OF TRANSMISSION CliNICAl SIGNIFICANCE NOTES
• 12 Herpes Respiratory Gingivostomatitis, keratoconjunctivitis fJ, lost common cause of sporadic
• 13 simplex secretions, saliva herpes labial is [lJ, herpetic whitlow on finger, encephalitis, can present as altered
virus-1 temporal lobe encephalitis, esophagitis, mental status, seizures, and/or
• 14
erythema multiforme. aphasia.
• 15
Herpes Sexual contact, Herpes genitalis [!1, neonatal herpes. Latent in sacral ganglia. Viral
• 16
simplex perinatal meni ngitis more common with
• 17 virus-2 HSV-2 than with HSV-L
• 18 Varicella- Respiratory Varicella-zoster (chickenpox [!], shingles 0 ), Latent in dorsal root or trigeminal
• 19 Zoster virus secretions encephalitis, pneumonia. ganglia; C ' V1 branch
• 20 (HHV-3) Most common complication of shingles is post- involvement can cause herpes
• 21
herpetic neuralgia . zoster ophthalmicus.
• •

6
lock
s
Suspend
0
End Block
Item: 7 of 3 8 ~ 1 • M k -<:J 1>- Jil ~· !:';-~
QIO: 4 0 5 0 ..L ar Pre v ious Next Lab fli!ltues Not es Calcula t o r
& &
1
Varicella- Respiratory Varicella-zoster (chickenpox (!], shingles 0 ), Latent in dorsal root or trigeminal
2 secretions
Zoster virus encephalitis, pncumon ia. ganglia; C ' 1 branch
3 (HHV-3) Most common complication of shingles is post- i1wolvement can cause herpes
4 herpetic neuralgia. zoster ophthalmicus.
5 Epstein-Barr Respiratory Mononucleosis-fe,·er, hepatosplenomegaly, Infects B cells through C02!.
6 virus (HHV-4) secret ions, pharyngitis, and lymphadenopath} (especially Atrpicallymphoc) tes on peripheral
sali\'a· aka posterior cervical nodes ). Avoid contact sports blood smear [!J-not infected B
7 '
"kissing disease." until resolution due to risk of splenic rupture. cells but reacti,·e crtotoxic T cells.
·8
(common in Associated with lymphomas (eg, endemic $ ~ fonospottest-heterophile
.9 teens, young Burkitt lymphoma), nasopharyngeal antibodies detected by agglutination
• 10 adults) carcinoma (especiall) Asian adu lts), of sheep or horse RBCs.
• 11 lymphoproliferati,·e disease in transplant Use of amoxicillin in mononucleosis
patients . can cause characteristic
• 12
maculopapular rash .
• 13
Cytomegalo- Congenital Mononucleosis (8 Monospot) in Infected cells have characteristic
• 14
virus (HHV-5) transfusion, immunocompetent patients; infection in "owl eye" inclusions Cl
• 15 sexual contact, immunocompromised, especially pneumonia Latent in mononuclear cells.
• 16 saJi,·a, urine, in transplant patients; esophagitis; AIOS
• 17 transplant retinitis ("~ight omega lovirus"): hemorrhage,
. 18
cotton-wool exudates, vision loss .
Congenital C~l
• 19
Human Saliva Roseola infantum (exanthem subitum): high Roseola: fever first, Rmic (checks)
• 20
herpes- fevers for several days that can cause seizures, later.
• 21 viruses 6 followed by diffuse macular rash HHV-7-Iess common cause of

a
Lock
s
Suspend
8
End Bl ock
Item: 7 of 3 8 ~ 1 • M k -<:J 1>- Jil ~· !:';-~
..L
QIO: 4 0 5 0

1
ar Pre v ious Next
. Lab fli!ltues
:
Not es
.
Calcula t o r

Human Saliva Roseola infantum (exanthem subitum): high Rmeola: fever first, Rmie (checks)
2
herpes- fevers for several days that can cause seizures, later.
3 viruses 6 followed br diffuse macular rash HHV-7-less common cause of
4 and 7 roseola.
5 Human Sexual contact Kaposi sarcoma (neoplasm of endothelial cells). Can also affect Gl tract and lungs.
6 herpesvirus Seen in HJV/AIDS and transplant patients.
7 8 Darkkiolaceous plaques or nodule~ D
representing vascular proliferations.
·8
.9
• 10
• 11
• 12
• 13
• 14
• 15
• 16
• 17
• 18
• 19
• 20
• 21

a
Lock
s
Suspend
8
End Bl ock
.9
FA17 p 197.3
• 10
Acyclovir, famciclovir, valacyclovir
• 11
MECHANISM Guanosine analogs. Monophosphorylated by liS '/VZV thymidine kinase and not phosphorylated
• 12
in uninfected cells ..... few ackerse effects. Triphosphate formed br cellular enzymes. Preferentially
• 13 inhibit viral Dt A polymerase by chain termination .
• 14
CLINICAL USE HSV and VZV. Weak activity against P.BV. o activity against CMV. Used for HSV-
• 15 induced mucocutaneous and genital lesions as well as for encephalitis. Prophylaxis in
• 16 immunocompromised patients. 1 o effect on latent forms of HSV and VZV. Valacyclovir, a
• 17 prodrug of acyclovir, has better oral bioavailability.
For herpes zoster, use famciclovir.
• 18
• 19
ADVERSEEFFECTS Obstructive crystall ine nephropathy and acute renal failure if not adequately hydrated .
• 20 MECHANISMOFRESISTANCE J\lutated viral thymidine kinase .
• 21

a
Lock
s
Suspend
8
End Block
Item: 8 of 3 8 ~ 1 • M k -<:J 1>- Jil ~· !:';-~
QIO: 420 2 ..L ar Pre v ious Next Lab fli!ltues Notes Calcula t o r

IAA]
& &
1
A 12-year-old chi ld was admitted to the hospita l for his th ird episode of genera lized convulsions that have
2
been ongoing for more than 30 minutes. After giving the patient intravenous diazepam, another chronic
3 antiseizure med ication is begun . Months later, the ch ild develops swol len gums, double vision, facia l rash,
4 and generalized joint swell ing.
5
6 Which of the following most accurately describes the mechanism of action of this patient's chronic antiseizure
medication?
7
·8 :
A . Blocks calcium flux at thalamic T-type calcium channels
.9
• 10 B. Blocks sodium channels
• 11 C. Increases the duration of chloride channel opening
• 12
D. Increases the frequency of chloride channel openings
• 13
• 14
E. Inhibits the reuptake of serotonin at the presynaptic neuron
• 15
• 16
• 17
• 18
• 19
• 20
• 21

a
Lock
s
Suspend
8
End Bl ock
Item:8of38 ~. , . M k <:] t> al ~· ~
QIO: 4202 .l. ar Previous Next lab 'lifllues Notes Calculator

1 •

2 The correct answer is B. 66°/o chose this.


3 Having been in a seizure for more than 30 minutes, this patient is in status epilepticus, which is a medical
emergency . After the emergent administration of intravenous diazepam, the patient can be started on
4
phenytoin, which is common ly used as prophylaxis for status epilepticus. Phenytoin binds to the inactive for m of
5 sodium channels. By impeding sodium channels, phenytoin is able to provide a use-dependent effect, inh ibiting
6 sodium conductance and altering the transm ission of the epileptic focus. Phenytoin is also considered first-l ine
7 treatment for tonic-clonic seizu res and can be used in the treatment of simple and complex partia l seizures.
Adverse effects include gingival hyperplasia, megaloblastic anemia, teratogenesis, and systemic lupus
8
erythematosus-like symptoms .
•9 Megaloblastic anemia Diazepam Phenytoin Status epilepticus Gingival enlargement Teratology Anemia Intravenous therapy Epileptic seizure Sodium channel

• 10 Medical emergency Generalised tonic-clonic seizure Partial seizure Epilepsy Systemic lupus erythematosus Preventive healthcare Sodium lupus erythematosus

· 11 Seizure types Therapy

• 12
A is not correct. 11% chose this .
• 13
This describes the mechan ism of action of ethosuximide, which is common ly used in the treatment of absence
• 14 (petite mal) seizu res. Common adverse effects include lethargy, headache, urticaria, and Stevens-Johnson
• 15 synd rome .
Ethosuximide Stevens-Johnson syndrome Urticaria lethargy Headache Epileptic seizure Mechanism of action
• 16
• 17 C is not correct. 10% chose this .
• 18 This describes the mechan ism of action of ba r biturates. These med ications are common ly used for anxiety,
• 19
seizures, insomnia, and induction of anesthesia .
Insomnia Mechanism of action Barbiturate Anesthesia Anxiety Epileptic seizure
• 20
• 21 D is not correct. 9°/o chose this .

6
lock
s
Suspend
0
End Block
Item:8of38 ~. , . M k <:] t> al ~· ~
QIO: 4202 .l. ar Previous Next lab 'lifllues Notes Calculator

1 • Ethosuximide Stevens-Johnson syndrome Urticaria lethargy Headache Epileptic seizure Mechanism of action

2 C is not correct. 10% chose this.


3 This describes the mechan ism of action of ba r biturates. These med ications are common ly used for anxiety,
4 seizures, insomnia, and induction of anesthesia .
Insomnia Mechanism of action Barbiturate Anesthesia Anxiety Epileptic seizure
5
6 D is not correct. 9°/o chose this.
7 This describes the mechan ism of action of benzod iazepines, wh ich can be used acutely for status epi lepticus .
Other uses include anxiety, alcohol detoxification (includ ing delirium tremens), night terrors, and sleepwal king .
8
Delirium tremens Status epilepticus Alcohol detoxification Benzodiazepine Anxiety Alcoholic beverage Mechanism of action Night terror Delirium Sleepwalking
•9
Alcohol
• 10

· 11 E is not correct. 4°/o chose this.


• 12
Selective serotonin reuptake inhibitors, as the name suggests, inhibit the reuptake of serotonin at the
presynaptic neuron, thereby increasing the life of serotonin at the synaptic cleft. These drugs are commonly
• 13 used for depression and obsessive compulsive disorder, not seizures .
• 14 Obsessive compulsive disorder Neuron Serotonin Synaptic cleft Chemical synapse Selective serotonin reuptake inhibitor Reuptake Epileptic seizure

• 15 Major depressive disorder Depression (mood)

• 16
• 17
Bottom Line:
• 18
Pharmacolog ic management of status epi lepticus begins with intravenous diazepam, followed by intravenous
• 19
phenytoin. Intravenous phenobarbital may be added for the treatment of resistant status epi lepticus .
• 20 Diazepam Phenobarbital Phenytoin Status epilepticus Intravenous therapy Pharmacology

• 21

6
lock
s
Suspend
0
End Block
Item: 9 of 38 ~ 1 • M k -<:J 1>- Jil ~· !:';-~
QIO: 5078 ..L ar Previous Next Lab fli!ltues Notes Calculator

IAA]
& &
1
A 24-year-old woman is brought to t he emergency department by her husband after she accidenta lly
2
overdosed on her antiseizure med icat ion. Her husband cannot recal l the name of the medication. The
3 patient's medical history includes a recent abortion of a fetus w ith spina bifida. On physical examination, the
4 patient is an overweight, extremely lethargic woman who appears confused and is oriented only to self.
5
6 What abnormality might this patient have as a result of her medication toxicity?
7 :

8 A. Decreased serum creatinine


.9 B. Elevated liver transaminases
• 10
C. Elevated serum pH
• 11
D. Gingival hyperplasia
• 12
• 13 E. I ncreased neutroph il count
• 14
• 15
• 16
• 17
• 18
• 19
• 20
• 21

a
Lock
s
Suspend
8
End Block
Item:9of38 ~. , . M k <:] t> al ~· ~
QIO: 5078 .l. ar Previous Next lab 'lifllues Notes Calculator

1 •

2 The correct answer is B. 52°/o chose this.


3 The patient described is like ly taking valproate as her antiseizure med ication. Her history of a recently aborted
fetus with spina bifida is consistent with va lproate's teratogen ic effect of causing neural tube defects. In
4
addition, the patient's body habitus suggests the possibility that she is also taking a medication, such as
5 valproate, that causes weight ga in. Acute valproate overdose typica lly manifests with mild lethargy that
6 resolves without t reatment; however, more severe cases can lead to worse lethargy, respiratory depression,
7 and hypotension. Other symptoms can include GI distress and neu rologic symptoms such as tremors or clonus.
This patient's lethargy and confusion are also suggestive of acute valproate overdose. Carnitine is used to
8
reverse va lproate toxicity. Valproate carries an FDA black-box warn ing about its hepatotoxicity, which has
9 caused reported fatal it ies. Patients taking valproate should have their liver function checked at baseline and
0 10 frequently after starting the medication, particularly during the first 6 months when toxicity is most likely. It is
also a folate antagonist, though th is is no longer thought to be the major mechanism for its teratogen icity. It
· 11
should be noted that phenytoin and carbamazepine also deplete fo late and cause weight gain, though classical ly
0
12 valproate is associated with folate dep letion leading to spina bifida.
0
13 Anticonvulsant Carbamazepine Spina bifida Phenytoin Clonus Teratology Hypoventilation Valproate Hepatotoxicity Hypotension Folic acid Receptor antagonist

0 14 Antifolate Neural tube Boxed warning Carnitine liver Fetus Morphology (biology) Neural tube defect Major depressive disorder Folate deficiency lethargy

0 15 Food and Drug Administration Depression (mood) Toxicity Pharmaceutical drug Neurology Drug overdose

0
16
A is not correct. 4°/o chose this.
17
0

Valproate is almost exclusively metabol ized in the liver, with about 5% being metabolized by the kidneys. It is
0
18 thus un likely to affect kidney function and serum creatinine levels. In addition, if it were to adversely affect
0 19 kidney function, one would expect an increase in serum creatinine, not a decrease.
Creatinine Valproate liver Kidney Renal function Blood plasma Metabolism Serum creatinine Drug metabolism Serum (blood)
0 20
0
21 C is not correct. 6°/o chose this.

6
lock
s
Suspend
0
End Block
Item:9of38 ~. , . M k <:] t> al ~· ~
QIO: 5078 .l. ar Previous Next lab 'lifllues Notes Calculator

1 •
C is not correct . 6°/o chose this.
2
Severe va lproat e toxicity can lead t o respiratory depression, lead ing to hypercapn ia and acidosis. Therefore, a
3 low serum pH is more likely in t his patient than a high serum pH.
4 Hypercapnia Hypoventilation PH Valproate Acidosis Blood plasma Toxicity Major depressive disorder Depression (mood) Serum (blood)

5 D is not correct. 33% chose this.


6 Ging iva l hyperplasia is a potential t oxicity of phenyt oin along with nystagmus, ataxia, diplopia, sedation, and a
7 systemic lupus erythematosus- like syndrome . It can also cause peripheral neu ropathy or megaloblastic anemia .
The adverse effect of the med ication is feta l hydant oin syndrome .
8
Fetal hydantoin syndrome Megaloblastic anemia Phenytoin Diplopia Nystagmus Gingival enlargement Hydantoin Ataxia Peripheral neuropathy Anemia Sedation
9
Hyperplasia Systemic lupus erythematosus Toxicity Adverse effect
0 10

· 11 E is not correct. 5°/o chose this.


0
12
Valproate's most common hematologic side effect is th rombocyt openia, and it is usua lly not severe . In ra re
cases ( < 1% ) it can lead to ag ranu locyt osis. One would therefore expect the neutrophi l count to decrease, not
0
13 increase. Carbamazepine is anot her seizure medication associat ed with blood dyscrasias t hat is frequen t ly
0 14 tested on exams.
0 15 Carbamazepine Neutrophil Agranulocytosis Thrombocytopenia Valproate Adverse effect Side effect Hematology Epileptic seizure Pharmaceutical drug Dyscrasia

0
16
17
0
Bottom Line:
0
18
Valproate can cause neura l tube defect s in t he fet us. I t can also cause a ra re but serious hepatotoxicit y. Liver
0 19 enzymes shou ld be closely mon itored in pat ients taking va lproat e. Acute valproic acid overdose can lead to
0 20 lethargy, confusion, and vit al sign inst ability in severe cases, though most commonly reso lves on its own.
Valproate Hepatotoxicity Neural tube liver Fetus Neural tube defect lethargy liver function tests
0
21

6
lock
s
Suspend
0
End Block
Item: 9 of 38 ~ 1 • M k -<:J 1>- Jil ~· !:';-~
QIO: 5078 ..L ar Previous Next Lab fli!ltues Notes Calculator
& &
1
2 FA17 p514.1
3 Epilepsy drugs
4 GENERALIZED

5 ~

.....
< ~

6
C>
~
:z
C>
~
...... ........ "'
.....
:::>

..."':z... .,., ~
~
< v :::>:'1:;
~
:z ,_ ~

7 ""
....
< .....
C>
<
.........
<-
"' ... MECHANISM SIDE EFFECTS NOTES
8 Ethosuximide • Blocks thalamic T-t~-pe Ca~• Jo:l GI IIJ- Fthosm:imide Sucks to haYe Silent
9 .I channels causes Fatigue. C l distress. (absence) Seizures
lleadache, Itching (and
• 10 urticaria), and Stevens-
• 11 Johnson syndrome
Benzodiazepines •• t CJ\BA" action Sedation, tolerance, Also for eclampsia seizures (1st
• 12
(eg, diazepam, .I dependence, respiratory line is MgS04)
• 13 lorazepam, dcprc~~ion
midazolam)
• 14
Phenobarbital .I .I t CABA,\action Sedation, tolerance, Ist line in neonates
• 15 dependence. induction
• 16 of cytochrome P--!50,
• 17
• 18
Phenytoin,
fosphenytoin
.I •
.I
...
.I
Blocks Na+ channels; 1ero-
order kinetics
cardiorespiratory depression
\Jcurologic: nystagmus, diplopia, ataxia, sedation, peripheral
neuropathy. Dermatologic: hirsutism, Stevens-Johnson
5) ndrome, gingival hyperplasia, DRESS syndrome.
• 19
1\lusculoskeletal: osteopenia, SLE-Iike syndrome. llematologic:
• 20 megaloblastic anemia. Reproductive: teratogenesis (fetal
hydantoinsy ndrome). Other: cytochrome P-450 induction
• 21
• r ...e...~-~ ..... -•-oft • ./ Q),..,.l,,. I\...~ + ,.),..,.,, .,.nlr f"' in1ol"\ni..,. ..,. ..." ;,.. hl""',l l c-l l: ... .a . " .. .-..: ....oo....... : ... .,J ..... .n ........ J... : ...

a
Lock Suspend
s 8
End Block
Item: 9 of 38 ~ 1 • M k -<:J 1>- Jil ~· !:';-~
QIO: 5078 ..L ar Previous Next Lab fli!ltues Notes Calculator

1
&
Phenytoin, ./ • *** Blocks Na+ channels; 1ero- \Jeurologic: nystagmus, di plopia, ataxia, sedation, peripheral &

fosphenytoin ./ ./ order kinetics neuropathy. Ocrmatologic: hi rsutism, Stc,·cns-)ohnson


2
S) ndrome, gi ngival hyperplasia, DRESS syndrome.
3 l\lusculoskeletal: osteopenia, SLE-Iike syndrome. llcmatologic:
megaloblastic anemia. Reproductive: teratogenesis (fetal
4
hyda ntoin S) ndrome). Other: C)tochrome P-450 induction
5 Carbamazepine ./ Blocks a• channels Diplopia, ala\ia, blood 1st line for trigeminal neuralgia
6 ./ d) M:r.uia} (agranulocytosis,
aplastic anemia), Jj,er
7 to:-.icit). teratogenesis.
induction of C\ tochrome
8
P-450, SIADI i, Ste,ens-
9 Johnson }) ndrome
• 10 Valproic acid ./ • ./ t -a+ channel inactivation, C l distress. rare but fatal Also used for myoclonic seizures,
./ t CABA concentration hepatotoxicity (measure bipolar disorder, migraine
• 11 by inhibiting CAB!\ LF'Ts), pancreatitis, neural prophylaxis
• 12 transaminase tube defects, tremor, weight
gain, contraindicated in
• 13 pregnancy
• 14 Vigabatrin t CABA by irrcvcrsibl) Permanent visual loss (black
inhibiting CABA box warning)
• 15
transaminase
• 16 Gabapentin Primarily inhibits high-voltage- Sedation, ataxia Also used for periphera I
• 17 activa ted Ca 2• channels; neuropathy, postherpetic
designed as CABA analog neuralgia
• 18
Topiramate ./ ./ Blocks 'a+ channels, t C \ BA Sedntion, mental dulling, Also used for migraine
• 19 action kidney ~I ones, weight loss, pre,·ention
glaucoma
• 20
Lamotrigine ./ ./ ./ Blocks , oltage-gatcd 'a+ Ste\ens-Johnson S) ndrome
• 21 channels, inhibits the release (mw.t be titrated slo\\ l}·)

a
Lock
s
Suspend
8
End Block
Item: 9 of 38 ~ 1 • M k -<:J 1>- Jil ~· !:';-~
QIO: 5078 ..L ar Previous Next Lab fli!ltues Notes Calculator

1
&
induction of C) tochrome &

P-450, Slt\011, Ste\ens-


2 Johnson syndrome
3 Valproic acid ./ • ./ I 'a+ channel inactivation, C l distress. rare but fatal Also used for myoclonic seizures,
./ I GABA concentration hepatotoxicity (measure bipolar disord~r, migraine
4
by inhibiting GABA LFTs), pancreatitis, neural proph}iaxis
5 transaminase tube defects, tremor, weight
gain, contraindicated in
6 pregnancy
7 Vigabatrin I GA BA b} irrc\'crsibl) Perm.menl 'isuallos~ (black
8 inhibiting CAB.\ bo' "arning)
transaminase
9
Gabapentin ./ Primaril} inhi~its high-,oltage- Sedation. ata,ia Also used for peripheral
• 10 acti\'ated Ca-+ channels; neuropathy, postherpetic
designed as CABA analog neuralgia
• 11
Topiramate ./ ./ Blocks •a+ channels, I C \ B!\ Sedation, mental dulling. Also used for migraine
• 12 action kidney ~ Iones, weight loss, prevention
• 13 gbucoma
Lamotrigine ./ ./ ./ Blocks voltage-gated 1 a' Stevens-Johnson syndrome
• 14
channels, inhibits the release (must be titra ted slowly)
• 15 of glutamate
• 16 Levetiracetam ./ ./ Unknown; may modulate ~;1 1 igue,
drows iness,
CABA and glutamate release headache, neuropsychiatric
• 17 svmptoms (eg, personality
• 18 changes)
• 19 Tiagabine ./ I CABA by inhibit ing rcuptake

• 20
= 1st line; = lst line for acute; ••• = lst line for prophylaxis.

• 21

a
Lock
s
Suspend
8
End Block
Item: 10 of 38 ~. I • M k <:] t> al ~· ~
QIO: 1738 .l. ar Previous Next Lab 'lifllues Notes Calculator

1 •
A 59-yea r-old man comes to the physician complaining of muscle cramps and fatigue. Lab studies revea l an
2
elevated creatinine and potassium. A diagnosis of rena l fai lure is made and the patient is started on
3 hemodialysis. After 2 years of this the rapy, the patient undergoes rena l al lograft transplantation and is
4 placed on an immunosuppressive regimen to prevent rejection. His rena l function is mon itored in fo llow up visits.
Th ree months after t ransplantation, the patient arr ives to clinic and complains of painfu l vesicles and redness as
5
seen in the image. His lesions respond to acyclovir.
6
7

8
9
0 10
o ll
0
12
0
13
0 14
0 15
0
16
0 17 Image courtesy of National Institute on Aging
0
18
0 19 Wh ich of the fo llowing best describes the infection shown in t his image?
0 20 :
0
21

6
lock
s
Suspend
0
End Block
I mage courtesy of National Institute on Aging
9
• 10
Which of the following best describes the infection shown in this image?
• 11
:
• 12
• 13
• 14 A. The current symptoms only manifest once the virus has developed a novel RNA polymerase
• 15 B. The infective virions co nta in vi ral thym idine kinase
• 16
C. The vira l reverse transcriptase may be inhibited by acyclovir
• 17
• 18
D. The virus incorporates host cell proteases into its genome in order to cleave and activate the vira l
polyp rote in
• 19
E. Treatment with acyclovir is disrupting viral synt hesis of guanine nucleotides
• 20
• 21

a
Lock
s
Suspend
8
End Block
Item: 10 of 38 ~. I • M k <:] t> al ~· ~
QIO: 1738 .l. ar Previous Next Lab 'lifllues Notes Calculator

1 •

2
The correct answer is B. 45°/o chose this.
3
Herpes zoster is character ized by reactivation of a latent var icella-zoster infection. Or iginal infection is
4
character ized by chickenpox, after which the virus lives in the ganglia of spinal nerve roots. When the infection
5 is reactivated, as often happens in immunosuppressed patients, eruptive vesicles develop in a dermatomal
6 pattern and do not cross the midline (l ike that shown in the image). Acyclovir is the primary treatment of zoster
7
inf ection. Acyclovir is an inactive precursor that is activated by viral thymid ine kinase when absor bed by
inf ected cells. When activated, it for ms a guanine analog that resu lts in chain termination when read by viral
8 DNA polymerase.
9 Chickenpox Aciclovir Shingles Thymidine kinase DNA polymerase Thymidine Herpes simplex Varicella zoster virus Guanine Immunosuppression Ganglion

10 Chain termination Virus Vesicle (biology and chemistry) DNA Polymerase Infection Kinase

· 11
A is not correct. 5°/o chose this .
• 12
Reactivation of the patient's latent varicella-zoster vi r us infection is most li kely secondary to his
• 13 immunosuppression . Although herpesviruses can develop resistance to acyclovir by modifications to the viral
• 14 DNA polymerase, mutations in the viral RNA polymerase do not lead to an increased incidence of reactivation .
Aciclovir Varicella zoster virus DNA polymerase Immunosuppression Herpesviridae Shingles Virus RNA RNA polymerase DNA Infection Mutation
• 15
• 16 C is not correct. 15% chose this .
• 17 Reverse transcriptase is not present in varicella-zoster vir us. Reverse t ranscriptase is found in HIV .
Varicella zoster virus Reverse transcriptase Shingles Virus
• 18
• 19 D is not correct. 12% chose this .
• 20 Herpes zoster virus encodes its own proteases. Host cell proteases are not incorporated into the zoster DNA, nor
do they cleave the viral polyprotein .
• 21
ShinniP.c: HP.rnP.c: c:imniP.x Viruc: ProtP.::lC:P. ONA

6
lock
s
Suspend
0
End Block
Item: 10 of 38 ~. I • M k <:] t> al ~· ~
QIO: 1738 .l. ar Previous Next Lab 'lifllues Notes Calculator

1 •
A is not correct. 5°/o chose this.
2 Reactivation of the patient's latent varicella-zoster vi r us infection is most li kely secondary to his
3 immunosuppression . Although herpesviruses can deve lop resistance to acyclovir by modifications to the viral
4 DNA polymerase, mutations in the viral RNA polymerase do not lead to an increased incidence of reactivation.
Aciclovir Varicella zoster virus DNA polymerase Immunosuppression Herpesviridae Shingles Virus RNA RNA polymerase DNA Infection Mutation
5
6 C is not correct. 15% chose this.
7 Reverse transcriptase is not present in varicella-zoster vir us. Reverse t ranscriptase is found in HIV.
Varicella zoster virus Reverse transcriptase Shingles Virus
8
9 D is not correct. 12% chose this.
10 Herpes zoster virus encodes its own proteases. Host cell proteases are not inco rporated into the zoster DNA, nor
do t hey cleave the viral polyprotein.
· 11
Shingles Herpes simplex Virus Protease DNA
• 12
• 13
E is not correct. 23% chose this .
The action of acyclovir does not affect the production of the vi r ion's native nucleotides. Cha in termination occu rs
• 14
as long as the vi rus contains thymid ine kinase .
• 15 Aciclovir Thymidine kinase Thymidine Virus Nucleotide Kinase Chain termination

• 16
• 17
Bottom Line:
• 18
• 19
Acyclovir is a guanosine analog antivi ra l prodrug that is activated by vi ra l thymidine kinase, resu lting in the
termination of DNA synthesis .
• 20 Prodrug Aciclovir Thymidine kinase Thymidine Guanosine Antiviral drug DNA replication Structural analog DNA Virus Kinase

• 21

6
lock
s
Suspend
0
End Block
Item: 10 of 3 8 ~ 1 • M k -<:J 1>- Jil ~· !:';-~
QIO: 1738 ..L ar Pre v ious Next Labfli!llues Notes Calcula t o r
& &
1

2
FA17 p 197.3
Acyclovir, famciclovir, valacyclovir
3
MECHANISM Guanosine analogs. Monophosphorylated by I IS /VZV th) midine kinase and not phosphorylated
4
in uninfected cells ..... few ad,·erse effects. Triphosphate formed by cellular enzymes. Preferentially
5
inhibit ,-iraI DNA polymerase by chain termination.
6
CliNICAL USE HSV and VZV \ eak activity against EB . 1\o activity against CMV Used for HS -
7 induced mucocutaneous and genital lesions as " ell as for encephalitis. Prophylaxis in
8 immunocompromised patients. 1\o effect on latent forms of HSV and VZ . Valacyclovir, a
9 prod rug of aC}clovir, has better oral bioa,·ailability.
10
ro r herpes zoster, use famciclovir.
• 11 ADVERSEEFFECTS Obstructi,·e crystalline nephropathy and acute renal failure if not adequately hydrated.
• 12 MECHANISM OF RESISTANCE Mutated viral thymidine kinase.
• 13
• 14 FA1 7 p451 .1
• 15 Skin infections
• 16 Bacterial infections
• 17 Impetigo Very superficial skin infection. Usually fr0111S aureus or S pyogenes. Highly contagious. Honey-
. 18 colored crusting f'J.
Bullous impetigo : has bullae and is usually caused by S aureus.
• 19
• 20
Erysipelas Infection involving upper dermis and superfi cial lymphatics, usually from S pyogenes. Presents with
well-defined demarcation bel\1 een infected and normal skin
• 21
• \ ..... . &-.... . .... .... : ..... C•• l ..._ .............1: ..... ,.. : ..... ( ........ , : .... ..... .... r . .1........ - ...... ..1.......- : ... ......... .J ..... t.. ..... . , ..... .............. . ... a.:- ......... J 1..... .... 11 • • f ....... ._

a
Lock
s
Suspend
8
End Bl ock
Item: 10 of 38 ~. I • M k <:] t> al ~· ~
QIO: 1738 .l. ar Previous Next Lab 'lifllues Notes Calculator
- - -
1 • •

2
FA17 p451 .1
3
Skin infections
4
Bacterial infections
5
Impetigo Very superficial skin infection. Usually from S aureus or S pyogenes. Highly contagious. Honey-
colored crusting a.
6
7 Bullous impetigo I}] has bullae and is usua lly caused by S aureus.
8 Erysipelas Infection involving upper dermis and superficial lymphatics, usually from S pyogenes. Presents with
9 well-defined demarcation between infected and normal ski n ~-
10 Cellulitis Acute, painful, spreading infection of deeper dermis and subcutaneous tissues. Usually from
· 11 S pyogenes or S aureus. Often starts with a break in skin from trauma or another infection [!].
• 12 Abscess Collection of pus from a wal led-off infection within deeper layers of skin 0 . Offending organism is
• 13 almost always S aureus .
• 14 Necrotizing fasciitis Deeper tissue injury, usually from anaerobic bacteria or S pyogenes. Pain may be out of proportion
• 15 to exam find ings. Results in crepitus from methane and C02 production. "Flesh-eating bacteria."
Causes bullae and a purple color to the skin [J.
• 16
• 17
Staphylococcal scalded Exotoxin destroys keratinocyte attachments in stratum granulosum on ly (\·s toxic epidermal
skin syndrome necrolysis, which destroys epidermal-dermal junction). Characterized by fever and generalized
• 18
erythematous rash with sloughing of the upper layers of the epiderm is ~ that heals completely.
• 19 Et> I ikolsky sign. Seen in newborns and children, adults with renal insufficiency.
• 20 Viral infections
• 21 •
• He roes Heroes virus infections (HSVl and HSV2) of skin can occur anvwherc from mucosal surfaces to
6
lock
s
Suspend
0
End Block
Item: 10 of 3 8 ~ 1 • M k -<:J 1>- Jil ~· !:';-~
QIO: 1738 ..L ar Pre v ious Next Labfli!llues Notes Calcula t o r
&
. - &
1
Herpes Herpes ,-irus infections (I ISVI and l iS 2) of skin can occur anywhere from mucosal surfaces to
2
normal skin. These include herpes labial is, herpes genitalis, herpetic whitlow CJ (finger).
3
Molluscum Umbilicated papules caused by a poxvirus. While frequently seen in children, it may be scxua ll~
4
contagiosum transmitted in adults.
5
Varicella zoster virus Causes ,·aricella (chickenpox) and zoster (shingles). Varicella present~ with multiple crops of
6 lesions in ,·arious stages from vesicles to crusts. Zoster is a reacti,·ation of the ,-irus in dermatomal
7 distribution {unless it is disseminated).
8 Hairy leukoplakia Irregular, white, painless plaques on lateral tongue that cannot be scraped off . EB mediated.
9 Occurs in HIV-positi,·e patients, organ transplant recipients. Contrast with thrush (scrapable) and
10 leukoplakia (precancerous).
• 11

• 12
• 13
• 14
• 15
• 16
• 17
. 18
• 19
• 20
• 21

a
Lock
s
Suspend
8
End Bl ock
Item: 10 of 38 ~. I • M k <:] t> al ~· ~
QIO: 1738 .l. ar Previous Next Lab 'lifllues Notes Calculator
~- r .. , . ~

iiU • fiJ '.~· ... ,,...,\. RU ~ fl.l

2
3 FA17 p 160.2

4 Herpesviru ses Enveloped, OS, and linear viruses


5 VIRUS ROUTE OF TRANSMISSION CLINICAL SIGNIFICANCE NOTES
6 Herpes Respiratory Gingivostomatitis, keratoconjunctivitis fJ, lost common cause of sporadic
7 simplex secretions, saliva herpes labial is ll), herpetic whitlow on finger, encephalitis, can present as altered
8 virus-1 temporal lobe encephalitis, esophagitis, mental status, seizu res, and/or
erythema multiforme. aphasia.
9
10
Herpes Sexual contact, Herpes genitali s ~' neonatal herpes. Latent in sacral ganglia. Viral
simplex perinatal meni ngitis more common with
· 11
virus-2 HSV-2 than with HSV-L
• 12
Varicella- Respiratory Varicella-zoster (chickenpox [!], shingles 0 ), Latent in dorsal root or trigeminal
• 13 Zoster virus secretions encephalitis, pneumonia. ganglia; C V1 branch
• 14 (HHV-3) Most common complication of shingles is post- involvement can cause herpes
• 15 herpetic neuralgia . zoster ophthalmicus.
• 16 Epstein-Barr Respiratory Mononucleosis- fever, hepatosplenomegaly, Infects B cells through CD2L
• 17 virus (HHV-4) secretions, pharyngitis, <mel lymphadenopathy (especially Atypical lymphocytes on peripheral
saliva· aka posterior cervical nodes 1)). Avoid contact sports blood smear ~ - not infected B
• 18 '
"kissing disease," until resolution clue to risk of splenic rupture. cells but reactive cytotoxic T cells.
• 19 <±) fonospot test-heterophile
(common in Associated with lymphomas (eg, endemic
• 20 teens, young Burkitt lymphoma), nasopharyngeal antibodies detected by agglutination
• 21 adults) carcinoma (especially Asian adults), of sheep or horse RBCs.
• . .... •

6
lock
s
Suspend
0
End Block
Item: 10 of 3 8 ~ 1 • M k -<:J 1>- Jil ~· !:';-~
QIO: 1738 ..L ar Pre v ious Next Labfli!llues Notes Calcula t o r
& &
1
Cytomegalo- Congenital Mononucleosis (8 Monospol) in Infected cells have characteristic
2 virus (HHV-5) transfusion, immunocompetent palicnls; infec tion in "owl eye" inclusions C).
3 sexual contact, immunocompromised, especial!) pneumonia Latent in mononuclear cells.
4 saJi,·a, urine, in transplant patients; esophagitis; AIDS
transplant re tiniti~ ('\ightomega lovirus''): hemorrhage,
5
cotton-wool exudates, ' ision loss.
6
Congenital C \I V
7
Human Sali\'a Roseola infantum (exanlhem subitum): high Ro~eola: fever first, Ro~ie (checks)
8 herpes- fevers for several days that can cause seizures, later.
9 viruses 6 followed by diffuse macular ra~h HHV-7-less common cause of
10 and 7 roseola.
• 11 Human Sexual contact Kaposi sarcoma (neoplasm of endothelial cells). Can also affect GJ tract and lungs.
• 12 herpesvirus Seen in 1-IIV/AIDS and transplant patients.
8 Dark/violaceous plaques or nodules D
• 13
representing vascular proliferations.
• 14
• 15
• 16
• 17
. 18
• 19
• 20
• 21

I
a
Lock
s
Suspend
8
End Bl ock
Item: 10 of 3 8 ~ 1 • M k -<:J 1>- Jil ~· !:';-~
QIO: 1738 ..L ar Pre v ious Next Labfli!llues Notes Calcula t o r

1
&
-. b - &

Human Saliva Roseola infantum (exanthem subitum): high Ro~eola: fever first, Rmie (checks)
2
herpes- fevers for several days that can cause seizures, later.
3 viruses 6 followed by diffuse macular rash HHV-7-less common cause of
4 and 7 roseola.
5 Human exual contact Kaposi sarcoma (neoplasm of endothelial cells). Can also affect G l tract and lungs.
6 herpesvirus Seen in HIV/AIDS and transplant patients.
7
8 Dark/\'iolaceous plaques or nodules D
representing \'3Scular proliferations.
8
9
10
• 11

• 12
• 13
• 14
• 15
• 16
• 17
. 18

I
• 19
• 20
• 21

a
Lock
s
Suspend
8
End Bl ock
Item: 11 of 3 8 ~ 1 • M k -<:J 1>- Jil ~· !:';-~
QIO: 1467 ..L ar Pre v ious Next Labfli!llues Notes Calcula t o r
& &
1
A 66-year-old asymptomatic man presents for his annual health ma intanence examination . On funduscopic
2
examination, the left optic nerve appears markedly concave, and further work-up demonstrates an elevated
3 intraocular pressure. The physician places the patient on a medication, but the patient returns complaining of
4 sensitivity to light. On examination, accommodation is intact, but the patient's pupils appear more dilated than at
the previous visit.
5
6
Which of the following agents is most likely to cause the observed symptoms?
7
8 :
A. Acetylcholine
9
10 B. Atropine
• 11 C. Epinephrine
• 12
D. Physostigm ine
• 13
• 14
E. Pi locarpine
• 15
• 16
• 17
• 18
• 19
• 20
• 21

a
Lock
s
Suspend
8
End Bl ock
Item: llof38 ~. , . M k <:] t> al ~· ~
QIO: 1467 .l. ar Previous Next lab 'lifllues Notes Calculator

1 •

2 The correct answer is C. 34°/o chose this.


3 This patient has open-ang le glaucoma, wh ich is often asymptomatic and detected with regula r screening via
fundoscopic examination, which may revea l optic cupping, as in this patient. In addition, intraocula r tone would
4
be elevated. The presumed pathophysiology of the condition is increased production and/or decreased dra inage
5 of aqueous humor. Th is leads to increased intraocu lar pressure and thus progressive optic nerve atrophy that
6 produces a stereotypic pattern of visual loss-first affecting the periphery resu lting in tunnel vision and f ina lly
7
creating complete loss of vision.

8 The patient was provided a therapeutic that produced mydriasis but not cyclopleg ia (the loss of
accommodation) . Knowing this, the answer to the question must include a sympathomimetic or an
9
antichol inergic agent. Howeve r, an antichol inergic wou ld cause cycloplegia, as well. Recal l the mnemon ic for
10 antichol inergic posioning, Hot as a hare, Dry as a bone, Red as a beet, Bl ind as a bat, and Mad as a hatter.
11 Surveying the answers, epinephrine is the sing le best answe r. The mostly like ly mechanism of action of
• 12
epinephrine is decreasing the amount of aqueous humor production v ia stimu lation of or receptors .
• 13 Anticholinergic Mydriasis Epinephrine Glaucoma Intraocular pressure Optic nerve Cycloplegia Sympathomimetic drug Ophthalmoscopy Mnemonic Pathophysiology

• 14 Aqueous humour Atrophy

• 15
• 16
• 17

• 18
• 19
• 20
Miosis Mydriasis
• 21
• 7 7
6
lock
s
Suspend
0
End Block
Item: 11 of 3 8 ~ 1 • M k -<:J 1>- Jil ~· !:';-~
QIO: 1467 ..L ar Prev ious Next Labfli!ll ues Notes Calculat o r

1
&
Surveying the answers, epinephrine is the sing le best answer. The mostly likely m echanism of act ion of &

2 epinephrine is decreasing t h e amount of aqueous humor production via stimu lation of arreceptors.
3 Anticholinergic Mydriasis Epinephrine Glaucoma Intraocular pressure Optic nerve Cycloplegia Sympathomimetic drug Ophthalmoscopy Mnemonic Pathophysiology

4 Aqueous humour Atrophy

5
6
7
8
9
10
Miosis My driasis
11
• 12
• 13 M3 agonist Alpha1-agonist M3 antagonist
Alpha1-antagonist
• 14 -ciliary muscle contracts -no effect on -cilia ry muscle relaxes
-n o effect on
"accommodates" for near ciliary muscle for far vision and cannot
• 15 ciliary muscle
vision and cannot relax = No blu rry vision contract
= No blu rry vision
• 16 = Blurred far vision = Blurred near vision

• 17
• 18
Blurred vision i s the hallmark of
• 19 musc,a rinic drugs.

• 20 Alpha1 = Radial muscle only


(pupillary d~ator muscle}
• 21

a
Lock
s
Suspend
8
End Block
Item: llof38 ~. , . M k <:] t> al ~· ~
QIO: 1467 .l. ar Previous Next lab 'lifllues Notes Calculator

1 •
A is not correct. 6°/o chose this.
2
Acetylchol ine, an agon ist at parasympathetic muscarinic M3 receptors. I t would cause contraction of the cil iary
3
muscle opening the trabecular meshwork increasing aqueous humor outflow. Additionally it would increase
4 pupil lary sphincter contraction (causing miosis) so that the iris wil l not abut and block the canal of Schlem m,
5 also increasing aq ueous humor out flow.
Ciliary muscle Miosis Trabecular meshwork Schlemm' s canal Aqueous humour Parasympathetic nervous system Acetylcholine Muscarinic acetylcholine receptor
6
Iris sphincter muscle Iris (anatomy) Agonist Sphincter Muscle Trabecula
7

8 B is not correct. 27% chose this.


9 Atropine, a muscarinic antagonist, would have the opposite effect of acetylcholine on the parasympathetic
10
nervous system . It causes both mydriasis via inhibition of the pupillary constrictor and cycloplegia (loss of
accom m odation) via inhibition of the cilia ry muscle. Note that the mechanism of myd r iasis with atropine is
11 different from that of dr ugs that have agonist activity at a 1 receptors . As accom m odation was intact in this
• 12 patient, atropine is not the most likely pharmacologic agent .
Cycloplegia Mydriasis Ciliary muscle Atropine Parasympathetic nervous system Muscarinic antagonist Acetylcholine Pharmaceutical drug Receptor antagonist
• 13
• 14 Muscarinic acetylcholine receptor Agonist Pharmacology Nervous system Receptor (biochemistry) Muscle

• 15 Dis not correct. 11% chose this .


• 16 Physostigmine, a reversible anticholinesterase inhibitor, would potentiate the actions of acetylcholine at
• 17 muscarinic receptors because it prevents the breakdown of acetylchol ine. Its effects wou ld therefore be the
same as those of acetylchol ine : miosis and ci liary muscle contraction . It is used in emergencies such as acute
• 18
angle-closure glaucoma .
• 19 Physostigmine Ciliary muscle Miosis Glaucoma Acetylcholinesterase inhibitor Acetylcholine Muscarinic acetylcholine receptor Muscle contraction

• 20 Receptor (biochemistry) Muscle


• 21

6
lock
s
Suspend
0
End Block
Item: llof38 ~. , . M k <:] t> al ~· ~
QIO: 1467 .l. ar Previous Next lab 'lifllues Notes Calculator

2 Dis not correct. 11% chose this.


3 Physostigmine, a reversible anticholinesterase inhibitor, would potentiate the actions of acetylcholine at
muscarinic receptors because it prevents the breakdown of acetylchol ine. Its effects wou ld therefore be the
4
same as those of acetylchol ine : miosis and ci liary muscle contraction . It is used in emergencies such as acute
5 angle-closure glaucoma.
6 Physostigmine Ciliary muscle Miosis Glaucoma Acetylcholinesterase inhibitor Acetylcholine Muscarinic acetylcholine receptor Muscle contraction

7 Receptor (biochemistry) Muscle

8 E is not correct. 22% chose this.


9 Pilocar pine is a cholinomimetic agent with action similar to acetylchol ine. It would therefore lead to miosis and
10 contraction of the ciliary muscle . I t is used to treat both open-angle and closed-angle glaucoma.
Pilocarpine Ciliary muscle Miosis Glaucoma Acetylcholine Parasympathomimetic drug Muscle
11
• 12
• 13 Bottom Line:
• 14 Sympathomimetics act on a1 -adrenerg ic receptors to cause myd riasis by contracting the radial muscle of the
• 15 iris (pupilla ry di lator muscle), whereas cholinomimetics cause miosis by contracting the pupil lary sphincter
• 16 muscle of the ir is. The cil iary muscle, responsible for accommodation, is solely under muscar inic cholinerg ic
control via M3 receptors. Prostaglandins are the first-line therapy for open-angle glaucoma; however,
• 17
epinephrine is also effective at decreasing aqueous humor production . Blurred vision is the hallmark of
• 18 muscarinic drugs, both agon ist and antagonists .
• 19 Mydriasis Ciliary muscle Miosis Glaucoma Epinephrine Iris dilator muscle Aqueous humour Sympathomimetic drug Cholinergic Prostaglandin
Muscarinic acetylcholine receptor Iris (anatomy) Iris sphincter muscle Agonist Parasympathomimetic drug Sphincter Muscle Receptor (biochemistry) Blurred vision
• 20
• 21

6
lock
s
Suspend
0
End Block
Item: llof38 ~. , . M k <:] t> al ~· ~
QIO: 1467 .l. ar Previous Next lab 'lifllues Notes Calculator

1 • •

2 FA17p234.1
3 Sympathomimetics
4 DRUG ACTION APPLICATIONS

5 Direct sympathomimetics
6 Albuterol, salmeterol ~z> ~~ Albuterol for acute asthma or COPO. Salmeterol
7 for long-term asthma or COPO control.
8 Dobutamine ~~>~'a Heart failure (llF) (inotropic > chronotropic),
9 cardiac stress testing.
10 Dopamine 0 1 = 0 2 >~ > a Unstable bradycardia, HF, shock; inotropic and
11
chronotropic effects at lower doses due to ~
effects; vasoconstriction at high doses due to a
• 12
effects.
• 13
Epinephrine ~>a Anaphylaxis, asthma, open-angle glaucoma;
• 14 a effects predominate at high doses.
• 15 Significantly stronger effect at ~-receptor than
• 16 norepinephrine .
• 17 Fenoldopam 01 Postoperative hypertension, hypertensive crisis.
• 18 Vasodilator (coronary, periphera l, renal, and
• 19
splanchnic). Promotes natriuresis. Can cause
hypotension and tachycardia .
• 20
• 21
Isoproterenol ~I=~ Electrophysiologic evaluation of
• t~f'hv::~rrhv l·hm i ~~ r.~n wor~Pn i~f'hPmi~ •

6
lock
s
Suspend
0
End Block
Item: 11 of 3 8 ~ 1 • M k -<:J 1>- Jil ~· !:';-~
QIO: 1467 ..L ar Pre v ious Next Labfli!llues Notes Calcula t o r
& &
1 Isoproterenol Eleclrophysiologic evaluation of
2 lachvarrhvthmias. Can worsen ischemia.
' '
3 Midodrine Autonomic insufficiency and postural
4 hrpotension. May exacerbate supine
5 hypertension.

6 Mirabegron Urinary urge incontinence or O\eractive bladder.


7 Norepinephrine llypotension, septic shock.
8
9 Phenylephrine llypotension (\·asoconstrictor), ocular procedures
(mydriatic), rhinitis (decongestant).
10
Indirect sympathomimetics
11

• 12 Amphetamine Indirect general agonist, reuptake inhibitor, also Narcolepsy, obesity, ADHD .
releases stored catecholamines
• 13
Cocaine Indirect general agonist, reup take inhibitor Causes \'asocons triction and local anesthesia.
• 14
1c,·er give ~-bl ockers if cocaine intoxication is
• 15
• 16
• 17 Ephedrine Indirect general agonist, releases stored
suspected (can lead to unopposed cx1 ac tivation
and extreme hypertension).
asal decongestion (pseudoephedrine), urinary
I
. 18 catecholamines incontinence, hypotension.
• 19
• 20
FA17 p 233.1
• 21 Muscarinic antagonists

a
Lock
s
Suspend
8
End Bl ock
Item: 11 of 3 8 ~ 1 • M k -<:J 1>- Jil ~· !:';-~
QIO: 1467 ..L ar Pre v ious Next Labfli!llues Notes Calcula t o r
& &
1

2
FA17 p233.1
3
Muscarinic antagonists
4 DRUGS ORGAN SYSTEMS APPLICATIONS
5 Atropine, Eye Produce mydriasis and cycloplegia.
6 homatropine,
7 tropicamide
8 Benztropine, Ci\ Parkinson disease {'" park my Ben7").
9
trihexyphenidyl Acute dystonia.
10 Glycopyrrolate C l, respiratory Parenteral: preoperative use to reduce airway
secretions.
11
Oral: drooling, peptic ulcer.
• 12
Hyoscyamine, Cl Antispasmodics for irritable bowel syndrome.
• 13
dicyclomine
• 14
lpratropium, Respiratory COPD, asthma ("' pray I can breathe soon!").
• 15
tiotropium
• 16
Oxybutynin, Genitourinary Reduce bladder spasms and urge urina ry
• 17 solifenacin, incontinence (o\·eractive bladder).
. 18 tolterodine
• 19 Scopolamine CNS lotion sickness.
• 20
• 21
• F A 17 n ?:\.~?

a
Lock Suspend
s 8
End Bl ock
Item: 11 of 3 8 ~ 1 • M k -<:J 1>- Jil ~· !:';-~
QIO: 1467 ..L ar Pre v ious Next Labfli!llues Notes Calcula t o r

1
- . ~ - . . .. . .. .
solifenacin, incontinence (O\'Cractivc bladder).
2 tolterodine
3 Scopolamine c l$ l\ lotion sickness.
4
5
FA17 p 233.2
6
Atropine ~ luscarinic antagonist. Used to treat brad) cardia and for ophthalmic applications.
7
ORGAN SYS!EM ACTION NOTES
8
Eye t pupil dilation, cycloplegia Blocks DU\lBBeLSS in cholinesterase
9
Airway ! secretions inhibitor poisoning. Does not block excitation
10 of skeletal muscle and C 'S (mediated b)
Stomach ! acid secretion
11 nicotinic receptors).
• 12 Gut ! motility
• 13 Bladder ! urgency in cystitis
• 14 ADVERSE EFFECTS t body temperature (due to l sweating); Side efFects:
• 15 rapid pulse; dry mouth; dr), fimhcd ~kin ; llot as a hare
cycloplegia; constipation ; di~ori cnt atio n Or) as a bone
• 16
Can cause acute angle-closure glaucoma in Red as a beet
• 17
elderly {due to mydriasis), urinary retent ion Blind as a bat
• 18 in men with prostatic hyperplasia, and ~lad as a hatter
hyperthermia in infa nts. Jimson weed (Datura) .... gardener's pupil

I
• 19
• 20 (mydriasis due to plant alkaloids)
• 21

a
Lock
s
Suspend
8
End Bl ock
Item: 12 of 38 ~. I • M k <:] t> al ~· ~
QIO: 3472 .l. ar Previous Next lab 'lifllues Notes Calculator

1 •
A 3-year- old boy is brought to the emergency department by his mother. She says that her son has been
2
comp laining that his "head hurts rea lly bad," and has recently been confused and letharg ic. The patient's
3 temperatu re is 39 .5°C (103 . 1°F) . Physica l examination reveals decreased range of motion on flexion and
4 extension of the neck. A sample of cerebrospina l fluid (CSF) shows glucose of 60 mg/d L, protein of 50 mg/dL, and
WBC count of 500/mm 3 (predominantly lymphocytes) . Po lymerase chain reaction of the CSF samp le confirms the
5
presence of a DNA v ir us. CSF cu lture grows no co lonies, and CT of the head shows no enhancing lesions . The child
6 is up to date on all vaccinations and the mother is hea lthy .
7

8 Wh ich of the fo llowing is the most appropriate treatment for this condition?
9
:
10 A. Acyclovir
11
B. I ntravenous fluids
• 12
• 13
C. Ribavirin
• 14 D. Rifampin
• 15 E. Ritonavir
• 16
• 17

• 18
• 19
• 20
• 21

6
lock
s
Suspend
0
End Block
Item: 12 of 38 ~. I • M k <:] t> al ~· ~
QIO: 3472 .l. ar Previous Next lab 'lifllues Notes Calculator

1 •

2
The correct answer is A. 69°/o chose this.
This patient is exhibiting signs and symptoms of meningoencephal itis, including headache, fever, confusion,
3
lethargy, and nuchal r igidity . CSF examination reveals a normal g lucose leve l, a slightly elevated protein leve l,
4 and lymphocytic pleocytosis. These features, especially the presence of lymphocytes, ind icate that the cause is
5 likely to be viral in origin. Presence of a DNA vi r us in the CSF requ ires treatment with acyclovir, as the virus
6
could be a herpesvi rus. Even if the causative agent is varicella-zoster virus or Epstein-Barr virus, acyclovir will
likely be efficacious as a result of the presence of thymidine kinase in all three viruses . After activation by
7
thymidine kinase, acyclovir acts to inhibit vira l DNA po lymerase and thus reduce prol iferation of the virus.
8 DNA virus Epstein-Barr virus Thymidine kinase Aciclovir Pleocytosis DNA polymerase Varicella zoster virus Thymidine Protein Herpesviridae lymphocyte Virus

9 Headache Glucose Shingles DNA Meningoencephalitis Kinase Neck stiffness lethargy Fever Cerebrospinal fluid lymphocytic pleocytosis Meningism

10 Kaposi' s sarcoma-associated herpesvirus

11
B is not correct. 13% chose this.
12
Supportive therapy is suggested for most causes of vira l encephal itis, especia lly if it is caused by RNA viruses.
• 13 However, the presence of herpes simplex DNA viruses as seen by polymerase chain reaction requires treatment
• 14 with acyclovir. Intravenous fluids are essential in the management of sepsis in which hypotension is observed .
Aciclovir Polymerase chain reaction Sepsis Hypotension Encephalitis Herpes simplex Virus Intravenous therapy RNA virus DNA RNA
• 15
• 16 C is not correct. 8°/o chose this .
• 17 Ribavirin is an antivira l med ication used to t reat respiratory syncytial virus and hepatit is C infection. Ribavirin
• 18
inhibits the initiation and elongation of RNA fragments .
Ribavirin Hepatitis C Human respiratory syncytial virus Antiviral drug Hepatitis Syncytium RNA Virus Pharmaceutical drug
• 19
• 20 D is not correct. 5°/o chose this .
• 21
Rifampin is an antimicrobial agent used for prophylaxis in close contacts of patients with Neisseria meningitidis
• t'H"' 1-/::u::unl'"'lnl-·tihte inf!J,.Qn7=:to I"'Y"'Qnint"'1 it i e =an""'I"\MI"'I 1'\thcr"" • •ece Th i e n:J~ti c.nt l i i/Q I\1 h=ae =a H i r:~~ l 1""\1'" =aeQnti r I"'Y"'Qn i nl"''iti e =ae

6
lock
s
Suspend
0
End Block
Item: 12 of 38 ~. I • M k <:] t> al ~· ~
QIO: 3472 .l. ar Previous Next lab 'lifllues Notes Calculator

1
D is not correct. 5°/o chose this.
2
Rifampin is an antimicrobial agent used for prophylaxis in close contacts of patients with Neisseria meningitidis
3 or Haemophilus influenzae men ing itis, among other uses. This patient likely has a vi ral or aseptic mening it is as
4 indicated by his CS F profi le : the presence of lymphocytes rather than neutrophi ls ind icates that the cause of the
5 symptoms is viral rather than bact erial. Rifampin wou ld not be a good choice for th is patient as he has not had
prolonged exposu re to a person with Neisseria meningitidis or Haemophilus influenzae mening it is. Rifampin act s
6
by inhibiting bacteria l DNA-dependent RNA polymerase, thus inh ibit ing bacterial RNA transcription .
7 Neisseria meningitidis Haemophilus influenzae Rifampicin Aseptic meningitis Meningitis Neutrophil Antimicrobial lymphocyte Transcription (genetics) Virus

8 Preventive healthcare RNA polymerase RNA Neisseria Haemophilus

9
E is not correct. 5°/o chose this.
10
Ritonavir is an antiretrovira l protease inh ibitor used to t reat HIV/AIDS. It is often prescribed as part of a high ly
11 active antiretroviral therapy (HAART) reg imen . Although HIV infection can cause vi ral meningitis, we have no
12 other reason to bel ieve that the chi ld is HIV-posit ive, as the mother is healthy.
Ritonavir Protease inhibitor (pharmacology) Management of HIV/AIDS Meningitis HIV Protease inhibitor (biology) Protease Virus Enzyme inhibitor
• 13
• 14
• 15 Bottom Line:
• 16 Aseptic or v iral mening it is is usual ly a self -limited disease that may manifest with headache or fever. Look for
• 17 an elevated opening pressure on lumbar punct ure, and a CS F with norma l protein and glucose levels but
• 18
increased cells, predominantly lymphocytes . The majority of cases are caused by enteroviruses, herpesviruses,
and HIV. Acyclovir is the drug of choice in cases when her pes simplex virus is the source .
• 19 lumbar puncture Aciclovir Meningitis Viral meningitis Herpes simplex virus Herpes simplex Headache Protein Herpesviridae Enterovirus lymphocyte Glucose Fever
• 20 Virus

• 21

6
lock
s
Suspend
0
End Block
Item: 12 of 38 ~. I • M k <:] t> al ~· ~
QIO: 3472 .l. ar Previous Next lab 'lifllues Notes Calculator

1 •

2 FA17 p 197.3
3 Acyclovir, famciclovir, valacyclovir
4 MECHANISM Guanosine analogs. Monophosphorylated by llSVIVZV thymidine kinase and not phosphorylated
5 in uninfected cells - few adverse effects. Triphosphate formed by cellular enzymes. Preferentially
inhibit viral D A polymerase by chain termination.
6
7 CliNICALUSE HSV and VZV Weak activity against EBV o activity against CMV Used for HSV-
induced mucocutaneous and genital lesions as well as for encephalitis. Prophylaxis in
8
immunocompromised patients. l o effect on latent forms ofH SV and VZV. alaC)'dovir, a
9 prodrug of acyclovir, has better oral bioava ilability.
10 For herpes zoster, use famciclov ir.
11 ADVERSE EFFECTS Obstructive crystalline nephropathy and acute renal failure if not adequately hydrated.
12 MECHANISM OF RESISTANCE Mutated viral thymidine kinase.
• 13
• 14
FA11 p 176.2
• 15 CSF findings in meningitis
• 16 OPENING PRESSURE CEll TYPE PROTEitl GlUCOSE
• 17 Bacterial t t PMNs t
• 18 Fu ngai/TB t t lymphocytes t
• 19 Viral ormal/t t lymphocytes ormal/t Normal
• 20
• 21
• FA17 p 176.1

6
lock
s
Suspend
0
End Block
Item: 12 of 38 ~. I • M k <:] t> al ~· ~
QIO: 3472 .l. ar Previous Next lab 'lifllues Notes Calculator

1 •

2 FA17 p 176.2
3 CSF findings in meningitis
4 OPENING PRESSURE CEll TYPE PROTEIN GlUCOSE

5 Bacterial t t PMNs t
6 Fungai/TB t t lymphocytes t
7 Viral ormal/t t lymphocytes ormal/t Normal
8
9
FA17 p 176.1
10 Common causes of meningitis
11 NEWBORN (0- 6 MO) CHILDREN (6 M0- 6 YR) 6- 60YR 60YR+

12 Croup B streptococci S pneumoniae S pneumoniae S pneumoniae


• 13 E coli l meningitidis N meningitidis (#l in teens) Cram 8 rods
Listeria H influenzae type B Enterovi ruses Listeria
• 14
Enteroviruses HSV
• 15
Give ceftriaxone and vancomycin empirically (add ampicillin if Listeria is suspected).
• 16
Viral causes of meningitis: enteroviruses (especially coxsackievirus), HSV-2 (llSV-1 =encephalitis), lllV, West ile virus (also
• 17 causes encephalitis), VZV.
• 18 In HIV: Cryptococcus spp.
• 19 Note: Incidence of H influenzae meningitis has ! greatly due to conjugate H influenzae vaccinations. Today, cases are usually
• 20
seen in unimmunized children .
• 21

6
lock
s
Suspend
0
End Block
Item: 13 of 3 8 ~ 1 • M k -<:J 1>- Jil ~· !:';-~
QIO: 1079 ..L ar Pre v ious Next Lab fli!ltues Notes Calcula t o r

IAA]
& &
1
A 41-year-old man is admitted to the hospital for progressive obtundation. On admission the patient's serum
2
sod ium level is 114 mEq/L. Treatment is initiated, and 7 hours later the patient's serum sodium level is 134
3 mEq/L. Over the next 4 days the patient's condition worsens with the development of dysarthria, dysphagia,
4 and paraparesis.
5
6 What pathologic process is most likely responsible for this patient's new symptoms?
7 :

8 A. Cerebral edema
9 B. Diffuse axonal injury
10
C. Intracerebral hemorrhage
11
D. Osmotic demyelination
12
• 13 E. Uncal herniation
• 14
• 15
• 16
• 17
• 18
• 19
• 20
• 21

a
Lock
s
Suspend
8
End Bl ock
Item: 13 of 38 ~. I • M k <:] t> al ~· ~
QIO: 1079 .l. ar Previous Next lab 'lifllues Notes Calculator

1 •
The correct answer is D. 73°/o chose this.
2
Osmotic demyel ination, also known as central pontine myelino lysis, can result from overagg ressive treatment of
3
hyponatremia. As hyponatremia develops, the bra in prevents cerebral edema by gradual ly reducing its own
4 osmola r ity, thus reducing the osmotic gradient that would otherwise force water intracel lularly . The brain can
5 gradually replace these lost osmo les as the serum osmolarity is corrected, but correction of the serum sodium
6
level at a rate usual ly no faster than 0.5 mEq/L/h (can go as fast as 1.0 mEq/L/h if very severe, but th is is
rare•) outpaces the brain's abi lity to compensate, resulting in neu rona l sh r inkage and death. The clinica l
7 manifestations occu r several days later and include dysarthria, dysphag ia, and flaccid quad ripa resis that can
8 become spastic and may progress to a "locked-in" syndrome, in wh ich the patient retains ful l awareness but can
9 move on ly the extraocu lar muscles.
Central pontine myelinolysis Hyponatremia Dysarthria Dysphagia Extraocular muscles Demyelinating disease Cerebral edema Flaccid paralysis
10
Osmotic concentration locked-in syndrome Tetraplegia Sodium Edema Blood plasma Osmosis Brain Human brain Serum (blood) Pons
11

12 A is not correct. 12% chose this.


13 This patient presents with centra l pontine myel inosis after rapid correction of hyponatremia over 7 hours. In the
hyponatremic, hypo-osmolar state, cerebra l edema occurs. If cor rection occu rs too quickly (greater than 0.5
• 14
mEq/L/h), for examp le with aggressive use of hypertonic sa line solution, fluid leaves the intracellular space and
• 15 leads to death of cells including oligodend rocytes, lead ing to demyelination. The patient in the vignette has
• 16 signs of locked-in syndrome, such as pa raparesis and cranial nerve involvement. Recal l that hyponatremia
• 17 greater than 12 mEq/24 h should never be corrected. Converse ly, rapid correction of HYPERnatremia can resu lt
in cerebral edema: rapidly lowering the sodium concentration can resu lt in an influx of water into neu rons,
• 18
which causes them to swell. Cerebral edema may result in seizu res, brain damage, or death .
• 19 Central pontine myelinolysis locked-in syndrome Hyponatremia Cerebral edema Demyelinating disease Edema Cranial nerves Tonicity Sodium Brain damage

• 20 Intracellular Neuron Epileptic seizure Oligodendrocyte Paraplegia Saline (medicine) Brain Pons

• 21
• B is not correct. 60/o chose this .
6
lock
s
Suspend
0
End Block
Item: 13 of 38 ~. I • M k <:] t> al ~· ~
QIO: 1079 .l. ar Previous Next lab 'lifllues Notes Calculator

1 •
B is not correct. 6°/o chose this.
2
Diffuse axonal inju ry occurs in the setting of central nervous system trauma or angu lar acceleration or both and
3
resu lts in disruption of the axon at the nodes of Ranvier. Diffuse axonal injury does not occur with electrolyte
4 abnormalities.
5 Diffuse axonal injury Node of Ranvier Axon Central nervous system Electrolyte Angular acceleration Nervous system Major trauma

6 C is not correct. 5°/o chose this.


7 I ntracerebral hemor rhage can occur as a result of hypertension, arteriovenous malformations, anticoagulation,
8 thrombolysis, or amyloid ang iopathy; however, it does not occur as a resu lt of hyponatremia or the associated
treatment.
9
Hyponatremia Thrombolysis Cerebral hemorrhage Anticoagulant Hypertension Bleeding Arteriovenous malformation Amyloid Cerebrum Angiopathy
10
Cerebral arteriovenous malformation Brain Cerebral cortex
11

12
E is not correct. 4°/o chose this.
Uncal hern iation can result only from focal processes within the crania l vault, such as intracran ial hemorrhage,
13
but does not occur with diffuse processes associated with electrolyte abnor mal ities .
• 14 Intracranial hemorrhage Electrolyte Cranial vault Skull Uncal herniation Bleeding Water-electrolyte imbalance Hernia Temporal lobe
• 15
• 16
• 17
Bottom Line:

• 18 Osmotic demyel ination syndrome, originally termed central pontine myel inolysis, man ifests fol lowing rapid
correction of significant hyponatremia (ie, > 1 mEq/L/h). Cerebral edema, in contrast, can occur as a result of
• 19
overly aggressive correction of hypernatremia .
• 20 Central pontine myelinolysis Hyponatremia Hypernatremia Demyelinating disease Cerebral edema Edema Pons Osmosis

• 21

6
lock
s
Suspend
0
End Block
Item: 13 of 3 8 ~ 1 • M k -<:J 1>- Jil ~· !:';-~
QIO: 1079 ..L ar Pre v ious Next Lab fli!ltues Notes Calcula t o r
& &
1
2 FA17 p 560.2
3 Electrolyte disturbances
4 ELECTROLYTE LOWSERUMCONCENTRATION HIGHSERUMCONCENTRATION
~-----

5 Na+ 'ausea and malaise, stupor, coma, seizures Irritability, stupor, coma
6 K U wa,·cs and Aatlened T waves on £CC, Wide QRS and peaked T waves on ECC,
7 arrh}thmias. muscle cramps, spasm, weakness arrh,thmias,

muscle weakness
8 Tetany, seizures, QT prolongation, h\ itching Stone~ (renal), bones (pain), groam (abdominal
9 (Ch,·ostek sign), spasm (Trousseau sign) pain), thrones (f urinary frequency), ps} chiatrie
O\ertones (anxiety, altered mental status)
10
Mg2+ Tetan}, torsades de pointes, h) pokalemia, l DTRs, lethargy, bradycardia, hypotension,
11
hypocalcemia (when [vlg2+] < 1.2 mg/dL) cardiac arrest, hypocalcemia
12
PO 4 3 Bone loss, osteomalacia (adults), rickets Renal stones, metastatic ca lcifications,
13
(children) hypocalcemia
• 14
• 15
FA17 p 492.2
• 16
• 17 Osmotic demyelination Acute paralysis, dysar th ria, dysphagia, diplopia, Correcting serum la+ too fast:
syndrome (central loss of consciousness. Can cause "locked-i n "From low to high, your pons will die"
• 18
pontine myelinolysis) syndrome." Massive axonal demyelination in (osmotic demyelination syndrome)
• 19
zo
pontine white matter fJ to osmotic changes. "From high to low, your brain will blow"
• 20 /
... '
~ . ,.I ~

Commonly iatrogenic, caused by O\ erly rapid (cerebral edema/herniation)


• 21 correction of hyponatremia. In contrast,
• -. f-

a
Lock
s
Suspend
8
End Bl ock
Item: 13 of 3 8 ~ 1 • M k -<:J 1>- Jil ~· !:';-~
QIO: 1079 ..L ar Pre v ious Next Lab fli!ltues Notes Calcula t o r

1
&
U waves and Aaliened T wa,•cs on ECC, Wide QRS and peaked T waves on ECC, &

2 arrhythmias, muscle cramps, spasm, weakness arrlwthmias, muscle weakness


'
3 Tetany, seizures, QT prolongation,'" itching Stoncl> (renal), bones (pain), groam (abdominal
4 (Chvostek sign), spasm (Trousseau sign) pain), thrones (t urinary frequency), psychiatric
mertone~ (anxiety, altered mental status)
5
Mg2+ 'lelan), lorsades de poinles, h) pokalcmia, l DTRs, lethargy, bradycardia, hypotension,
6
hn>ocalcemia (when [\ lg2+] < 1.2 mg/dL) cardiac arrest, hypocalcemia
7
PO4 3 Bone loss, osteomalacia (adults), rickets Renal stones, metastatic calcifications,
8
(children) h) pocalcemia
9
10
FA17 p 492.2
11
Osmotic demyelination Acute paralysis, dysarthria, dysphagia, diplopia, Correcting serum Na+ too fast :
12
syndrome (central loss of consciousness. Can cause "locked-i n "From low to high, your pons will die"
13
pontine myelinolysis) syndrome." Massive axonal demyelination in (osmotic demyelination syndrome)
• 14 zo
pontine " ·hitc matter fJ to osmotic changes. • "From high to low, your brain will blow"
• 15 Commonly iatrogenic, c;~used by overly rapid (cerebral edema/herniation)
• 16 correction of hyponatremia. In contrast,
• 17
correcting hypernatremia too quickly results in
cerebral edema/hcrn iation .
. 18
• 19
• 20
• 21

a
Lock
s
Suspend
8
End Bl ock
Item: 14 of 3 8 ~ 1 • M k -<:J 1>- Jil ~· !:';-~
QIO: 1478 ..L ar Pre v ious Next Lab fli!ltues Not es Calcula t o r

IAA]
& &
1
A 25-year-old man undergoes arthroscopic knee surgery to repair a torn lateral meniscus in his right knee.
2
The patient consented to general anesthesia for the operation. During the surgery the patient suddenly
3 develops jerking of the neck and extremities. There is no associated rig idity or fever.
4
5 Which of the following inhaled anesthetics was most likely used?
6 :
7 A. Enflurane
8
B. Halothane
9
C. Isoflurane
10
11 D. Nitrous oxide
12 E. Thiopental
13
• 14
• 15
• 16
• 17
• 18
• 19
• 20
• 21

a
Lock
s
Suspend
8
End Bl ock
Item: 14 of 38 ~. I • M k <:] t> al ~· ~
QIO: 1478 .l. ar Previous Next lab 'lifllues Notes Calculator

1 • The correct answer is A. 30°/o chose this .


2 Enflurane is more like ly to produce central nervous system excitation and seizure activity compared to other
3
inhaled anesthetic agents. Although it is a less potent anesthetic than halothane, it produces a more rapid
induction and a quicker recovery than ha lothane. Seizures induced by enflurane are usua lly self- limited and are
4
not believed to have any permanent neu rolog ic effects.
5 Enflurane Halothane Central nervous system Inhalational anaesthetic Anesthesia Anesthetic Epileptic seizure

6
B is not correct. 27% chose this.
7
Halothane is a very potent inhaled anesthetic that is considered the prototype of its class. A common ly
8 associated side effect with halothane is hepatotoxicity, not seizu res.
Halothane Hepatotoxicity Inhalational anaesthetic Adverse effect Anesthetic Anesthesia Epileptic seizure
9
10 C is not correct. 15% chose this.
11 I soflurane is an inhaled anesthetic that causes very little organ toxicity. I t does not precipitate seizu res.
Isoflurane Inhalational anaesthetic Anesthetic Anesthesia Epileptic seizure Toxicity
12
13 D is not correct. 12% chose this.
14 Nitrous oxide is a very weak inhaled anesthetic and is thus often used in conjunction with other anesthetics. It
• 15 does not cause seizu res .
Nitrous oxide Inhalational anaesthetic Anesthesia Anesthetic Epileptic seizure Oxide
• 16
• 17 E is not correct. 16% chose this .
• 18 Thiopental is an intravenous anesthetic that is not inhaled. Furthermore, it does not produce seizu res as a side
effect. Seizures with thiopental are very rare, and the dr ug can be used to treat convulsive states fo llowing
• 19
anesthesia .
• 20 Anesthesia Sodium thiopental Intravenous therapy Adverse effect Anesthetic Side effect Epileptic seizure

• 21

6
lock
s
Suspend
0
End Block
Item: 14 of 38 ~. I • M k <:] t> al ~· ~
QIO: 1478 .l. ar Previous Next lab 'lifllues Notes Calculator

1 • Halothane is a very potent inhaled anesthetic that is considered the prototype of its class. A common ly
2
associated side effect with halothane is hepatotoxicity, not seizu res.
Halothane Hepatotoxicity Inhalational anaesthetic Adverse effect Anesthetic Anesthesia Epileptic seizure
3
4 C is not correct. 15% chose this.
5
I soflurane is an inhaled anesthetic that causes very little organ toxicity. I t does not precipitate seizu res.
Isoflurane Inhalational anaesthetic Anesthetic Anesthesia Epileptic seizure Toxicity
6
7 D is not correct. 12% chose this.
8
Nitrous oxide is a very weak inhaled anesthetic and is thus often used in conjunction with other anesthetics. It
does not cause seizu res.
9 Nitrous oxide Inhalational anaesthetic Anesthesia Anesthetic Epileptic seizure Oxide
10
E is not correct. 16% chose this.
11
Thiopental is an intravenous anesthetic that is not inhaled . Furthermore, it does not produce seizu res as a side
12 effect. Seizures with thiopental are very rare, and the dr ug can be used to treat convulsive states fo llowing
13 anesthesia.
Anesthesia Sodium thiopental Intravenous therapy Adverse effect Anesthetic Side effect Epileptic seizure
14
• 15
• 16 Bottom Line:
• 17
Know the unique toxicities of inha led anesthetics . Enflurane is known to be a pro-convu lsant. Ha lothane can
• 18 cause hepatotoxicity and has the biggest propensity for causing mal ignant hyperthermia. Desflurane can cause
• 19 airway ir ritability. Methoxyflurane is associated with neph rotoxicity .
Desflurane Enflurane Halothane Malignant hyperthermia Methoxyflurane Hepatotoxicity Nephrotoxicity Anesthetic Hyperthermia Convulsant Inhalational anaesthetic
• 20
• 21

6
lock
s
Suspend
0
End Block
Item: 14 of 3 8 ~ 1 • M k -<:J 1>- Jil ~· !:';-~
QIO: 1478 ..L ar Pre v ious Next Lab fli!ltues Not es Calcula t o r
& fi R ST AID fAC T S &
1

2
FA17p519.1
3
Inhaled anesthetics DesAurane, halothane, enAurane, isoAurane, sevoAurane, methoxyflurane, 1 20.
4
MECHANISM t\ lechanism unkno" n.
5
EFFECIS lyocardial depression, respiratory depression, nausea/emesis, t cerebral blood Aow (l cerebral
6
metabolic demand).
7
ADVERSE EFFECIS Hepatotoxicity (halothane), ncphroto\icit) (methoxyflurane), proconmlsant (enAurane,
8 epileptogenic), expansion of trapped gas in a body ca\'ity ( ~20).
9 Malignant hyperthe rmia- rare, life-threatening condition in which inhaled anesthetics or
10 succinylcholine induce fe\'er and se,ere muscle contractions. Susceptibilitr is often inherited as
11
autosomal dominant" ith ,·ariable penetranee. f\ lutations in voltage-sensitive ryanodine receptor
cause t Ca2+ release from sarcoplasmic reticulum . Treatment: dantrolene (a ryanodine receptor
12
antagonist).
13
14 FA17 p 519.2
• 15
Intravenous T he Mighty King Proposes to Oprah.
• 16 anesthetics
• 17
Barbiturates l ligh potency, high lipid solubility, rapid entry into brain. Used for induction of anesthesia and
. 18 (Thiopental) short surgical procedures. Effect terminated by rapid redistribution into tissue and fat. l cerebral
• 19 blood flow.
• 20 Benzodiazepines Used for endoscopy; used adjuncti,·cly "it h gaseous anesthetics and narcotics. .\ lay cause severe
• 21 (Midazolam) postoperati,·e respiratory depression, l BP (treat overdose" ith Aumazenil), anterograde amnesia.

a
Lock
s
Suspend
8
End Bl ock
Item: 14 of 3 8 ~ 1 • M k -<:J 1>- Jil ~· !:';-~
QIO: 1478 ..L ar Pre v ious Next Lab fli!ltues Not es Calcula t o r
& &
1
2 FA17 p 519.2
3 Intravenous The ~ Ii ghty King Proposes to Oprah.
4 anesthetics
5 Barbiturates High potencr. high lipid solubility, rapid entry into brain. Used for induction of anesthesia and
6 (Thiopental) short surgical procedures. Effect terminated b) rapid redistribution into tissue and fat. ! cerebral
7 blood flo\\'.
8 Benzodiazepines Used for endoscopy; used adjunctively ''it h gaseous anesthetics and narcotics. ~ lay cause severe
9
(Midazolam) postoperative respirator) depression, ! BP (treat overdose "ith Aumazenil), anterograde amnesia.
10
Arylcyclohexylamines PCP analogs that act as dissociative anesthetics. Block M DA receptors. Cardiovascular
(Ketamine) stimulants. Cause disorientation, hallucination, unpleasant dreams. t cerebral blood Aow.
11
Propofol Used for sedation in ICU, rapid ane thesia induction, short procedures. Less postoperative nausea
12
than thiopental. Potentiates CABAA'
13
Opioids Morph ine, fentanyl used with other C S depressants during general anesthesia.
14
• 15
• 16 FA17 p 518.6

• 17 Anesthetics- general C S drugs must be lipid soluble (cross the blood-brain barrier} or be actively transported.
• 18
principles Drugs with ~ solubi l ity in blood = rapid induction and recovery times.
• 19 Drugs with t solubility in lipids = t potency = I
M C
• 20
• 21
lAC = .\1inimal Ah·eolar Concentration (of inhaled anesthetic) required to pre\·ent 50% of

a
Lock
s
Suspend
8
End Bl ock
Item: 14 of 3 8 ~ 1 • M k -<:J 1>- Jil ~· !:';-~
QIO: 1478 ..L ar Pre v ious Next Lab fli!ltues Not es Calcula t o r

1 : • g P y, g I y, I Y I

2
(Thiopental) short surgical procedures. Effect terminated by rapid redistribution into tissue and fat. ! cerebral
blood flow.
3
Benzodiazepines Used for endoscopy; used adjuncti,·cly with gaseous anesthetics and narcotics. \ lay cause severe
4
(Midazolam) postoperati\'e respiratory depression, ! BP (treat overdose " ith Aumazenil), anterograde amnesia.
5
Arylcyclohexylamines PC P analogs that act as dissociative a nesthetics. Block ~IDA receptors. Cardio,·ascular
6
(Ketamine) stimulants. Cause disorientation, hallucinat ion, unpleasant dreams. t cerebral blood Aow.
7
Propofol Used for sedation in ICU, rapid anesthesia induction. short procedures. Less postoperati,·e nausea
8 than thiopental. Potentiates CABAA'
9
Opioids \ lorphine, fentanyl used with other CNS depressants during general anesthesia.
10
11
FA17 p 518.6
12
Anesthet ics- general C S drugs must be lipid soluble (cross the blood-brain barrier) or be actively transported.
13
principles Drugs with ! solubi lity in blood = rapid induction and recovery times.
14
• 15 Drugs with t solubi lity in lipids = t poteucy = 1
MAC
• 16
l\IAC = Ylinimal Alveolar Concentration (of inhaled anesthetic) required to pre\'ent 50% of
• 17
subjects from moving in response to noxious stimulus (eg, skin incision).
• 18 Examples: nitrous oxide ( 20 ) has ! blood and lipid solubility, and thus fast induction and low
• 19 potency. Halothane. in contrast, has t lipid and blood solubility, and thus high potency and slow
• 20 induction.
• 21

a
Lock
s
Suspend
8
End Bl ock
Item: 15 of 3 8 ~ 1 • M k -<:J 1>- Jil ~· !:';-~
QIO: 5262 ..L ar Pre v ious Next Lab fli!ltues Notes Calcula t o r

IAA]
& &
1
A 52-year-old man comes t o his physician with complaints of muscle weakness, especia lly near the end of
2
the day. His cranial nerve examinat ion is shown in the video clip.
3
OPEN MEDIA
4
5
6 Which of the following drugs can be used to confirm the diagnosis?
7 :
8 A. Atropine
9 B. Donepezil
10
C. Edrophonium
11
12 D. Pancuronium
13 E. Rivastigmine
14
0 15
0
16
0
17
0
18
• 19
0 20
• 21

a
Lock
s
Suspend
8
End Bl ock
Item: 15 of 38 ~. I • M k <:] t> al ~· ~
QIO: 5262 .l. ar Previous Next lab 'lifllues Notes Calculator

1 •

2 The correct answer is C. 73°/o chose this.


3
This patient is suffering from myasthenia gravis, an autoimmune neuromuscular junction disorder caused by
antibodies that block the acetylchol ine receptors on the postsynaptic membrane. This patient's physica l exam
4
reveals one of the most common clin ical symptoms of myasthen ia gravis, muscle fatigue that prog ressively
5 worsens with muscle use. The diagnosis of myasthenia gravis can be confirmed using the edrophonium test.
6 Edrophonium, an acetylcho linesterase inhibitor that works primarily in the neuromuscu lar junction, will briefly
relieve muscle weakness.
7
Acetylcholinesterase inhibitor Myasthenia gravis Neuromuscular junction Acetylcholinesterase Edrophonium Acetylcholine Autoimmune disease Chemical synapse
8
Muscle weakness Autoimmunity Antibody Fatigue (medical) Muscle Receptor (biochemistry) Biological membrane Acetylcholine receptor
9
10
A is not correct. 7°/o chose this.
Atropine is an antagon ist of muscarinic acetylchol ine receptors . It has no effect on the nicotinic acetylcho line
11
receptors of the neu romuscular junction.
12 Atropine Neuromuscular junction Acetylcholine Antagonist Receptor antagonist Muscarinic acetylcholine receptor Nicotinic acetylcholine receptor
13 Receptor (biochemistry) Acetylcholine receptor
14
B is not correct. 6°/o chose this.
15
Donepezil, another centra lly acting acetylchol inesterase inhibitor, is used to treat dementia associated with
• 16 Alzheimer disease. Donepezi l has a long half- life and acts primarily within the centra l nervous system, making it
• 17 unsu itable for testing for myasthen ia gravis .
Acetylcholinesterase inhibitor Donepezil Myasthenia gravis Acetylcholinesterase Central nervous system Half-life Dementia Alzheimer' s disease Nervous system
• 18
Enzyme inhibitor
• 19
• 20 D is not correct. 4°/o chose this .
• 21 Pancuronium is a competitive acetylcholine antagonist that causes nondepolar izing para lysis at the

6
lock
s
Suspend
0
End Block
Item: 15 of 3 8 ~ 1 • M k -<:J 1>- Jil ~· !:';-~
QIO: 5262

1
&
..L
. . . ar Pre v ious Next Lab fli!ltues Notes Calcula t o r
&

2
FA17 p232.1
3 Cholinomimetic agents
4 DRUG ACTION APPLICATIONS
5 Direct agonists
6 Bethanechol Activates bowel and bladder smooth muscle; Postoperative ileus, neurogenic ileus, urinar}
7 resistant to AChE, no n icotinic acti,·ity. retention
8 "Bethany, call (bethanechol) me to activate
rour bowels and bladder."

9
10
Carbachol C arbon copy of acetr lcholine (but resistant to Constricts pupil and relieves intraocular
AChE). pressure in open-angle glaucoma
11
Methacholine Stimulates m uscarinic receptors in airway when C hallenge test for diagnosis of asthma
12
inhaled.
13
Pilocarpine Contracts ciliary m uscle of eye (open-:mglc Potent stimulator of sweat, tears, and saliva
14
glaucoma), pupillary sphincter (closed-angle O pen-angle and closed-angle glaucoma,
15 glaucoma); resistant to AC h E, can cross blood- xerostomia (Sjogren syndrome)
• 16 brain barrier (tertiary amine). "You cry, drool,
• 17 and swea t on your ' p•.Iow.'"
. 18 Indirect agonists (anticholinesterases)
• 19 Galantamine, t ACh. Al:rheimer disease (AI7heimer patients gallantly
• 20 donepezil, swim down the ri' er)
rivastigmine
• 21

a
Lock
s
Suspend
8
End Bl ock
Item: 15 of 3 8 ~ 1 • M k -<:J 1>- Jil ~· !:';-~
QIO: 5262 ..L ar Pre v ious Next Lab fli!ltues Notes Calcula t o r
& &
1 Indirect agonists (anticholinesterases)
2 Galantamine, t ACh. Al:rhei mer disease (AI7heimer patients gallantly
3 donepezil, swim down the ri' er)
4 rivastigmine
5 Edrophonium t Ch. Historicalh- used to diagnose myasthenia gra,·is;
6 replaced by anti-.\ ChR Ab (anti-acetylcholine
receptor antibody) test.
7
Neostigmine t ACh. Postoperative and neurogenic ileus and
8
r\eo CKS = :\o CN'S penetration (quaternary urinary retention, myasthenia gravis,
9
amine). reversal of neuromuscular junction blockade
10 (postoperative).
11 Physostigmine t ACh. Ph~ sostigmine "ph) \Cs" atropine Antidote for anticholinergic toxicity; phrcely
12 overdose. (freely) crosses blood-brain barrier - Cl S
13 (tertiary ami ne).
14 Pyridostigmine t ACh; t muscle strength. Pyridostigm inc gets Myasthenia gravis (long acting); does not
15 rid of myasthenia gravis. penetrate CNS (quaternary ami ne).
• 16 Note: With all cholinom imetic agents, watch for exacerbat ion of COPD, asthma, and peptic ulcers when giving to susceptible
• 17 patients.
. 18
• 19 FA17 p445.2
Neuromuscular junction diseases
• 20
• 21
Myasthenia gravis lambert-Eaton myasthenic syndrome

a
Lock
s
Suspend
8
End Bl ock
Item: 15 of 3 8 ~ 1 • M k -<:J 1>- Jil ~· !:';-~
QIO: 5262 ..L ar Pre v ious Next Lab fli!ltues Notes Calcula t o r
•• • •
1
Physostigmine t ACh. Ph~ sostigm i ne "ph) -:es" atropine Antidote for anticholinergic toxicity; phrccly
2
o,·erdose. (freely) crosses blood-brain barrier - C l$
3 (tertiary amine).
4
Pyridostigmine t ACh; t muscle strength. Pyridostigmine gets lrasthenia gravis (long acting); does not
5 rid of mrasthenia gra,·is. penetrate C1 S (quaternary amine).
6 1 ote: With all cholinomimetic agents, watch for exacerbation ofCOPD, asthma, and peptic ulcers when giving to susceptible
7 patients.
8
9 FA17 p 445.2
10 Neuromuscular junction diseases
11 Myasthe nia gravis lambert-Eaton myast henic syndrome
12 FREQUENCY Most common 1 MJ disorder Uncommon
13 PATHOPHYSIOLOGY Autoantibodies to postsynaptic ACh receptor Autoantibodies to presr naptic Ca2+ channel
14 - ! Ch release
15 CLINICAL Ptosis, diplopia, weakness Proximal muscle weakness, autonomic
• 16 Worsens with muscle use symptoms (dry mouth, impotence)
• 17 Improvement after cdrophon ium (tensilon) test Improves with muscle use
• 18 ASSOCIATED WITH Thymoma, thymic hyperplasia Small cell lung cancer
• 19 AChE INHIBITOR ADMINISTRATION Re,·erses symptoms (edrophonium to diagnose, linimal effect
• 20 p)Tidostigmine to treat)
• 21

a
Lock
s
Suspend
8
End Bl ock
Item: 16 of 3 8 ~ 1 • M k -<:J 1>- Jil ~· !:';-~
QIO: 1446 ..L ar Pre v ious Next Lab fli!ltues Not es Calcula t o r
& &
1
A researcher is examining the different properties of anesthetic gases in a series of experiments. Each gas is
2
determined to have a specific minimum alveolar concentration (MAC) and blood solubility. I n t he
3 experiments, the MAC of a gas is defined as t he end-expiratory concentration at which half of patients wi ll
4 not elicit a motor response to a standardized surgical incision. The inhaled anesthetic drugs A, B, C, and D have
the properties indicated in the chart .
5
Minimum
6 alveolar Blood
concentration solubility
7
8 Drug A 7.3 0.5

9
Drug B 3.1 1.0
10
Drug C 2 .0 0.1
11
12 Drug D 4.4 0.15
13
14
Which of the following statements best describes the properties of these drugs?
15
:
• 16
A . Drug A is more potent than drug D
• 17
• 18
B. Drug B is less soluble in blood than drug C
• 19 C. Drug C wi ll have both the highest potency and t he most rapid induction
• 20 D . Drug D w ill induce anesthesia more rapidly than drug C
• 21

a
Lock
s
Suspend
8
End Bl ock
Item: 16 of 3 8 ~ 1 • M k -<:J 1>- Jil ~· !:';-~
QIO: 1446 ..L ar Pre v ious Next Lab fli!ltues Not es Calcula t o r

experiments, the MAC of a gas is defined as the end-expiratory concentration at which ha lf of patients wi ll
&
1 •
2 not elicit a motor response to a standardized surgical incision. The inhaled anesthetic drugs A, B, C, and D have
the properties indicated in the chart .
3
Minimum
4 Blood
alveolar
concentration solubility
5
6 Drug A 7.3 0.5

7
OrugB 3.1 1.0
8
Drug C 2.0 0.1
9
10 Drug D 4.4 0.15
11

12
Which of the following statements best describes the properties of these drugs?
13
:
14
A. Drug A is more potent than drug D
15
• 16
B. Drug B is less soluble in blood than drug C

• 17 C. Dr ug C w ill have both the highest potency and the most rap id induction
. 18 D. Drug D w ill induce anesthesia more rapidly than drug C
• 19
E. Patients t reated with drug B will recover more quickly f rom anesthesia t han drug A
• 20
• 21

a
Lock
s
Suspend
8
End Bl ock
Item: 16 of 38 ~. I • M k <:] t> al ~· ~
QIO: 1446 .l. ar Previous Next lab 'lifllues Notes Calculator

1 •

2 The correct answer is C. 68°/o chose this.


3 The potency of inha led anesthetics is quantified as the minimum alveolar concentration (MAC) . This is the
4 concentration of inhaled gas that is needed to el iminate movement in 50% of patients who are cha llenged by
surgica l incision. For potent anesthetics, the MAC will be numerically smal l, mean ing that it is inverse ly
5
proportiona l to the anesthetic's potency. Drug C, which has the smal lest MAC va lue, is thus the most potent.
6 The blood solubility of an anesthetic is the physical property that determines both the speed of induction and
7 time to recovery. Drugs with low blood so lubility, such as nitrous oxide, will rapidly induce anesthesia, and
patients will recover quickly. I n contrast, an anesthetic gas with high blood solubility, such as ha lothane, wi ll
8
have a longer t ime to induction and a slower time to recovery. Therefore, drug C, wh ich has the lowest blood
9 so lubility, wi ll have the most rapid induction.
10 Halothane Minimum alveolar concentration Nitrous oxide Anesthetic Anesthesia Pulmonary alveolus Potency (pharmacology) Solubility Inhalational anaesthetic

11
A is not correct. 7°/o chose this.
12 The MAC for a dr ug is inversely related to its potency. Drug A has a larger MAC than drug D and will be less
13 potent than drug D.
Media access control Mid-American Conference
14
15 B is not correct. 4°/o chose this.
16 The solubility of inha led anesthetic drugs is described using the blood-gas partit ion coefficient, which is the ratio
• 17 of the concentration of the drug in the blood to the concentration of the drug in the gas adjacent to the blood .
For drug B, with a so lubility of 1.0 (there is an equal concentration of drug Bin the blood and in the gas). For
• 18
drug C, with its solubility value of 0. 1, the concentration of dr ug C in blood is 1/ 10th that of the concentration in
• 19 the gas. Therefore dr ug B is more soluble in blood than dr ug C.
• 20 Partition coefficient Anesthetic Solubility Arterial blood gas Inhalational anaesthetic

• 21 D is not correct. 12% chose this .


6
lock
s
Suspend
0
End Block
Item: 16 of 38 ~. I • M k <:] t> al ~· ~
QIO: 1446 .l. ar Previous Next lab 'lifllues Notes Calculator

1 •
B is not correct. 4°/o chose this.
2
The solubility of inha led anesthetic drugs is described using the blood-gas partition coefficient, which is the ratio
3 of the concentration of the drug in the blood to the concentration of the drug in the gas adjacent to the blood .
4 For drug B, with a so lubility of 1.0 (there is an equal concentration of drug Bin the blood and in the gas). For
5
drug C, with its solubility value of 0. 1, the concentration of dr ug C in blood is 1/ 10th that of the concentration in
the gas. Therefore dr ug B is more soluble in blood than dr ug C.
6 Partition coefficient Anesthetic Solubility Arterial blood gas Inhalational anaesthetic
7
D is not correct. 12% chose this.
8
An anesthetic with greater blood solubility will have a slower induction t ime. Dr ug D, which is more so luble in
9
blood than drug C, wi ll therefore induce anesthesia less rapidly than dr ug C.
10 Anesthesia Anesthetic Solubility

11
E is not correct. 9°/o chose this.
12 Recovery t ime from anesthesia is based on the blood solubility of a gas. Gases with high blood solubi lity result
13 in slower recovery than for drugs that have lower blood solubi lity. Dr ug B has the highest blood solubility of al l
14 those that are listed and thus will have the slowest recovery time of all.
Solubility Anesthesia
15
16
• 17 Bottom Line:
• 18 The MAC of an inha led anesthetic is inversely proportional to its potency. Blood solubility deter mines the speed
• 19 of induction and t ime to recovery. Low blood solubility ind icates an agent that has a rapid onset and rapid
• 20
offset, whereas an agent with high blood solubility has a slow onset and slow t ime to recovery .
Anesthesia Anesthetic Potency (pharmacology) Solubility
• 21

6
lock
s
Suspend
0
End Block
Item: 16 of 3 8 ~ 1 • M k -<:J 1>- Jil ~· !:';-~
QIO: 1446 ..L ar Pre v ious Next Lab fli!ltues Not es Calcula t o r
& &
1
2 FA17 p 518.6
3 Anesthetics- general Ct S drugs must be lipid soluble (cross the blood-brain barrier) or be acti,·ely transported.
4 principles Drugs " ith l solubility in blood = rapid induction and reco\ery limes.
5
Drugs with t solubility in lipids = t potency = I
6 \I C
7 \lAC = ~ l inima l Alveolar Concentration (of inhaled anesthetic) required to prevent 50% of
8 subjects from moving in response to no-: ions stimulus (eg, skin incision}.
Examples: nitrous oxide (i 20 ) has l blood and lipid solubilit), and thus fast induction and IO\\
9
potency. Halothane, in contrast, has t lipid and blood solubility, and thus high potency and slow
10
induction.
11
12
FA17p519.1
13 Inhaled anesthetics Desflurane, halothane, enflurane, isoAurane, sevoAuranc, methoxyflurane, l 20.
14
MECHANISM Mechanism unknown.
15
EFFECTS Myocardial depression, respiratory depression, nausea/emesis, t cerebral blood flow 0 cerebral
16 metabolic demand).
• 17
ADVERSEEFFECTS Hepatotoxicity (halothane), nephrotoxicity (methoxyflurane), procon\'ulsant (enflurane,
• 18 epileptogenic}, expansion of trapped gas in a body Ca\'ity (N 20 ).
• 19 Malignant hyperthermia-rare, life-threatening condition in which inhaled anesthetics or
• 20 succinylcholine induce fe,er and se,cre muscle contractions. Susceptibility is often inherited as
• 21
autosomal dominant with \'ariable penetrance. lutations in ,·oltage-sensiti,·e ryanodine receptor
• " " " ' U" O t r .... 2T" rola,ra f ..,.u...... r .......,..,..,l,.,.. .-.... ;,. r .ot;",,), ,.,.... Tronlt··n onJ-• ..J .... nf--r,.looo ( , - · .... n.n....l;.,a, t"O-"I:U.......u •

a
Lock
s
Suspend
8
End Bl ock
Item: 16 of 3 8 ~ 1 • M k -<:J 1>- Jil ~· !:';-~
QIO: 1446 ..L ar Pre v ious Next Lab fli!ltues Not es Calcula t o r

1 9 g I ( ) p
principles Drugs with ~ solubi l ity in blood = rapid induction and recovery limes.
2
3 Drugs with t solubility in lipids= t potency = I
\1 c
4
lAC = ~1 in i ma l Ah-eolar C oncentration (of inhaled anesthetic) required to pre\·ent 50% of
5
subjects from moving in respome to no-:ious stimulus (eg, skin incision).
6
Examples: nitrous o>.idc (1 20 ) has l blood and lipid ~olubil it), and thus fast induction and Jo,,
7 potency. Halothane, in contrast, has t lipid and blood solubility, and thus high potencr and slow
8 induction.
9
10 FA17 p 519.1
11 Inhaled anesthetics Desflurane, halothane, enflurane, i ofluranc. scvofl uranc, methoxyflurane, f 20 .
12 MECHANISM Mechanism unknO\\ n.
13 EFFECTS Myocardial depression, respiratory depression, nausea/emesis, t cerebral blood fl ow (~ cerebral
14 metabolic demand).
15 ADVERSE EFFECTS Hepatotoxicity (halothane), nephrotoxicity (methoxyflurane), proconvulsant (enflurane,
16 epileptogenic), expansion of tmppcd g:~s in a body cavity (N20 ).
Malignant hyperthermia- rare, life-threatening cond ition in which inhaled anesthetics or
• 17
succinylcholine induce fever and sc,·cre muscle contractions. Susceptibility is often inherited as
• 18
autosomal dominant with variable penetrance. ~ lulations in voltage-sensitive ryanodine receptor
• 19 cause t Ca 2+ release from sarcoplasm ic reticulum. Treatment: dantrolene (a ryanod ine receptor
• 20 antagonist).
• 21

a
Lock
s
Suspend
8
End Bl ock
Item: 17 of 38 ~. I • M k <:] t> al ~· ~
QIO: 4199 .l. ar Previous Next Lab 'lifllues Notes Calculator

1 •
A 67-year-old man arrives at the physician's office complaining of tremor. On further question ing, he states ~~AI
2 that his hands shake when he lays them on the table. The shaking subsides when he moves his hands
3 around . He also reports fee ling clumsy and unbalanced. Based on his symptoms, the physician prescribes a
4 medication that is also used as an antiviral agent.
5
6 What is the presumed mechanism of action of this med ication'
7 :

8 A. Blocks the bind ing of epinephrine to ~-adrenergic receptors


9 B. Converts to dopamine in the central nervous system and binds to dopamine receptors
10
C. Prevents the reuptake of serotonin from synapses
11
D. Promotes release of dopamine from presynaptic fibers
12
13 E. Se lectively inhibits monoamine oxidase-B
14
15
16
• 17

• 18
• 19
• 20
• 21

6
lock
s
Suspend
0
End Block
Item: 17 of 38 ~. I • M k <:] t> al ~· ~
QIO: 4199 .l. ar Previous Next Lab 'lifllues Notes Calculator

1 •

2 The correct answer is D. 58°/o chose this.


3 The patient has been prescribed amantadine, wh ich is an antiviral med ication used in the treatment of influenza
4
and rubel la. In add ition, it is used in the treatment of Parkinson disease because it promotes the synthesis and
release of dopamine and prevents its reuptake. Adverse effects include ataxia, dizziness, and slurred speech .
5 Due to its action as a dopamine agon ist, acute toxic psychosis may result in the event of an amantadine
6 overdose.
Dopamine agonist Amantadine Dopamine Ataxia Antiviral drug Psychosis Agonist Influenza Parkinson' s disease Reuptake Rubella Pharmaceutical drug Dizziness
7
Toxicity Dysarthria
8
9 A is not correct. 5°/o chose this.
10 ~-Adrenergic antagonists are particu larly used for the management of card iac arr hthymias. They have no use as
11 antivira l agents. This class includes propranolo l, carvedilo l, labeta lol, metoprolol, and atenolol.
labetalol Propranolol Atenolol Metoprolol Carvedilol
12
13 B is not correct. 18% chose this.
14 Dopamine does not cross the blood-brain barr ier. However, its precursor, levodopa, can cross the blood-brain
barrier. After crossing the blood-brain barrier, levodopa is decarboxylated to dopamine and binds to dopamine
15
receptors.
16 l-DOPA Dopamine Blood-brain barrier Decarboxylation Receptor (biochemistry) Dopamine receptor

17
C is not correct. 7°/o chose this .
• 18
Se lective serotonin reuptake inhibitors prevent the reupta ke of serotonin in synapses. They are used in the
• 19 treatment of depression and include citalopram, escita lopram, fluoxetine, fluvoxamine, paroxetine, and
• 20 sertral ine .
Sertraline Escitalopram Citalopram Fluvoxamine Paroxetine Fluoxetine Serotonin Selective serotonin reuptake inhibitor Reuptake Synapse
• 21

6
lock
s
Suspend
0
End Block
Item: 17 of 38 ~. I • M k <:] t> al ~· ~
QIO: 4199 .l. ar Previous Next Lab 'lifllues Notes Calculator

1 •
B is not correct. 18% chose this.
2
Dopamine does not cross the blood-brain barr ier. However, its precursor, levodopa, can cross the blood-brain
3
barrier. After crossing the blood-brain barrier, levodopa is decarboxylated to dopamine and binds to dopamine
4 receptors.
l-DOPA Dopamine Blood-brain barrier Decarboxylation Receptor (biochemistry) Dopamine receptor
5
6 C is not correct. 7°/o chose this.
7 Se lective serotonin reuptake inhibitors prevent the reuptake of serotonin in synapses. They are used in the
8 treatment of depression and include citalopram, escita lopram, fluoxetine, fluvoxamine, paroxetine, and
sertral ine.
9
Sertraline Escitalopram Citalopram Fluvoxamine Paroxetine Fluoxetine Serotonin Selective serotonin reuptake inhibitor Reuptake Synapse
10
Major depressive disorder Depression (mood)
11

12
E is not correct. 12% chose this.
Se leg iline inh ibits monoamine oxidase-B and increases the availability of dopamine. It is an option for the ear ly
13
treatment of Parkinson disease due to its modest effect and can be used in combination with levodopa .
14 Selegiline Dopamine l-DOPA Monoamine oxidase B Parkinson' s disease Monoamine neurotransmitter
15
16
Bottom Line:
17
• 18 Parkinson disease (PD) is primarily due to an imbalance between dopamine and acetylchol ine. Drugs that are
used to treat PD generally increase dopaminergic activity or decrease chol inergic activity .
• 19 Dopamine Acetylcholine Cholinergic Dopaminergic Parkinson' s disease
• 20
• 21

6
lock
s
Suspend
0
End Block
Item: 17 of 3 8 ~ 1 • M k -<:J 1>- Jil ~· !:';-~
QIO: 4199 ..L ar Pre v ious Next Labfli!llues Not es Calcula t o r
&
- - -
&
1

2 FA17p517.1
3
Parkinson disease Parkinsonism is due to loss of dopaminergic neurons and excess cholinergic activity.
4 drugs Bromocriptine, .\ mantadine, Le,·odopa (with carbidopa), Selegiline (and COMT inhibitors),
5 Antimuscarinics (B.\ LS.\ ).
6 STRATEGY AGENTS

7 Dopamine agonists Ergot- Bromocriptine


1
on-ergot (preferred)-pramipexole, ropinirole
8
9 f dopamine availability Amantadine (t dopamine release and l dopamine reuptake): toxicity= ataxia, livedo reticularis.
10 f L-DOPA availability Agents prevent peripheral (prc-BBB) L-DOPA degradation - t L-DOPA entering CNS - t central
L-DOPA available for com·ersion to dopamine.
11
Lc,·odopa (L-DOPA)/carbidopa-carbidopa blocks peripheral conversion of L-OOP to
12
dopamine by inhibiting DOP decarboxylase. Also reduces side effects of peripheral L-DOPA
13 conversion into dopam ine (eg, nausea, vomiting).
14 Entacapone, tolcapone-prcvent periphera l L-OOP degradation to 3-0-methyldopa (3-0 fD)
15 by inhibiting COMT.
16 Prevent dopamine Agents act centrally (post-BBB) to inhibit breakdown of dopamine.
17
breakdown • Selegiline-blocks conversion of dopamine into DOPAC by selectively inhibiting MAO-B.
• Tolcapone-bloeks com·crsion of dopamine to 3-methoxytyramine (3-MT) by inhibiting central
. 18
COMT.
• 19
Curb excess cholinergic Benztropine, trihexyphenidyl (Antimuscarinic; impro,·es tremor and rigidity but has little effect on
• 20
activity .
bradrkinesia .
in Parkinson disease). Park rour 1ercedes-Ben z.
• 21

a
Lock
s
Suspend
8
End Bl ock
Item: 17 of 38 ~ 1 • M k -<:J 1>- Jil ~· !:';-~
QIO: 4199 ..L ar Previous Next Labfli!llues Notes Calculator
& &
1
Curb excess cholinergic Benztropine, trihexyphenidyl (Antimuscarinic; impro,·es tremor and rigidity but has little effect on
2 bradykinesia in Parkinson disease). Park your Mercedes-Benz.
activity
3
CI~ULATION
4 3-0MO

5
L·OOPA
---~
DOC C:J"' 1--~ ->---

6
7
BL()()().
BRAIN
BARRIER
--{
)
8
9
L·OOPA
10
11
PRESYNAPTIC
12 TERMINAL FROM THE Dopamine
Tolcapone

l
SUBSTANTIA NIGRA
13
14
J
15
16
Reuptake
17 Selegiline
Rasagillne
. 18 0
0 0
• 19
DOPAJoiiNE
0
AVAJLABIUTY 0
• 20 0
0 0 __ o
• 21

Amarodo~

- ~

a
Lock
s
Suspend
8
End Block
Item: 17 of 3 8 ~ 1 • Ma rk -<:J I> ~ £!1}>'
• !!":-~
QIO: 4199 ..L Prev ious Next Lab lues Not es Cal culat o r

& &
1
LEn~aeapone
2
3
BLOOD-
BRAIN
BARRIER --{ Tolcapone

4
L·OOPA
5
6 ooc)
PRESYNAPTK
7 TERMINAL FROM THE Oopanune
SUBSTANTlA NIGRA Tolcapone
8
9
"!:
. _. J..MT
-. OOPAC

10
11
12 Setegiline
Rasaglhne
13 0
14 0 0
0
15 0
16 Amantadine 0 0
17
. 18
POSTSYNAPTIC
eJ Dopamine r«@l)lors

. 19 TERMINAL IN
THE STRIATUM
• 20
. 21

a
Lock
s
Suspend
8
End Bl ock
Item: 17 of 3 8 ~ 1 • M k -<:J 1>- Jil ~· !:';-~
QIO: 4199 ..L ar Pre v ious Next Labfli!llues Not es Calcula t o r
& &
1
FA17 p489.1
2
Movement disorders
3
DISORDER PRESENTATION CHARACTERISTIC LESION NOTES
4
Akathisia Restlessness and intense urge Can be seen with neuroleptic
5 to mo,·e use or in Parkinson disease.
6 Asterixis Extension of wrists causes Associated with hepatic
7 '·fl apping" motion encephalopathy, \ ilson
8 disease, and other metabolic
9 derangements.
10 Athetosis Slow, snake-like, \\Tithing Basal ganglia
movements; especially seen in
11
the fingers
12
Chorea Sudden, jerky, purposeless Basal ganglia Chorea = dancing.
13
movements Sydenham chorea seen in acute
14 rheumatic fever.
15 Dystonia Sustained, involuntary muscle Writer's cramp, bleph::uospasm,
16 contractions torticoll is.
17 Essential tremor lligh-frequency tremor Often famili al. Patients oft en
. 18 with sustained posture self-medicate with alcohol,
• 19 (eg, outstretched arms), which ' tremor amplitude.
• 20
worsened with mo,·ement or Treatment: nonselective
when anxious ~ -bl ockers (eg, propranolol),
• 21
• nrimiclont>.
a
Lock
s
Suspend
8
End Bl ock
Item: 17 of 3 8 ~ 1 • M k -<:J 1>- Jil ~· !:';-~
QIO: 4199 ..L ar Pre v ious Next Labfli!llues Not es Calcula t o r

2
Dystonia Sustained, involuntary muscle Writer's cramp, blepharospasm,
contractions torticollis.
3
Essential tremor ll igh-frequency tremor Often familial. P<llients often
4
with sustained posture self-medicate with alcohol,
5
(eg, outstretched arms), ,,·hich I tremor amplitude.
6 worsened with mo,·ement or Treatment: nonsclecti,·e
7 when anxious ~-blockers (eg, propranolol),
8 primidone.
9 Hemiballismus Sudden, wild Railing of I arm Coni ra lateral subthalamic Pronounce "I lalf-of-bod)
10 +1- ipsilateral leg nucleus (eg, lacunar stroke) ball is tic."
Contralateral lesion.
11
Intention tremor Slow, zigzag motion when Cerebellar dysfunction
12
pointing/extending toward a
13
target
14
Myoclonus Sudden, brief, uncontrol led Jerks; hiccups; common in
15 muscle contraction metabolic abnormalities such
16 as renal and liver fai lure.
17 Resting tremor Uncontrolled movement of distal Substantia nigra (Parkinson Occurs at rest; "pill-rolling
. 18 appendages (most noticeable disease) tremor" of Parkinson disease.
. 19 in hands); tremor alleviated br When you park your car, it is
intentionaJ movement at rest.
• 20
. 21

a
Lock
s
Suspend
8
End Bl ock
Item: 18 of 3 8 ~ 1 • M k -<:J 1>- Jil ~· !:';-~
QIO: 1450 ..L ar Pre v ious Next Lab fli!ltues Not es Calcula t o r

IAA]
A A
• 18
A 65-year-old woman with a history of anxiety presents to her doctor with renewed anxiety. She had been
• 19 prescribed barbiturates in the past, but has heard that benzod iazepines are now more commonly prescribed .
• 20 She wants to know how the medications are different .
• 21
• 22 Which of the following statements accurately contrasts the features of benzodiazepines with barbiturates?
• 23 :
• 24 A. Benzodiazepines are less safe for the treatment of anxiety
• 25 B. Benzodiazepines have a higher incidence of respiratory depression
• 26
C. Benzodiazepines have a shorter half-life
• 27
D. Benzodiazepines increase the duration of the chloride channel opening
• 28
• 29 E. Benzodiazepines increase the frequency of the chloride channel opening
o30
• 31
• 32
• 33
o34
o35
• 36
• 37
• 38

a
Lock
s
Suspend
8
End Bl ock
Item: 18 of 38 ~. I • M k <:] t> al ~· ~
QIO: 1450 .l. ar Previous Next lab 'lifllues Notes Calculator

18
• 19 The correct answer is E. 75°/o chose this .
• 20
Benzodiazepi nes an d barbitu rates differ in t heir effect on t he ch loride channel adj acent to the y -ami nobutyric
acid receptor. Benzodiazepi nes increase t he freq uency of the channe l's open ing , whil e ba r biturates increase t he
• 21 duration the channel is open.
• 22 Chloride channel Benzodiazepine Barbiturate Chloride

• 23
A is not correct. 2°/o chose this .
• 24 Benzodiazepi nes have large ly replaced ba r biturates in the treatment of anxiety because of their more f avora ble
• 25 after-effect profile. They are less likely tha n barbiturates to lead to respiratory depression and central cardiac
• 26 depression .
Hypoventilation Benzodiazepine Barbiturate Anxiety Major depressive disorder Depression (mood)
• 27

• 28 B is not correct. 4°/o chose this .


• 29 Barbitu rates are far more likely than benzodiazepi nes to cause centra l respiratory and ca rdiac depression an d
are significantly less safe .
o30 Benzodiazepine Barbiturate Major depressive disorder Depression (mood)
• 31
C is not correct. 6°/o chose this .
• 32
Half- lif e depends on the pharm aco kinetics of the particular medication and not its class. Some ha lf-li ves m ay be
• 33
longer, som e shorter.
o34 Half-life Pharmacokinetics Pharmaceutical drug

o35
D is not correct. 13% chose this .
• 36
Barbitu rates, not benzod iazepines, increase the durat ion of chlor ide channel opening .
• 37 Chloride channel Benzodiazepine Barbiturate

• 38

6
lock
s
Suspend
0
End Block
Item: 18 of 38 ~. I • M k <:] t> al ~· ~
QIO: 1450 .l. ar Previous Next lab 'lifllues Notes Calculator
.. .
18 Hypoventilation Benzodiazepine Barbiturate Anxiety Major depressive disorder Depression (mood)
• 19
B is not correct. 4°/o chose this .
• 20
Barbitu rates are fa r more likely than benzodiazepi nes to cause centra l respiratory and ca rdiac depression an d
• 21 are significantly less safe .
• 22 Benzodiazepine Barbiturate Major depressive disorder Depression (mood)

• 23 C is not correct. 6°/o chose this .


• 24 Half- lif e depends on the pharm acokinetics of the particular medication and not its class. Some ha lf-li ves m ay be
• 25 longer, som e shorter .
Half-life Pharmacokinetics Pharmaceutical drug
• 26
• 27 D is not correct. 13% chose this .
• 28 Barbitu rates, not benzod iazepines, increase the duration of chloride channel opening .
Chloride channel Benzodiazepine Barbiturate
• 29
o30
• 31 Bottom Line:
• 32 Benzodiazepi nes increase the freq uency of ch loride chan nel opening ( rem em be r th is mnemonic: "Ben
• 33 zapzapzaps the channels" ), whereas ba r biturates increase t he duration t hat t he ch loride chan nel rem ains
o34 open ( rem em be r "Barrrbara keeps it open longer" ) . BenzoDIAZEPINES are GABA DEPENDENT, w heras
barbiturates at high doses can open the channe l without GABA (eg, GABA mi metics) . Both dr ugs effectively
o35
oppose excess neuronal firing by hy perpolarizing t he membrane, making depolarization less likely .
• 36 Chloride channel Gamma-Aminobutyric acid Benzodiazepine Depolarization Barbiturate Hyperpolarization (biology) Chloride

• 37
• 38 ~

6
lock
s
Suspend
0
End Block
Item: 18 of 3 8 ~ 1 • M k -<:J 1>- Jil ~· !:';-~
QIO: 1450 ..L ar Pre v ious Next Lab fli!ltues Not es Calcula t o r
A
• • A

FA17 p 515.2

Diazepam, lorazcpam, triazolam, temazepam, oxazepam, midazolam, chlordiazepoxide,


alprazolam.
Facilitate CABA,\ action by t frequency of "Frcnzodiazepines" t frequency.
Cl- channel opening. l RE~ I sleep. ~l ost Benzos, barbs, and alcohol all bind the
ha\·e long half-li\·es and acti\·e metabolites C B .\receptor. which is a ligand-gated CJ-
(exceptions (ATO~ I]: Alprazolam, Triazolam, channel.
Oxazepam, and :\Iidazolam are short acting Oxazepam, Temazepam, and Lorazepam are
- higher addictive potential ). metabolized O utside T he Li\·er
Anxiety, spasticity, status epilepticus (lorazepam
and diazepam), eclampsia, detoxification
(especially alcohol withdrawai-DTs), night
terrors, sleepwalking, general anesthetic
(amnesia, muscle relaxation), hypnotic
(insomn ia).
Dependence, additive C S depression effects
with alcohol. Less risk of respiratory depression
and coma than \\ ith barbiturates.
Treat overdose with Aumazenil (competit ive
antagonist at CABA benzodiazcpine receptor).
Can precipitate seizures by causing acute
benzodiazepine withdrawal.

a
Lock
s
Suspend
8
End Bl ock
Item: 18 of 3 8 ~ 1 • M k -<:J 1>- Jil ~· !:';-~
QIO: 1450 ..L ar Pre v ious Next Lab fli!ltues Not es Calcula t o r
., , ~ t ~ •• •
18
and diazepam}, eclampsia, detoxification
• 19 (especially alcohol withdrawal-DTs}, night
• 20 terrors, sleepwalking, general anesthetic
• 21 (amnesia, muscle relaxation), hypnotic
• 22
(insomnia).

• 23
ADVERSE EFFECTS Dependence, additi,·e C ·s
depression effects
with alcohol. Less risk of respiratory depression
• 24
and coma than" ith barbiturates.
• 25
Treat o,·erdose with Aumazenil (competiti' e
• 26 antagonist at CABA benzodiazepine receptor).
• 27 Can precipitate seizures b) causing acute
• 28 benzodiazepine withdrawal.
• 29
o30 FA17 p 515.1

• 31
Barbiturates Phenobarbital, pentobarbital, thiopental, secobarbital.

• 32
MECHANISM Facil itate CABAA action by f duration orct- channel opening, thus t neuron firing (barbidunltCs
f duration). Contraindicated in porph)'ria .
• 33
CLINICAL USE Sedative for anxiety, seizures, insomnia, induction of anesthesia (thiopental).
o34
o35
ADVERSE EFFECTS Respiratory and cardio,·ascular depression (can be fatal ); Cl S depression (can be exacerbated by
alcohol use}; dependence; drug interactions (induces cytochrome P-450).
• 36
Overdose treatment is supportive (assist respiration and maintain BP).
• 37
• 38

a
Lock
s
Suspend
8
End Bl ock
Item: 19 of 3 8 ~ 1 • M k -<:J 1>- Jil ~· !:';-~
QIO: 440 9 ..L ar Pre v ious Next Lab fli!ltues Not es Calcula t o r

IAA]
A A

18
A 25-year-old day laborer on a fru it farm is brought to the emergency department by a coworker because he
• 19 is having diarrhea and difficulty breathing after their lunch break. His heart rate is 50/mi n, blood pressure is
• 20 80/60 mm Hg, respiratory rate is 23/ m in, and temperature is 36.9°C (98.4°F). On examination he is
• 21
diaphoretic, drool ing, and unable to answer questions coherently .

• 22
What should the physician give him to treat this condition?
• 23
• 24 :

• 25
A. Dantrolene
• 26 B. Pralidoxime
• 27 C. Pyridostigmine
• 28
D. Succinylcholine
• 29
o30
• 31
• 32
• 33
o34
o35
• 36
• 37
• 38

a
Lock
s
Suspend
8
End Bl ock
Item: 19 of 38 ~. I • M k <:] t> al ~· ~
QIO: 4409 .l. ar Previous Next lab 'lifllues Notes Calculator


18
19
The correct answer is B. 59°/o chose this.
This patient is suffering from acute organophosphate poisoning, most likely due to parath ion or a related
0 20
insecticide. These acetylcholinesterase inh ibitors cause symptoms of increased cholinergic stimulation such as
0
21 diarrhea, miosis, bronchospasm, bradycard ia, sweating, and sa livation. Bradycardia and hypotension are usually
0
22 present in moderate to severe poisonings . The symptoms are often irreversible after a short period of t ime, so it
0
23
is important to administer either atropine, an anticholinergic medication, or pralidoxime, a compound that
regenerates active cholinesterase, as soon as possible.
0 24 Anticholinergic Atropine Miosis Organophosphate Bronchospasm Insecticide Parathion Acetylcholinesterase Cholinergic Pralidoxime Bradycardia Hypotension
0 25
Acetylcholinesterase inhibitor Cholinesterase Diarrhea Saliva Organophosphate poisoning Perspiration Pharmaceutical drug
0
26
0 27
A is not correct. 8°/o chose this.
Dantrolene prevents the release of calcium from the sarcoplasmic reticulum of skeletal muscle. It is used to
0
28
treat neuroleptic malignant syndrome and malignant hyperthermia.
0 29 Neuroleptic malignant syndrome Dantrolene Malignant hyperthermia Hyperthermia Sarcoplasmic reticulum Skeletal muscle Antipsychotic Calcium Cancer Muscle

o30
C is not correct. 23% chose this.
0
31
Pyridostigmine is an anticholinesterase used to treat myasthenia gravis. Its use would worsen organophosphate
0
32 poisoning .
0
33 Myasthenia gravis Organophosphate Pyridostigmine Acetylcholinesterase inhibitor Organophosphate poisoning

0 34 D is not correct. 10% chose this.


0 35 Succinylchol ine is a depolarizing neuromuscu lar blocking drug used primarily in anesthesiology during
0
36 intubation. I t is potentiated by chol inesterase inhibitors and is not used to treat their toxicity.
Suxamethonium chloride Neuromuscular-blocking drug Acetylcholinesterase inhibitor Cholinesterase Neuromuscular junction Intubation Depolarization Toxicity
0 37
0 38 ~

6
lock
s
Suspend
0
End Block
Item: 19 of 38 ~. I • M k <:] t> al ~· ~
QIO: 4409

18
.l.
-·- ---- ar
-
Previous
:.;- . - --- - - .. - .. .-
Next lab 'lifllues Notes Calculator

Anticholinergic Atropine Miosis Organophosphate Bronchospasm Insecticide Parathion Acetylcholinesterase Cholinergic Pralidoxime Bradycardia Hypotension
19
Acetylcholinesterase inhibitor Cholinesterase Diarrhea Saliva Organophosphate poisoning Perspiration Pharmaceutical drug
0 20
A is not correct. 8°/o chose this.
0
21
Dant rolene prevents the release of ca lcium from the sa rcoplasm ic ret icu lum of skeleta l muscle . It is used to
0
22
t reat neuroleptic mal ignant syndrome and mal ignant hyperthermia.
0
23 Neuroleptic malignant syndrome Dantrolene Malignant hyperthermia Hyperthermia Sarcoplasmic reticulum Skeletal muscle Antipsychotic Calcium Cancer Muscle

0 24
C is not correct. 23% chose this.
0 25
Pyridostigmine is an antichol inesterase used t o treat myasthenia gravis. Its use would worsen organophosphat e
0
26 poison ing .
0 27 Myasthenia gravis Organophosphate Pyridostigmine Acetylcholinesterase inhibitor Organophosphate poisoning

0
28 D is not correct. 10% chose this.
0 29 Succinylcholine is a depolarizing neuromuscu lar blocking d rug used prima rily in anesthesiology during
o30 intubation. I t is pot ent iated by cholinesterase inh ibit ors and is not used to treat their toxicity .
Suxamethonium chloride Neuromuscular-blocking drug Acetylcholinesterase inhibitor Cholinesterase Neuromuscular junction Intubation Depolarization Toxicity
0
31
0
32
0
33 Bottom Line:
0 34 Organophosphat e poison ing result s in excessive cholinergic st imu lation that can be summed up by the
0 35 mnemonic DUMBBELSS : Diarr hea, Urination, Miosis, Bronchospasm, Bradycardia, Excit ation of skeleta l
0
36 muscle and the central nervous system, Lacr imation, Sweating, and Salivation). Pral idoxime and atropine are
antidotes .
0 37
Atropine Organophosphate Central nervous system Cholinergic Pralidoxime Organophosphate poisoning Skeletal muscle Mnemonic Diarrhea Nervous system
0 38 ~

6
lock
s
Suspend
0
End Block
Item: 19 of 3 8 ~ 1 • M k -<:J 1>- Jil ~· !:';-~
QIO: 440 9 ..L ar Pre v ious Next Lab fli!ltues Not es Calcula t o r
A A

18
19 FA17 p232.2
• 20 Cholinesterase Often due to organophosphates, such as DU\1BBELSS.
• 21 inhibitor poisoning parathion, that irreversibly inhibit AChE. Organophosphates are often components of
• 22 Causes Diarrhea, Urination, \I iosis, insecticides; poisoning usually seen in farmers.
Bronchospasm, Bradycardia, E\citation Antidote-atropine (competitive inhibitor) +
• 23
of skeletal muscle and CNS, Lacrimation, pmlidoxime (regenerates AChE if gi,cn early).
• 24
Sweating. and Salivation . .\ lay lead to
• 25 respiratory failure if untreated .
• 26
• 27 FA17 p 239.1
• 28
Specific toxicity TOXIN TREATMENT
• 29 treatments Acetaminophen N-acetylcysteine (replenishes glutathione)
o30
AChE inhibitors, organophosphates Atropine> pralidoxime
• 31
Antimuscarinic, anticholinergic agents Physostigmine, control hyperthermia
• 32
Arsenic Dimercaprol, succimer
• 33
Benzocl iazepines Flumazen il
o34
~-blockers Atropine, glucagon
o35
Carbon monoxide 100% 0 2, hyperbaric 0 2
• 36
• 37
Copper Penicillamine, trientine (Copper penny)
• 38 Cyanide 1'\itrite +thiosulfate, hvdroxocobalamin

a
Lock Suspend
s 8
End Bl ock
Item: 19 of 3 8 ~ 1 • M k -<:J 1>- Jil ~· !:';-~
QIO: 440 9 ..L ar Pre v ious Next Lab fli!ltues Not es Calcula t o r


18 FA17 p239.1
19 Specific toxicity TOXIN TREATMENT
• 20 treatments Acetaminophen N-acetylcysteine (replenishes glutathione)
• 21
AChE inhibitors, organophosphates Atropine> pralidoxime
• 22
Antimuscarinic, anticholinergic agents Physostigmine, control hyperthermia
• 23
Arsenic Dimercaprol. succi mer
• 24
Benzodiazepines Flumazenil
• 25
~-blockers Atropine, glucagon
• 26
Carbon monoxide 100% 0 2, hyperbaric 0 2
• 27
Copper Penicillamine, trientine (Copper penny)
• 28
• 29 Cyanide Nitrite+ thiosulfate, hydroxocobalamin
o30
Digitalis (digoxin) Anti-dig Fab fragmen ts
• 31 Heparin Protamine sulfate
• 32 Iron Deferoxam ine, deferasirox, deferiprone
• 33 Lead ED'lf\, dimercaprol, succimer, penicil lamine
o34 l\lcrcury Dimercaprol, succimcr
o35 Methanol, ethylene glycol (antifreeze) F'omepizole >ethanol, dialysis
• 36 ~ lethemoglobin 1\ lethvlene blue, ,·itamin C
'
• 37 OpiO ids 'alOxO nc
• 38

a
Lock
s
Suspend
8
End Bl ock
Item: 19 of 3 8 ~ 1 • M k -<:J 1>- Jil ~· !:';-~
QIO: 440 9 ..L ar Pre v ious Next Lab fli!ltues Not es Calcula t o r
• g g • ) g
18
Arsenic Dimercaprol, succimer
19
• 20 Benzodiazepines Flumazen il
• 21 ~-blockers Atropine, glucagon
• 22 Carbon monoxide 100% 0 2, hyperbaric 0 2
• 23 Copper Penicillamine, trientine (Copper penny)
• 24 Cvanide ~itrite + thiosulfate, hrdroxocobalamin
'
• 25 Digitalis (digoxin) Anti-dig Fab fragments
• 26 Heparin Prolamine sulfate
• 27 Iron Deferoxamine, deferasirox, deferiprone
• 28 ED'lA, dimercaprol. succimer, penicillamine
Lead
• 29
l\Iercury Dimercaprol, succimer
o30
Methanol, ethylene glycol (antifreeze) Fomepizole >ethanol, dialysis
• 31
1\lethemoglobin 1\lethylene blue, vitamin C
• 32
O piO ids aiO xO nc
• 33
Sa Iicyla tes aiiC03 (a lkali nize urine), dialysis
o34
TCAs 'aHC03
o35
Warfarin ilamin K (delayed effect), fresh frozen plasma
• 36
(immediate)
• 37
• 38

a
Lock
s
Suspend
8
End Bl ock
Item: 20 of 3 8 ~ 1 • M k -<:J 1>- Jil ~· !:';-~
QIO: 52 79 ..L ar Pre v ious Next Lab fli!ltues Not es Calcula t o r

IAA]
A A

18
A 35-year-old woman presents to her primary care physician complaining of lower back pain. The physician
19 completes a ful l neurologic examination, which reveals a normal patellar deep tendon reflex. X-ray of the
• 20 lumbar/sacral spine is shown . The physician asks the patient if her mother took a certain med ication during
• 21
her pregnancy, and the patient confirms his suspicion .
---~
• 22
• 23
• 24
• 25
• 26
• 27
• 28
• 29
o30
• 31
• 32
• 33
• 34
• 35 I mage courtesy of Wikimedia Commons

• 36
• 37 Wh ich of the following drugs did the patient's mother most likely take during the pregnancy?
• 38 :

a
Lock
s
Suspend
8
End Bl ock
18
19
• 20
• 21
• 22
• 23
• 24
• 25
• 26
• 27
I mage courtesy of Wikimedia Commons
• 28
• 29 Which of the fol lowing drugs did the patient's mother most likely take during the pregnancy?
o30
:
• 31 A. Aspirin
• 32
B. Meropenem
• 33
• 34 C. Seleg iline
• 35 D. Sumatri ptan
• 36
E. Valproic acid
• 37
• 38

a
Lock
s
Suspend
8
End Block
Item: 20 of 38 ~. I • M k <:] t> al ~· ~
QIO: 5279 .l. ar Previous Next lab 'lifllues Notes Calculator


18
19 The correct answer is E. 79°/o chose this.
20 Although the patel lar deep tendon reflex is norma l, the X-ray shows that part of the Sl vertebra is not
• 21 completely fused (as shown by the circle in the stem image). These findings together ma ke the diagnosis of
• 22
spina bifida occu lta more likely than spina bifida cystica, since there are no neuro log ic deficits. Spina bifida
occu lta is a neural tube defect that occu rs in the first month of gestation and is associated with several materna l
• 23 factors, including folate deficiency, anticonvu lsant medications, and diabetes. Valproic acid, which is used as
• 24 both an antiepi leptic and a mood stabi lizer, can cause neural tube defects in the fetus .
Neural tube defect Stretch reflex Anticonvulsant Spina bifida Valproate Vertebra Folate deficiency Folic acid Diabetes mellitus Neural tube Reflex Fetus Tendon
• 25
• 26 X-ray Gestation Neurology

• 27 A is not correct. 3°/o chose this .


• 28 Aspirin is an NSAID that carries increased r is k of cleft pa late in the newborn if administered early in pregnancy .
• 29 Neural tube defects are not a know n toxicity .
Nonsteroidal anti-inflammatory drug Aspirin Cleft lip and palate Neural tube Neural tube defect Toxicity Pregnancy Palate
o30
• 31 B is not correct. 6°/o chose this .
• 32 Meropenem is a broad-spectrum antibiotic belonging to the carbapenem subgroup . Neural tube defects are not a
known tox icity .
• 33
Broad-spectrum antibiotic Carbapenem Meropenem Antibiotics Neural tube Neural tube defect
o34
o35
C is not correct. 5°/o chose this.
Se leg iline selectively inhibits monoamine oxidase B, which thereby increases the availabi lity of dopamine. It is
• 36
used to treat Parkinson disease. Neural tube defects are not a known tox icity .
• 37 Selegiline Monoamine oxidase B Dopamine Monoamine oxidase Parkinson' s disease Monoamine neurotransmitter Neural tube defect Neural tube
• 38 ~

6
lock
s
Suspend
0
End Block
Item: 20 of 38 ~. I • M k <:] t> al ~· ~
QIO: 5279 .l. ar Previous Next lab 'lifllues Notes Calculator


18 Aspirin is an NSAID that carries increased r isk of cleft pa late in the newborn if administered ea rly in pregnancy.
19 Neu ral tube defects are not a known toxicity .
Nonsteroidal anti-inflammatory drug Aspirin Cleft lip and palate Neural tube Neural tube defect Toxicity Pregnancy Palate
20
• 21 B is not correct. 6°/o chose this .
• 22 Meropenem is a broad-spectrum antibiotic belonging to the carbapenem subgroup . Neu ral tube defects are not a
• 23 known toxicity .
Broad-spectrum antibiotic Carbapenem Meropenem Antibiotics Neural tube Neural tube defect
• 24
• 25 C is not correct. 5°/o chose this .
• 26 Se leg iline selectively inhibits monoamine oxidase 8, which thereby increases the availabi lity of dopamine. It is
used to treat Pa rkinson disease. Neural tube defects are not a known toxicity .
• 27 Selegiline Monoamine oxidase B Dopamine Monoamine oxidase Parkinson' s disease Monoamine neurotransmitter Neural tube defect Neural tube
• 28
D is not correct. 7°/o chose this .
• 29
Sumatr iptan is a seroton in agonist that causes vasoconstriction and is used to t reat acute mig raines and cluster
o30
headache attacks. Neu ral tube defects are not a known toxicity .
• 31 Cluster headache Sumatriptan Serotonin Vasoconstriction Serotonin receptor agonist Migraine Agonist Headache Neural tube defect Toxicity

• 32
• 33
Bottom Line:
o34
Spina bifida occu lta is a neural tube defect associated with several maternal facto rs, including folate deficiency,
o35
anticonvulsant med ications, and diabetes. Va lproic acid, an antiepileptic and mood stabil izer, can cause neu ral
• 36 tube defects in the fetus .
• 37 Neural tube defect Anticonvulsant Spina bifida Valproate Folate deficiency Folic acid Mood stabilizer Diabetes mellitus Neural tube Fetus

• 38 ~

6
lock
s
Suspend
0
End Block
Item: 20 of 3 8 ~ 1 • M k -<:J 1>- Jil ~· !:';-~
QIO: 52 79 ..L ar Pre v ious Next Lab fli!ltues Not es Calcula t o r
A A

18
FA17 p 461 .1
19
Neural tube defects europores fail to fuse (4th week) - persistent connection between amniotic cavity and spinal
20
canal. Associated \\ ith maternal diabetes as \\ell as low folic acid intake before concept ion and
• 21
during pregnancy. t a-fetoprotein (AFP) in amniotic Auid and maternal serum (except spina
• 22 bifida occulta =normal FP). t acetylcholinesterase (AChE) in amniotic Auid is a helpful
• 23 confirmatory test (fetal AChE in CSF AO\\~ through defect into amniotic Auid).
• 24 Anencephaly Failure of rostral neuropore to close - no forebrain , open cakarium. Clinical findings:
• 25 polyhydramnios (no swallowing center in brain).
• 26 Spina bifida occulta Failure of caudal neuropore to close, but no herniation. Usually seen at lower vertebral levels. Dura
• 27 is intact. Associated with tuft of hair or skin dimple at level of bony defect.
• 28 Meningocele \leninges (but no neural tissue) herniate through bony defect. Associated with spina bifida C)Stica.
• 29 Meningomyelocele l\lcninges and neural tissue (eg, cauda equina) herniate through bony defect.
o30 _,.. ~ +1- Tuft of hair / Skin defecVthinning / Skin thin or absent
Skin ~ ~ +1- Skin dimple
• 31
Subarachnoid
• 32 space

• 33 { / ; Dura
y-Leptomeninges
o34
o35 «~
• 36 Sptnal ~ \_Transverse
cord process
• 37
• 38

a
Lock
s
Suspend
8
End Bl ock
Item: 20 of 3 8 ~ 1 • M k -<:J 1>- Jil ~· !:';-~
QIO: 52 79 ..L ar Pre v ious Next Lab fli!ltues Not es Calcula t o r
A A

18
FA17 p514.1
19
Epilepsy drugs
20
GENERALIZED
• 21 ~
<
...... ~
• 22 "'
0
~
:z
0
~
...... ...:z...... ::::>
......
.....,"' .,., ;::::
~

• 23 ""
;:::: v =>fb
,_
""
~
:z
,_
0
<-
,_
"'
......
~

MECHANISM SIDE EFFECTS NOTES


• 24 ""
Ethosuximide • Blocks thalamic T-type Ca~· 1•:1 GIIIJ- Fthosuximide Sucks to have Silent
• 25 (absence) Seizures
~ channels causes Fatigue. C l distress.
• 26 lleadache, Itching (and
urticaria). and Stevens-
• 27 Johnson syndrome
• 28 Benzodiazepines •• t C,\ BA.\ action Sedation, tolerance, Also for eclampsia sci7llrcs (lsi
(eg, diazepam, ~ dependence, respiratory line is lgS04)
• 29
lorazepam, dcprc~~ion
o30 midazolam)
• 31 Phenobarbital ~ ~ t CAB!\ A action Sedation, tolerance, 1st line in neonates
dependence, induction
• 32 of cytochrome P--150,
• 33
o34 Phenytoin, ~ •
~
...
~
Blocks Na+ channels; 1cro-
order kinetics
cardiorespiratory depression
\leurologic: nystagmus, di plopia, ataxia, sedation, peripheral
neuropathy. Dermatologie: hirsutism, Stc1·cns-)ohnson
fosphenytoin
o35 S) ndrome, gingival hyperplasia, DRESS syndrome.

• 36
1\lusculoskeletal: osleopenia, SLE-Iike syndrome. llematologic:
megaloblastic anemia. Reproducti1·e: teratogenesis (fetal
• 37 h)danloin S) ndrome). Other: C)tochrome P-4;0 induction
• 38 Carbamazepine ~ Blocks a• channels Diplopia: ala\ia, bl~ 1st line for trigeminal neuralgia

a
Lock Suspend
s 8
End Bl ock
Item: 20 of 3 8 ~ 1 • M k -<:J 1>- Jil ~· !:';-~
QIO: 52 79 ..L ar Pre v ious Next Lab fli!ltues Not es Calcula t o r

18
19
Phenytoin, ./ •
./
...
./
Blocks 'a+ channels; 1ero-
order kinetics
p y p
\Jeurologic: nystagmus, di plopia, ataxia, sedation, peripheral
neuropathy. Dermatologic: hi rsutism, Stevens-Johnson
fosphenytoin
~) ndrome. gingi,':tl hyperplasia, DRESS syndrome.
20
1\lusculoskeletal: osteopenia, SLE-Iike syndrome. l lematologic:
• 21 megaloblastic anemia. Reproductive: teratogenesis (fetal
hydantoin S) ndrome). Other: cytochrome P-4;0 induction
• 22
Carbamazepine ./ Blocks a• channels Diplopia, ata\ia, blood 1st line for trigeminal neuralgm
• 23 ./ dy scr~ia~ (agranulocytosis,
• 24 apl;lstic anemia), )j, er
tol>icity. teratogenesis.
• 25 induction of C\ tochrome
• 26 P-450, Sl 0 1i, Ste' ens-
Johnson syndrome
• 27 •
Valproic acid ./ ./ t ~a+ channel inactivation, C l distress. rare but fatal Also used for myoclonic seizures,
• 28 ./ t CABA concentration hepatoto..:icity (measure bipolar disorder, migraine
by inhibiting C r\Bi\ LF'Ts), pancreatitis, neural prophyla..:is
• 29 transaminase tube defects, tremor, weight
o30 gain, contraindicated in
pregnancy
• 31
Vigabatrin t CABA by irreversibly Perma nent visual loss (black
• 32 inhibiting CAB box warning)
tmnsaminase
• 33
Gabapentin Primarily inhibi ts high-voltage- Sedation, ataxia Also used for peripheml
o3 4 acti\':tted Ca1• channels; neuropathy, postherpetic
o35 designed as CABA analog neuralgia
• 36 Topiramate ./ ./ Blocks 'a+ channels, t C \ 8 \ Sedntion, mental dulling, Also used for migraine
action kidne) stones, weight loss, pre,·ention
• 37 glaucoma
• 38

lamotrigine ./ ./ ./ Bl~ks ' o_ltage:g~tcd ! a+ Ste\ ens~Johnson ~) ·~dr?me

a
Lock
s
Suspend
8
End Bl ock
Item: 20 of 3 8 ~ 1 • M k -<:J 1>- Jil ~· !:';-~
QIO: 52 79 ..L ar Pre v ious Next Lab fli!ltues Not es Calcula t o r
A
induction of C) tochrome A

18
P-450, SIAOII, Stelens-
19 Johnson syndrome
20 Valproic acid ./ • ./ I Na+ channel inactivation, Cl distress. rare but fatal Also used for myoclonic seizures,
./ I CA BA conccntrntion hepatotoxicity (measure bipolar disord~r, migraine
• 21 by inhibiting CABA LFTs), pancreatitis, neural prophylaxis
• 22 trn nsa m inase tube defects, tremor, weight
gain, contraindicated in
• 23 pregnancy
• 24 Vigabatrin I C.\ BA b} irrcl'crsibl) Perm.ment 1 isualloss (black
• 25
inhibiting C.\ 8.\ bo' 11arning)
t rnnsa m inase
• 26 Gabapentin ./ Primaril} inhi~its high-,ol tage- Sedation. ata,ia Also used for periphernl
• 27 actil'ated Ca-• channels; neuropathy, postherpetic
designed as CABA analog neuralgia
• 28
Topiramate ./ ./ Blocks 'a+ channels, I C \ 1:3/\ Sedation, mental dulling. Also used for migraine
• 29 action kidney ~ tones, weight loss, prevention
glaucoma
o30
Lamotrigine ./ ./ ./ Blocks voltage-gated 1 a 1 Stevens-Johnson syndrome
•31 channels, inhibits the release (must be tit rn ted slowly)
• 32 of glutamate
Levetiracetam ./ ./ Unknown; may modulate l•:1t igue, drows iness,
• 33
CABA and glutamate release hcad<~che, neuropsychiatric
o34 svmptoms (eg, personality
changes)
o35
Tiagabine ./ I CABA by inhibit ing rcuptakc
• 36
= 1st line; = lst line for acute; ••• = lst line for prophylaxis.
• 37
• 38

a
Lock
s
Suspend
8
End Bl ock
Item: 21 of 3 8 ~ 1 • M k -<:J 1>- Jil ~· !:';-~
QIO: 2222 ..L ar Pre v ious Next Lab fli!ltues Not es Calcula t o r

IAA]
A A

18
A 45-year-old woman with a 15-year history of schizophren ia failed to respond to tria ls of risperidone and
19 aripiprazole. She has had six prior psychiatric admissions. She begins clozapine therapy to treat her
20 refractory schizophrenia . A routine complete blood count (CBC) is performed 6 months later. The patient's
• 21
WBC count is found to be <500/mm 3 and gram-positive cocci are isolated on blood culture. The patient is
subsequently started on antibiotic therapy .
• 22
• 23
In addition to cessation of the offend ing medication, enrollment in a clinical trial testing which of the following
• 24 experimental treatments wou ld be most appropriate for this patient's neutropenia?
• 25
• 26 A. Bone marrow transplant
• 27
B. C-kit ligand
• 28
• 29 C. Erythropoietin
o30 D. Granulocyte colony-stimulating factor
• 31
E. I nterleukin-11
• 32
F. Monocyte colony-stimulating fact or
• 33
o34
o35
• 36
• 37
• 38

a
Lock
s
Suspend
8
End Bl ock
Item: 21 of 38 ~. I • M k <:] t> al ~· ~
QIO: 2222 .l. ar Previous Next lab 'lifllues Notes Calculator


18 The correct answer is D. 75°/o chose this.
19 Clozapine, an atypical antipsychotic agent, is effective in patients with sch izophrenia who do not respond to
20 standard antipsychotic agents. Howeve r, ag ranulocytosis is a serious and potentially life-th reatening
complication seen in some patients receiving clozapine. Thus patients taking clozapine must undergo wee kly
21
WBC and abso lute neutrophi l counts for the fi rst 24 wee ks of t reatment. The correct t reatment strategy in
• 22 clozapine-induced neutropenia is to discontinue the clozapine and treat any associated infections.
• 23 Additiona lly, administration of granulocyte co lony-stimu lating factor (GCSF) can improve neutrophi l recovery .
• 24 GCSF acts by promoting the proliferation and matu ration of granu locytes. Alternatively, granulocyte-acrophage
co lony-stimu lating factor can be used, but it has more severe adverse eff ects, including capillary leak syndrome
• 25
and peripheral edema. Of note, G-CSF is not cu r rently approved for clozapine-associated neutropenia, though it
• 26 is a plausible treatment for this patient .
• 27 Atypical antipsychotic Neutrophil Granulocyte colony-stimulating factor Agranulocytosis Clozapine Neutropenia Antipsychotic Schizophrenia Granulocyte Edema

• 28 A is not correct. 5°/o chose this .


• 29 Neutropen ia caused by clozapine is both transient and reversible fol lowing withd rawal of the drug. A bone
o30 ma rrow t ransplant wou ld be too invasive at this point.
Neutropenia Hematopoietic stem cell transplantation Clozapine Bone marrow
• 31
• 32 B is not correct. 3°/o chose this .
• 33 C-kit ligand plays a role in plur ipotent stem ce ll development prio r to lineage differentiation .
ligand Stem cell ligand (biochemistry) Pluripotency Cell potency Pluripotent
o34
o35 C is not correct. 5°/o chose this.
• 36 Eryth ropoietin, a g lycoprotein growth facto r that stimu lates RBC production, is used to treat anem ia in renal
• 37 fai lure. It would not appropriate fo r the treatment of neutropenia .
Erythropoietin Neutropenia Glycoprotein Anemia Growth factor Red blood cell Kidney
• 38 ~

6
lock
s
Suspend
0
End Block
Item: 21 of 38 ~. I • M k <:] t> al ~· ~
QIO: 2222 .l. ar Previous Next lab 'lifllues Notes Calculator
- • '' - • - ,, • • • • - • • l':- :t'- - ••

18 Neutropenia Hematopoietic stem cell transplantation Clozapine Bone marrow


19
B is not correct. 3°/o chose this.
20
C-kit ligand plays a role in pluripotent stem ce ll development prior to lineage differentiation.
21 ligand Stem cell ligand (biochemistry) Pluripotency Cell potency Pluripotent
• 22
C is not correct. 5°/o chose this .
• 23
Eryth ropoietin, a g lycoprotein growth factor that stimu lates RBC production, is used to treat anem ia in rena l
• 24
fai lure. It would not appropriate fo r the treatment of neutropenia .
• 25 Erythropoietin Neutropenia Glycoprotein Anemia Growth factor Red blood cell Kidney

• 26
E is not correct. 4°/o chose this .
• 27
I nterleukin-11, a cytokine, is used to t reat thrombocytopenia .
• 28 Interleukin 11 Cytokine Thrombocytopenia

• 29
F is not correct. 8°/o chose this.
o30 Monocyte co lony-stimu lating factor promotes the prol iferation of monocytes. I t would not be appropriate for the
• 31 treatment of neutropen ia .
• 32 Monocyte Neutropenia Colony-stimulating factor

• 33
o34 Bottom Line:
o35
A serious adverse effect of the antipsychotic agent clozapine is agranu locytosis. GCSF can be beneficial in th is
• 36 setting by promoting proliferation and maturation of g ranulocytes .
• 37 Agranulocytosis Clozapine Antipsychotic Adverse effect Granulocyte Granulocyte colony-stimulating factor

• 38 ~

6
lock
s
Suspend
0
End Block
Item: 21 of 3 8 ~ 1 • M k -<:J 1>- Jil ~· !:';-~
QIO: 2222 ..L ar Pre v ious Next Lab fli!ltues Not es Calcula t o r
A
• • A

18
19 FA17 p 543.2
20
Atypical Aripiprazole, asenapine, clozapinc, olanzapinc, quctiapinc, iloperidone, paliperidonc,
21 antlpsychotics risperidone, lurasidone, ziprasidonc.
• 22 MECHANISM Not completely understood. \ lost arc D2
• 23 antagonists; aripiprazole is 0 2 partial agonist.
• 24 Varied effects on 5-I IT 2, dopamine, and
• 25 a- and H1-receptors.
• 26 CliNICAl USE Schizophrenia-both positi\'e and negative Use clozapine for treatment-resistant
• 27
symptoms. Also used for bipolar disorder, schizophrenia or schizoaffective disorder and
OCD, anxiety disorder, depression, mania, for suicidality in schizophrenia.
• 28
Tourette syndrome.
• 29
ADVERSE EFFECTS Al l-prolonged QT interval, fewer EPS and
o30 antichol inergic side effects than typical
• 31 antipsychotics.
• 32 "-pines"- metabolic syndrome (weight gain, Olanzapine .... O besity
• 33 diabetes, hyperlipidem ia).
C lozapine-agranulocytosis (monitor WBCs MusJ· watch bone marrow clo7cly with cloJ:apinc.
o3 4
frequently) and seizures (dose related).
o35
Risperidone-hyperprolactinemia (amenorrhea,
• 36 galactorrhea, gynecomastia).
• 37
• 38 FA17 P 117.2

a
Lock
s
Suspend
8
End Bl ock
Item: 21 of 3 8 ~ 1 • M k -<:J 1>- Jil ~· !:';-~
QIO: 2222

18
..L ar Pre v ious Next
' ;.
Lab fli!ltues
. ' . Not es
.
frequen tly) and seizures (dose related).
Calcula t o r
. . '

19 Risperidone-hyperprolactinemia (amenorrhea,
20 galactorrhea, gynecomastia).
21
• 22 FA17 p 117.2

• 23 Recombinant AGENT CliNICAl USES


• 24 cytokines and clinical AJdesleukin (IL-2) Renal cell carcinoma, metastatic melanoma
• 25
uses
Epoetin alfa (erythropoietin) Anemias (especially in renal failure)
• 26 Filgrastim (C-CSF) RecO\ CT} of bone marro\\' and WBC counts by
• 27 gra nulOC} te stimulation
• 28 Sargramostim (CM-CSF) Rcco,·ery of bone marrow and WBC counts by
• 29 granulocyte and monocyte stimulation
o30 IF -ex Chronic hepatitis B and C, Kaposi sarcoma,
• 31 malignant melanoma, hairy cell leukemia,
condyloma acu minata, renal cell carcinoma
• 32
lFN-P lulliple sclerosis
• 33
o34
IF -y Chronic granulomatous disease
o35 Romiplostim (thrombopoietin analog), Thrombocytopenia
eltrombopag (thrombopoietin receptor agonist)
• 36
• 37
Opreh-ekin (IL-1 1) Throm bocyt·openia
• 38

a
Lock
s
Suspend
8
End Bl ock
Item: 22 of 3 8 ~ 1 • M k -<:J 1>- Jil ~· !:';-~
QIO: 4198 ..L ar Pre v ious Next Labfli!llues Not es Calcula t o r

IAA]
A A

18
A 13-year-old girl with recurrent tonic-clonic seizures presents to her pediatrician compla ining of fatigue and
19 red swollen gums after her neurologist placed her on medication to control her seizures. Laboratory stud ies
20 show:
21
Hemoglobin: 9.0 g/ dl
• 22 Hematocrit: 26.0%
• 23 Mean corpuscular volume: 110/ mm 3
• 24 Mean corpuscular hemoglobin : 30 pg/ cell
• 25
• 26
The medication that caused her symptoms acts through which mechanism?
• 27 :

• 28 A . Blocks thalamic T-type calcium channels


• 29 B. Increases duration of chloride channel opening
o30
C. I ncreases frequency of chloride channel opening
• 31
D. Increases GABA ava ilability at t he postsynaptic junction
• 32
• 33 E. Increases sodium channe l inact ivation
o34
o35
• 36
• 37
• 38

a
Lock
s
Suspend
8
End Bl ock
Item: 22 of 38 ~. I • M k <:] t> al ~· ~
QIO: 4198 .l. ar Previous Next lab 'lifllues Notes Calculator


18
19 The correct answer is E. 63°/o chose this.
20 Phenytoin increases sodium channel inactivation, thereby decreasing the flux
of sodium ions across the neu rona l memb rane. It is an agent used effectively
21
in the treatment of partial and tonic-clonic seizures and status epilepticus . I n
22 fact, it is the t reatment of choice for initial therapy of tonic-clonic seizures.
• 23 Common adverse effects incl ude ging ival hyperplasia (as shown in the
• 24
image), Steven-Johnson syndrome, megaloblastic anemia, te ratogenesis,
and, very ra rely, drug-induced systemic lupus erythematosus. It also induces
• 25 the cytochrome P-450 system.
• 26 Megaloblastic anemia Systemic lupus erythematosus Status epilepticus Gingival enlargement Sodium channel

• 27 Teratology Cytochrome P450 Anemia Epileptic seizure Phenytoin Generalised tonic-clonic seizure Sodium Image courtesy of Wikimedia
• 28 Hyperplasia lupus erythematosus
Commons
• 29
A is not correct. 9°/o chose this.
o30
Ethosuximide blocks tha lamic T-type ca lcium channels and is the drug of choice for absence ( petit mal)
• 31 seizures . It is not used in the t reatment of tonic-clonic seizures .
• 32 Ethosuximide Absence seizure Calcium channel Thalamus T-type calcium channel Voltage-dependent calcium channel Seizure types Generalised tonic-clonic seizure

• 33 Epileptic seizure Calcium

o34 B is not correct. 8°/o chose this.


o35 Phenobar bita l is a ba rbiturate that faci litates y-aminobutyric acid action by increasing the duration of ch loride
• 36 channel opening . Dependence, respirato ry and ca rdiovascu lar depression, as wel l as induction of the
• 37
cytochrome P-450 system are its major adverse effects. I t is often used as f irst- line therapy in neonates and
second- line in status epilepticus but not typica lly used in adu lts .
• 38 ~ R::lrhitur~tP. PhP.noh~rhit~l St~tuc: P.niiP.ntiruc: C'hloric1P. rh::lnnP.I C'vtorhromP. P4SO M~ior c1P.nrP.c:c:ivP. c1ic:orc1P.r Tnf~nt C'irrul;:ltorv c:vc:tP.m OP.nrP.c:c:ion (moocH C'hloric1P.

6
lock
s
Suspend
0
End Block
Item: 22 of 38 ~. I • M k <:] t> al ~· ~
QIO: 4198 .l. ar Previous Next Lab 'lifllues Notes Calculator

• B is not correct. 8°/o chose this .


18
19 Phenobar bita l is a ba rbiturate that faci litates y-aminobutyric acid action by increasing the duration of ch loride
channel opening . Dependence, respirato ry and ca rdiovascu lar depression, as wel l as induction of the
20
cytochrome P-450 system are its major adverse eff ects. It is often used as f irst- line therapy in neonates and
21 second- line in status epilepticus but not typica lly used in adu lts.
22 Barbiturate Phenobarbital Status epilepticus Chloride channel Cytochrome P450 Major depressive disorder Infant Circulatory system Depression (mood) Chloride

• 23 C is not correct. 8°/o chose this.


• 24 Lorazepam is a benzod iazepine used as a fi rst-l ine treatment for status epi lepticus. It acts by facilitating y-
• 25 aminobutyric acid action by increasing the frequency of chlor ide channel opening . Dependence and centra l
• 26
ne rvous system depression are its majo r adverse effects .
Benzodiazepine lorazepam Central nervous system Status epilepticus Chloride channel Central nervous system depression Nervous system First-line treatment
• 27
Chloride Depression (mood) Therapy
• 28
• 29 D is not correct. 12% chose this .
o30
Valp roic acid, although used as treatment for complex-partial and absence seizu res, does not cause gingiva l
hyperp lasia. Its mechan ism of increased GABA availability does lead to other side effects, including,
• 31 th rombocytopen ia, swel ling of the hands/ feet and alopecia .
• 32 Valproate Thrombocytopenia Gamma-Aminobutyric acid Gingival enlargement Epileptic seizure Absence seizure Hair loss Hyperplasia Adverse drug reaction

• 33 Side effect

o34
o35
Bottom Line:
• 36
Phenytoin is a fi rst-l ine seizure med ication that is associated with adverse effects that include ging iva l
• 37
hyperp lasia, megaloblastic anemia, and induction of the cytochrome P-450 system .
• 38 ~ Megaloblastic anemia Phenytoin Gingival enlargement Anemia Cytochrome P450 Hyperplasia Epileptic seizure

6
lock
s
Suspend
0
End Block
Item: 22 of 3 8 ~ 1 • M k -<:J 1>- Jil ~· !:';-~
QIO: 4198 ..L ar Pre v ious Next Labfli!llues Not es Calcula t o r
A
FIRST AID FAC T S
. . -
A

18
19
FA17p487.1
20
Seizures Characterized by synchronized, high-frequency neuronal firing. Variety of forms.
21
Partial (focal) seizures Affect single area of the brain. lost commonly Epilepsy- a disorder of recurrent seizures
22
originate in medial temporal lobe. Often (febrile seizures are not epilepsy).
• 23
preceded by seizure aura; can secondari l~ Status epilepticus-continuous (> 5-30 min)
• 24 or recurring seizures that may result in brain
generalize. Types:
• 25 Simple partial (consciousness intact)- 111Jury.
• 26 motor, sensor), autonomic, psychic Causes of seizures by age:
• 27 • Complex partial {impaired consciousness) • Children-genetic. infection (febrile),
Generalized seizures Diffuse. Types: trauma, congenital, metabolic
• 28
Absence (petit mal)-3 Hz spike-and-wa\'e Adults-tumor, trauma, stroke, infection
• 29
discharges, no postictal con fusion, blank Elderly-stroke, tumor, trauma, metabolic,
o30
stare infection
• 31 Myoclonic- quick, repetitive jerks
• 32 • Tonic-clonic (grand ma l)-a ltcrnating
• 33 stiffening and movement
Tonic-stiffening
o3 4
• Atonic- "drop" seizures (falls to floor};
o35
commonly mistaken for fainting
• 36
• 37
FA 17 p 514.1
• 38

a
Lock Suspend
s 8
End Bl ock
Item: 22 of 3 8 ~ 1 • M k -<:J 1>- Jil ~· !:';-~
QIO: 4198 ..L ar Pre v ious Next Labfli!llues Not es Calcula t o r
A A

18
19 FA17 p514.1
20 Epilepsy drugs
21 GENERALIZED
~

22 .....
< ~

• 23
C>
~
:z
C>
~
...... ........ "'
.....
:::>

..."':z... .,., ~
~
< v :::>:'1:;
~
:z ,_ ~

• 24 ""
....
< .....
C>
<
.........
<-
"' ... MECHANISM SIDE EFFECTS NOTES
• 25 Ethosuximide • Blocks thalamic T-t~-pe Ca~• Jo:l GI IIJ- Fthosm:imide Sucks to haYe Silent
.I channels causes Fatigue. C l distress. (absence) Seizures
• 26
lleadache, Itching (and
• 27 urticaria), and Stevens-
Johnson syndrome
• 28
Benzodiazepines •• t CJ\BA" action Sedation, tolerance, Also for eclampsia seizures (1st
• 29 (eg, diazepam, .I dependence, respiratory line is MgS04)
o30 lorazepam, dcprc~~ion
midazolam)
• 31
Phenobarbital .I .I t CABA,\action Sedation, tolerance, Ist line in neonates
• 32 dependence. induction
of cytochrome P--!50,
• 33
o34 Phenytoin,
fosphenytoin
.I •
.I
...
.I
Blocks Na+ channels; 1ero-
order kinetics
cardiorespiratory depression
\Jcurologic: nystagmus, diplopia, ataxia, sedation, peripheral
neuropathy. Dermatologic: hirsutism, Stevens-Johnson
o35
5) ndrome, gingival hyperplasia, DRESS syndrome.
• 36 1\lusculoskeletal: osteopenia, SLE-Iike syndrome. llematologic:
• 37 megaloblastic anemia. Reproductive: teratogenesis (fetal
hydantoin 5) ndrome). Other: cytochrome P-450 induction
• 38
• r ...e...~-~ ..... -•-oft • ./ Q),..,.l,,. I\... ~ + ,.),..,.,,.,.nlr f"' in1ol"\ni..,. ..,. ..." ;,.. hl""',l l c-l l: ... .a . " .. .-..: ....oo....... : ... ., J ..... .n ........ J... : ...

a
Lock Suspend
s 8
End Bl ock
Item: 22 of 3 8 ~ 1 • M k -<:J 1>- Jil ~· !:';-~
QIO: 4198 ..L ar Pre v ious Next Labfli!llues Not es Calcula t o r

18
A

Carbamazepine • ./ Blocks Na+ channels Diplopia, atm..ia, blood 1st line for trigeminal neuralgia
A

./ d) scra~ia~ (agranulocytosis,
19
aplastic anemia), liver
20 toxicit). teratogenesis.
induction of C) tochrome
21 P-·-150, SIADII, Ste1 ens-
22 johnson s1 ndrome
• 23 Valproic acid ./ • ./ I '\a+channel inactil·ation, C l distress. rare but futal Also used for myoclonic seizures.
./ I CA BA concentration hepatotoxicity (measure bipolar disorder, migraine
• 24 by inhibiting CABA LFTs), pancreatitis, neural prophylaxis
transaminase tube defects, tremor, weight
• 25
gain, contraindicated in
• 26 pregnancy
• 27 Vigabatrin t CABA by irrcversibl) Perm.111cnt visual loss (black
inhibiting C.\B.\ bo' liMning)
• 28 transaminase
• 29 Gabapentin ./ Primaril) inhibits high-voltage- Sedation, ata,ia Also used for peripheral
activated Ca 2+channels; neuropathy, postherpetic
o30
designed as CABA analog neuralgia
• 31 ./ ./ Blocks a+ channels, I C BA Sedation, mental dulling, Also used for migraine
Topiramate
• 32 action kidney stones, weight loss, prevention
glaucoma
• 33
Lamotrigine ./ ./ ./ Blocks voltage-gated a' Stevens-Johnson syndrome
o34 channels, inhibits the release (must be til ra ted slowly)
of glutamate
o35
Levetiracetam ./ ./ Unknown; may modulate l•:1t igue, dro11siness .
• 36 CABA and glutamate release headache, neuropsrchiatric
• 37 S\ mptoms {eg, personality
changes)
• 38
• "9"'1 - - - L I - -
, • ,....-. n--. 1 • • •1 • •• • I

a
Lock
s
Suspend
8
End Bl ock
Item: 22 of 3 8 ~ 1 • M k -<:J 1>- Jil ~· !:';-~
QIO: 4198 ..L ar Pre v ious Next Labfli!llues Not es Calcula t o r
A A

18
19 FA17 p241.2
20 Drug reactions- musculoskeletal/skin/connective tissue
21 DRUG REACTION CAUSAL AGENTS NOTES

22 Fat redistribution P rotease inhibitors, G lucocorticoids Fat PiG


• 23 Gingival hyperplasia Phenytoin, Ca 2.. channel blockers, cyclosporine
• 24 Hyperuricemia (gout) Pvrazinamide, T hia1ides, Furosemide, "\ iacin, Painful 'lophi and Feet eed C are
'
• 25 C~closporine

• 26 Myopathy Statins, fibrates, niacin, colchicine, daptomycin,


• 27
hydroxychloroquine, interferon-a,
penicillamine, glucocorticoids
• 28
Osteoporosis Corticosteroids, depot medroxyprogesterone
• 29
acetate, Gn RH agonists, aromalase inhibitors,
o30
anticonvulsants, heparin
• 31
Photosensitivity Sulfonamidcs, Amiodarone, 'lclracyclines, 5-F U SAT For Photo
• 32
Rash (Stevens- Anti-epileptic drugs (especially lamolrigine), Steven Johmon has epileptic allergy to mlfa
• 33 Johnson syndrome) allopurinol, sulfa drugs, penicillin drugs and penicillin
o34
SLE-Iike syndrome Sulfa drugs, Hyd ralazine, Isoniazid, I laving lupus is "SI IIPP-E "
o35 P rocainamide, Phenytoin, Elanereepl
• 36 Teeth discoloration Tctracycl ines Tcethracycl ines
• 37 Tendonitis, tendon Pluoroquinolones
• 38 rupture, and

a
Lock
s
Suspend
8
End Bl ock
Item: 23 of 3 8 ~ 1 • M k -<:J 1>- Jil ~· !:';-~
QIO: 5 0 77 ..L ar Pre v ious Next Lab fli!ltues Notes Calcula t o r

IAA]
A A

18
A previously healthy 19-year-old man recently diagnosed with a seizure disorder is started on a new
19 medication for his seizures . A few weeks later, he presents to the emergency department with fevers up to
20 40.0°C (104°F), productive cough, and pleuritic chest pain. X-ray of his chest is shown below. Complete
blood cel l count shows:
21
22 WBCs: 3000/ mm3
• 23 Neutrophi ls: 1%
• 24 Absolute neutrophil count: 30
Lymphocytes: 70%
• 25
Monocytes: 8%
• 26 Eosinophi ls: 0%
• 27 Basophi ls: 0%
• 28
• 29
o30
• 31
• 32
• 33
• 34
o35
• 36
• 37
• 38

a
Lock
s
Suspend
8
End Bl ock
Item: 23 of 3 8 ~ 1 • M k -<:J 1>- Jil ~· !:';-~
QIO: 5 0 77 ..L ar Pre v ious Next Lab fli!ltues Notes Calcula t o r
A A

18 Abso lute neutroph il count : 30


19 Lymphocytes: 70%
Monocytes: 8%
20
Eosinoph ils: 0%
21 Basoph ils: 0%
22
• 23
• 24
• 25

• 26
• 27
• 28
• 29

o30
• 31
• 32

• 33
• 34

o35
• 36
• 37
• 38 _ What is the mechanism of action of the seizure medication that the patient is most likely taking?

a
Lock
s
Suspend
8
End Bl ock
18
19
20
21
22
• 23
• 24
• 25
• 26
• 27
• 28
• 29 What is the mechan ism of action of the seizu re med ication that the patient is most likely taking?
o30
:
• 31 A . Blocks tha lam ic T-type calcium channels
• 32
B. Decreases GABA action
• 33
o34 C. Inactivates sod ium channels
o35 D . Increases frequency of ch loride channel open ing
• 36
E. Prolongs duration of chloride channel opening
• 37
• 38

a
Lock
s
Suspend
8
End Block
Item: 23 of 38 ~. I • M k <:] t> al ~· ~
QIO: 5077 .l. ar Previous Next lab 'lifllues Notes Calculator


18
The correct answer is C. 59°/o chose this.
19
The patient's X-ray is consistent with right middle lobe pneumonia, which the patient likely developed due to his
20 decreased WBC count. He has an extremely low absolute neutrophil count, demonstrating severe neutropenia
21 (severe neutropenia is defined as <500/mm 3 ) . Furthermore, he also has no eosinoph ils or basoph ils. Th is
patient has agranu locytosis, likely an adverse effect of his new seizure medication. Carbamazepine is like ly to
22
be the drug he is ta king. Carbamazepine is an antiepileptic drug that inactivates sod ium channels and inhibits
23 N-methyl-0-aspartate receptors . Its adverse effects include liver toxicity, diplopia, ataxia, teratogenesis, and
• 24 blood dyscrasias such as ag ranulocytosis and aplastic anemia. Complete blood ce ll count and liver function tests
• 25 must be closely monitored in patients taking carbamazepine .
Carbamazepine Neutrophil Agranulocytosis Diplopia Aplastic anemia Neutropenia liver function tests Complete blood count Pneumonia Anemia Ataxia Teratology
• 26
Absolute neutrophil count Anticonvulsant Hepatotoxicity Eosinophil liver Adverse effect Sodium Sodium channel X-ray Epileptic seizure Toxicity
• 27
Pharmaceutical drug
• 28
• 29 A is not correct. 11% chose this .
o30 Ethosuximide is an antiepi leptic drug that blocks T-type calcium channe ls. Its adverse effects include Stevens-
Johnson syndrome, gastrointestinal distress, headache, lethargy, and urticaria. I t typica lly does not cause
• 31
agranulocytosis .
• 32 Ethosuximide Stevens-Johnson syndrome Agranulocytosis Urticaria Anticonvulsant Headache lethargy Calcium channel Calcium T -type calcium channel

• 33 Voltage-dependent calcium channel Human gastrointestinal tract

o3 4
B is not correct. 9°/o chose this.
o35
Carbamazepine wor ks by increasing sod ium channel inactivation, it does not decrease GABA activity .
• 36 Topi ramate is an antiepileptic drug increases sodium channe l inactivation and increases GABA. Topiramate's
• 37 adverse effects include kidney stones, weight loss, mental dul ling, and sedation . It typical ly does not cause
• 38 agranulocytosis .
~

6
lock
s
Suspend
0
End Block
Item: 23 of 38 ~. I • M k <:] t> al ~· ~
QIO: 5077 .l. ar Previous Next lab 'lifllues Notes Calculator

18
Dis not correct. 11% chose this.
19
Drugs that increase the frequency of chlor ide channel opening to stimu late an increase in GABA activity include
20 benzodiazepines such as lorazepam . It is used as first-l ine treatment for status epilepticus. Due to the increased
21 r isk of tolerance to and dependence on the drug with frequent use, it is not typical ly prescribed as a da ily
medication for seizure prevention, so it is unl ikely that the patient was prescribed it for dai ly use to prevent his
22
seizures . Adverse effects of benzodiazepines include sedation, tolerance, and dependence. I t typically does not
23 cause ag ranulocytosis.
• 24 lorazepam Agranulocytosis Gamma-Aminobutyric acid Status epilepticus Chloride channel Benzodiazepine Sedation Epileptic seizure Chloride First-line treatment

• 25 Pharmaceutical drug Therapy

• 26
E is not correct. 10% chose this .
• 27
Barbitu rates prolong the duration of ch loride channe l opening in GABAA receptors. In ra re cases, Stevens-
• 28 Johnson syndrome is associated with barbiturate use. However, this class of dr ugs is not common ly associated
• 29 with agranulocytosis .
Barbiturate Stevens-Johnson syndrome Chloride channel Agranulocytosis Gamma-Aminobutyric acid Receptor (biochemistry) Chloride
o30
• 31
• 32 Bottom Line:
• 33 Among carbamazepine's most ser ious adverse effects are blood dyscrasias such as ag ranulocytosis or aplastic
o34 anemia. Other adverse effects include liver toxicity, diplopia, atax ia, and teratogenesis. Complete blood cel l
count and liver function tests shou ld be closely monitored in patients taking carbamazepine to avoid serious
o35
adverse effects .
• 36 Carbamazepine Agranulocytosis Diplopia Aplastic anemia liver function tests Complete blood count Anemia Ataxia Teratology Hepatotoxicity liver Dyscrasia
• 37 Toxicity

• 38

6
lock
s
Suspend
0
End Block
Item: 23 of 3 8 ~ 1 • M k -<:J 1>- Jil ~· !:';-~
QIO: 5 0 77

18
A
..L
. . . ar Pre v ious Next Lab fli!ltues Notes Calcula t o r
A

19 FA17 p514.1
20 Epilepsy drugs
21 GENERALIZED

22
23
• 24
SIDE EFFECTS NOTES
• 25
Ethosuximide • Blocks thalamic T-type Ca~• El GI IIJ- Fthosm:imide Sucks to have Silent
• 26 channels causes Fatigue. C l distress. (absence) Seizures
lleadache, Itching (and
• 27 urticaria). and Stevens-
• 28 Johnson syndrome
Benzodiazepines •• t CABA.\ action Sedation, tolerance, Also for eclampsia sci1urcs (1st
• 29
(eg, diazepam, dcpcnclcnce, respiratory line is MgS04)
o30 lorazepam, dcprc~~ion
midazolam)
• 31
Phenobarbital ./ t CABAA action Sedation, tolerance, Ist line in neonates
• 32 dependence. induction
• 33 of cytochrome P--!50,
o34
Phenytoin, ./ • ... Blocks Na+ channels; 7cro-
cardiorespiratory depression
\leurologic: nystagmus, diplopia, ataxia, sedation, peripheral
o3 5 fosphenytoin order kinetics neuropathy. Oermatologic: hirsutism, Stevens-johnson
S) ndrome, gingival hyperplasia, DRESS syndrome.
• 36
l\lmculoskelelal: osleopenia, SLE-Iike syndrome. llemalologic:
• 37 megaloblastic anemia. Reproductive: teratogenesis (fetal
• 38 ,
h)danloin S) ndrome). Other: C) tochrome P-4;0 induction
..... . ' . ' • . •· r . • ..
a
Lock
s
Suspend
8
End Bl ock
Item: 23 of 3 8 ~ 1 • M k -<:J 1>- Jil ~· !:';-~
QIO: 5 0 77 ..L ar Pre v ious Next Lab fli!ltues Notes Calcula t o r

18
19
Phenytoin, ./ •
./
...
./
Blocks 'a+ channels; 1ero-
order kinetics
p y p
\Jeurologic: nystagmus, di plopia, ataxia, sedation, peripheral
neuropathy. Dermatologic: hi rsutism, Stevens-Johnson
fosphenytoin
~) ndrome. gingi,':tl hyperplasia, DRESS syndrome.
20
1\lusculoskeletal: osteopenia, SLE-Iike syndrome. l lematologic:
21 megaloblastic anemia. Reproductive: teratogenesis (fetal
hydantoin S) ndrome). Other: cytochrome P-4;0 induction
22
Carbamazepine ./ Blocks a• channels Diplopia, ata\ia, blood 1st line for trigeminal neuralgm
23 ./ dy scr~ia~ (agranulocytosis,
• 24 apl;lstic anemia), )j, er
tol>icity. teratogenesis.
• 25 induction of C\ tochrome
• 26 P-450, Sl 0 1i, Ste' ens-
Johnson syndrome
• 27 •
Valproic acid ./ ./ t ~a+ channel inactivation, C l distress. rare but fatal Also used for myoclonic seizures,
• 28 ./ t CABA concentration hepatoto..:icity (measure bipolar disorder, migraine
by inhibiting Cr\Bi\ LF'Ts), pancreatitis, neural prophyla..:is
• 29 transaminase tube defects, tremor, weight
o30 gain, contraindicated in
pregnancy
• 31
Vigabatrin t CABA by irreversibly Permanent visual loss (black
• 32 inhibiting CAB box warning)
tmnsaminase
• 33
Gabapentin Primarily inhibi ts high-voltage- Sedation, ataxia Also used for peripheml
o3 4 acti\':tted Ca1• channels; neuropathy, postherpetic
o35 designed as CABA analog neuralgia
• 36 Topiramate ./ ./ Blocks 'a+ channels, t C \ 8 \ Sedntion, mental dulling, Also used for migraine
action kidne) stones, weight loss, pre,·ention
• 37 glaucoma
• 38

lamotrigine ./ ./ ./ Bl~ks ' o_ltage:g~tcd ! a+ Ste\ ens~Johnson ~) ·~dr?me

a
Lock
s
Suspend
8
End Bl ock
Item: 23 of 3 8 ~ 1 • M k -<:J 1>- Jil ~· !:';-~
QIO: 5 0 77 ..L ar Pre v ious Next Lab fli!ltues Notes Calcula t o r
A
induction of C) tochrome A

18
P-450, SIAOII, Stelens-
19 Johnson syndrome
20 Valproic acid ./ • ./ I Na+ channel inactivation, Cl distress. rare but fatal Also used for myoclonic seizures,
./ I CA BA concentrntion hepatotoxicity (measure bipolar disord~r, migraine
21 by inhibiting CABA LFTs), pancreatitis, neural prophylaxis
22 trn nsa m inase tube defects, tremor, weight
gain, contraindicated in
23 pregnancy
• 24 Vigabatrin I C.\ BA b} irrel'ersibl) Perm.ment 1 isualloss (black
• 25
inhibiting C.\ 8.\ bo' 11arning)
t rnnsa m inase
• 26 Gabapentin ./ Primaril} inhi~its high-,ol tage- Sedation. ata,ia Also used for periphernl
• 27 actil'ated Ca-• channels; neuropathy, postherpetic
designed as CABA analog neuralgia
• 28
Topiramate ./ ./ Blocks 'a+ channels, I C \ 1:3/\ Sedation, mental dulling. Also used for migraine
• 29 action kidney ~ tones, weight loss, prevention
glaucoma
o30
Lamotrigine ./ ./ ./ Blocks voltage-gated 1 a 1 Stevens-Johnson syndrome
•31 channels, inhibits the release (must be tit rn ted slowly)
• 32 of glutamate
Levetiracetam ./ ./ Unknown; may modulate I•:1t igue, drows iness,
• 33
CABA and glutamate release hcad<~che, neuropsychiatric
o34 svmptoms (eg, personality
changes)
o35
Tiagabine ./ I CABA by inhibit ing reuptakc
• 36
= 1st line; = lst line for acute; ••• = lst line for prophylaxis.
• 37
• 38

a
Lock
s
Suspend
8
End Bl ock
Item: 24 of 3 8 ~ 1 • M k -<:J 1>- Jil ~· !:';-~
QIO: 3626 ..L ar Pre v ious Next Lab fli!ltues Notes Calcula t o r

IAA]
A A

18
A medical student is completing his fami ly medicine rotation in a rura l farming commun ity when a young
19 woman comes into the office. Since yesterday she has been experiencing diarrhea, weakness, and watery
20 eyes. She says that she works in the fields every day. She recently lost the gloves she uses to grab the
produce and has been using her hands instead.
21
22
Wh ich of the following is the best therapy for th is patient?
23
• 24 4 2 8835 :
A. Atropine
• 25
• 26 B. Bethanechol
• 27 C. Botulism bivalent antitoxin
• 28
D. Duloxetine
• 29
o30
E. Pyridostigmine
• 31
• 32
• 33
o34
o35
• 36
• 37
• 38

a
Lock
s
Suspend
8
End Bl ock
Item: 24 of 3 8 ~ 1 • M k -<:J 1>- Jil ~· !:';-~
QIO: 3626 ..L ar Pre v ious Next Lab fli!ltues Notes Calcula t o r

IAA]
A A

18
A medical student is completing his fami ly medicine rotation in a rura l farming commun ity when a young
19 woman comes into the office. Since yesterday she has been experiencing diarrhea, weakness, and watery
20 eyes. She says that she works in the fields every day. She recently lost the gloves she uses to grab the
produce and has been using her hands instead.
21
22
Wh ich of the following is the best therapy for th is patient?
23
• 24
A. Atropine
• 25
• 26 B. Bethanechol
• 27 C. Botulism bivalent antitoxin
• 28
D. Duloxetine
• 29
o30
E. Pyridostigmine
• 31
• 32
• 33
o34
o35
• 36
• 37
• 38

a
Lock
s
Suspend
8
End Bl ock
Item: 24 of 38 ~. I • M k <:] t> al ~· ~
QIO: 3626 .l. ar Previous Next lab 'lifllues Notes Calculator


18
19 The correct answer is A. 67°/o chose this.
20 The patient is exhibiting symptoms of poisoning by an organophosphate-based pesticide, which acts as a
21 chol inesterase inhibitor resulting in increased levels of acetylchol ine. Symptoms of excess cholinergic activity
can be remembered by the mnemonic SLUDge: Salivation, Lacrimation, Urination, and Defecation. Atropine, a
22
muscarinic receptor antagonist, prevents the binding of acetylchol ine to its receptor, thereby antagonizing the
23 effects of the acetylchol ine and al lowing the regeneration of active chol inesterase. Although atropine reverses
24 the toxic effects of cholinerg ic excess that organophosphates cause at sweat glands and parasympathetic end-
• 25
organ receptors, it does not alter the nicotinic effects of organophosphates at ske letal muscles (includ ing
muscles of respiration). For that reason, it is recommended that pral idoxime be given in addition to atropine to
• 26 reverse the nicotinic effects .
• 27 Acetylcholinesterase inhibitor Atropine Muscarinic acetylcholine receptor Acetylcholine Pesticide Pralidoxime Cholinergic Parasympathetic nervous system

• 28 Organophosphate Cholinesterase Muscarinic antagonist Skeletal muscle Sweat gland Mnemonic Receptor antagonist Receptor (biochemistry) Enzyme inhibitor

• 29 Antagonist Nicotinic acetylcholine receptor

o30
B is not correct. 7°/o chose this .
• 31
Bethanechol is a direct chol inergic agon ist used to treat postoperative and neurogenic ileus and urinary
• 32 retention. I t is not indicated in the t reatment of cho linesterase inhibitor poisoning and wou ld in fact worsen her
• 33 symptoms .
Acetylcholinesterase inhibitor Urinary retention Cholinergic Ileus Cholinesterase Agonist Bethanechol Nervous system Enzyme inhibitor
o34
o35 C is not correct. 5°/o chose this.
• 36 The patient is not exh ibiting symptoms of f laccid paralysis, which would be associated with botu linum toxin
• 37
exposure . The bivalent antitoxin is the antidote to botul inum toxin-induced para lysis .
Botulinum toxin Flaccid paralysis Antitoxin Paralysis Toxin Antidote
• 38 ~

6
lock
s
Suspend
0
End Block
Item: 24 of 38 ~. I • M k <:] t> al ~· ~
QIO: 3626 .l. ar Previous Next lab 'lifllues Notes Calculator

• Acetylcholinesterase inhibitor Urinary retention Cholinergic Ileus Cholinesterase Agonist Bethanechol Nervous system Enzyme inhibitor
18
19 C is not correct. 5°/o chose this.
20 The patient is not exh ibiting symptoms of f laccid paralysis, which would be associated with botu linum toxin
exposure . The biva lent antitoxin is the antidote to botul inum toxin-induced para lysis.
21
Botulinum toxin Flaccid paralysis Antitoxin Paralysis Toxin Antidote
22
23
D is not correct. 4°/o chose this.
24
Duloxetine inhibits norepinephrine and serotonin reuptake. It is used to treat major depressive disorder,
general ized anxiety disorder, neuropathic pain, and pain related to f ibromyalg ia. It has no use in the treatment
• 25 of chol inergic toxicity .
• 26 Major depressive disorder Fibromyalgia Duloxetine Generalized anxiety disorder Serotonin Norepinephrine Neuropathic pain Cholinergic Anxiety disorder Reuptake

• 27 Anxiety Toxicity Peripheral neuropathy

• 28
E is not correct. 17% chose this .
• 29
Pyridostigmine is a reversible acetylchol inesterase inhibitor used to treat myasthenia gravis. Administration of
o30 pyr idostigmine wou ld worsen this patient's cholinerg ic symptoms.
Acetylcholinesterase inhibitor Myasthenia gravis Acetylcholinesterase Cholinergic Pyridostigmine Enzyme inhibitor
• 31
• 32
• 33 Bottom Line:
o34
Organophosphate posion ing man ifests as excess chol inergic activity (SLUDge : Sal ivation, Lacr imation,
o35 Ur ination, and Defecation) . Atropine, wh ich acts as a competit ive inhibitor of ACh at muscarinic receptors, is
• 36 the t reatment of choice. Pra lidoxime is recommended as an adjunct therapy .
Atropine Organophosphate Cholinergic Pralidoxime Muscarinic acetylcholine receptor Competitive inhibition Receptor (biochemistry) Enzyme inhibitor
• 37
• 38 ~

6
lock
s
Suspend
0
End Block
Item: 24 of 38 ~. I • M k <:] t> al ~· ~
QIO: 3626 .l. ar Previous Next lab 'lifllues Notes Calculator

• •
18
FA17 p 232.2
19
20 Cholinesterase Often due to organophosphates, such as DUMBBF:LSS.
inhibitor poisoning parathion, that irreversibly inhibit AChE. Organophosphates are often components of
21
Causes Diarrhea, Urination, Miosis, insecticides; poisoning usually seen in farmers.
22
Bronchospasm, Bradycardia, Excitation Antidote- atropine (competitive inhibitor) +
23 of skeletal muscle and C1 S, Lacrimation, pralidoxime (regenerates AChE if given early).
24 Sweating, and Sali\·ation. May lead to
• 25 respiratory failure if untreated.
• 26
• 27 FA17 p 233.2

• 28 Atropine Muscarinic antagonist. Used to treat bradycardia and for ophthalmic applications.
• 29 ORGAN SYSTEM ACTION NOTES

o30 Eye t pupil dilation, cycloplegia Blocks DUVfBBeLSS in cholinesterase


• 31 Airway ~ secretions inhibitor poisoning. Docs not block excitation
of skeletal muscle and C S (mediated by
• 32 Stomach ~ acid secretion
nicotinic receptors).
• 33 Gut ~ motility
o3 4 Bladder ~ urgency in cystitis
o35 ADVERSE EFFECTS t body temperature (due to I sweating); Side effects:
• 36 rapid pulse; dry mouth; dry, Amhed skin; Hot as a hare
• 37 cycloplegia; constipation; disorientation Dry as a bone
• 38 Can cause acute angle-closure glaucoma in Red as a beet •

6
lock
s
Suspend
0
End Block
Item: 24 of 3 8 ~ 1 • M k -<:J 1>- Jil ~· !:';-~
QIO: 3626 ..L ar Pre v ious Next Lab fli!ltues Notes Calcula t o r

18
A
Bronchospasm, Bradrcardia, Excitation Antidote-atropine (competitive inhibitor) + A

of skeletal muscle and CNS, Lacrimal ion, pralidoxime (regenerates AChE if given early).
19
Sweating, and Sali,·ation. ~1ay lead to
20
respiratory failure if untreated.
21
22
FA17 p 233.2
23 Atropine ~Iuscarinic antagonist. Used to treat bradycardia and for ophthalmic applications.
24
ORGAN SYS !EM ACTION NOTES
• 25
Eye f pupil dilation, cycloplegia Blocks Dll\tBBeLSS in cholinesterase
• 26 inhibitor poisoning. Docs not block excitation
Airway l secretions
• 27 of skeletal muscle and C 'S (mediated bv
Stomach l acid secretion '
• 28 nicotinic receptors).
Gut l motility
• 29
Bladder l urgency in cystitis
o30
• 31
ADVERSE EFFECTS t body temperature (due to 1 sweating); Side effects:
rapid pulse; dry mouth; dry, Amhcd ~ kin ; I lot as a hare
• 32
cycloplegia; constipation; d i\oricnt at ion O r) as a bone
• 33 Can cause acute angle-closure glaucoma in Red as a beet
o3 4 elderly (due to mydriasis), urinary retention Blind as a bat
o3 5 in men with prostatic hyperplasia, and i\Jad as a hatter
• 36 hyperthermia in infants. Jimsonweed (Datura) .... gardener's pupil
(mydriasis due to plant al kaloids)
• 37
• 38

a
Lock
s
Suspend
8
End Bl ock
Item: 25 of 3 8 ~ 1 • M k -<:J 1>- Jil ~· !:';-~
QIO: 5776 ..L ar Pre v ious Next Lab fli!ltues Notes Calcula t o r

IAA]
A A

18
A 72-year-old man with a history of hypertension and hypercholesterolemia presents to the emergency
19 department with weakness of his left leg and arm that began 5 hours ago. His temperature is 37.0°C
20 (98.6°F), pulse is 90/min, respiratory rate is 18/ min, and blood pressure is 180/85 mm Hg. Results of
laboratory tests are within normal limits. Noncontrast CT of the head is shown .
21
22
23
24
• 25
• 26
• 27
• 28
• 29
o30
• 31
• 32
• 33
o34
o35
• 36 I mage courtesy of Lucien Monfils
• 37
• 38 Wh ich of the following medications is the best initial therapy for th is patient?

a
Lock
s
Suspend
8
End Bl ock
18
19
20
21
22
23
24
• 25
• 26
• 27
• 28
• 29 Which of the fol lowing medications is the best init ial therapy for this patient?
o30
:
• 31 A. Abciximab
• 32
B. Alteplase
• 33
o34 C. Aspirin
o35 D. Cilostazol
• 36
E. Clopidogrel
• 37
• 38

a
Lock
s
Suspend
8
End Block
Item: 25 of 38 ~. I • M k <:] t> al ~· ~
QIO: 5776 .l. ar Previous Next lab 'lifllues Notes Calculator


18
19 The correct answer is C. 32°/o chose this.
20
21
22
23
24
25
• 26
• 27
• 28
• 29
o30
• 31
• 32 Images courtesy of Lucien Monfils (left) and Dr. James
Heilman (right)
• 33
o34
This patient has three of the risk factors for ischemic stroke : hypertension, hypercholesterolemia, and
advanced age; other risk factors include cigarette smoking and diabetes mellitus. Neuroimaging is needed to
o35 differentiate ischemic stroke from hemorrhagic stroke (see images) . I schemic stroke (left image) appears as a
• 36 hypodense area of infarction, whereas hemorrhag ic stroke (right image) appears as hyperdense and " bright" on
• 37 CT .
• 38 ~
Thrombolytic therapy is the most effective therapy for salvaging noninfarcted ischemic brain t issue. Agents ~

6
lock
s
Suspend
0
End Block
Item: 25 of 38 ~. I • M k <:] t> al ~· ~
QIO: 5776 .l. ar Previous Next lab 'lifllues Notes Calculator


18
Thrombolytic therapy is the most effective the rapy for salvaging noninfarcted ischemic brain t issue. Agents
19 such as alteplase and streptokinase are very effective, but t hei r benefit decreases as their administration is
20 de layed. In this patient, w hose stroke happened 5 hou rs ago, the eff ectiveness of alteplase therapy is
21
unproven . Other choices for management include antiplatelet drugs and anticoagu lants .
22 Aspirin is the only antiplatelet agent with proven effectiveness in the ea r ly t reatment of ischemic stroke . Th is
nonsteroidal anti-inflammatory agent irreversibly inhibits cyclooxygenase, thus preventing the production of
23
th romboxane A 2 . There is some evidence that combination therapy with cilostazol (wh ich inhibits
24 phosphodiesterase and thus platelet aggregation) and aspirin may be more eff ective for secondary stro ke
25 prevention than aspir in alone, but cilostazol by itse lf is not as effective as aspirin in th is patient population.
• 26 Abciximab (a glycoprotein receptor lib/ Ilia inhibitor ) and clopidogrel (an adenosine diphosphate) are used to
• 27 prevent acute coronary synd rome, not ischemic stroke as in this patient. Warfa r in, an anticoagulant, is more
• 28
likely to be used fo r chronic prophylaxis against further clot formation .
• 29 Cilostazol Warfarin Anticoagulant Clopidogrel Stroke Antiplatelet drug Tissue plasminogen activator Streptokinase Acute coronary syndrome Thrombolytic drug

Adenosine diphosphate Abciximab Thrombolysis Diabetes mellitus Aspirin Ischemic stroke Cyclooxygenase Hypercholesterolemia Thromboxane A2 Platelet
o30
• 31 Hypertension Glycoprotein Platelet aggregation Ischemia Thromboxane Neuroimaging Anti-inflammatory Adenosine Tobacco smoking Phosphodiesterase Brain

• 32 Preventive healthcare Thrombus Smoking

• 33 A is not correct. 10% chose this.


o34 Abciximab is used to prevent ca rdiac complications such as acute co ronary synd rome or for percutaneous
o35 transluminal coronary angioplasty. It is a monoclona l antibody aga inst the glycop rotein receptor lib/ Ilia on
• 36
activated plate lets, thereby preventing platelet aggregation .
Acute coronary syndrome Angioplasty Abciximab Platelet Monoclonal antibody Percutaneous coronary intervention Antibody Platelet aggregation Glycoprotein
• 37
• 38 B is not correct. 29% chose this .
~

6
lock
s
Suspend
0
End Block
Item: 25 of 38 ~. I • M k <:] t> al ~· ~
QIO: 5776 .l. ar Previous Next lab 'lifllues Notes Calculator

18 B is not correct. 29% chose this.


19 Alteplase is a tissue plasminogen activator, a thrombolytic that enhances the conve rsion of plasminogen to
20
plasmin, the major fibrinolytic enzyme that cleaves thrombin and f ibr in clots. I t is used to treat early myocard ial
infarction (when primary percutaneous coronary intervention is not avai lable) and early ischemic stroke.
21 However, it is contra indicated if more than 3-4.5 hou rs have passed since the onset of symptoms, or if the
22 patient has abnor mal laboratory test va lues, high blood pressure, or a history of recent surgery. Therefore, it is
23 not indicated in this patient.
Tissue plasminogen activator Percutaneous coronary intervention Plasmin Myocardial infarction Fibrin Thrombin Enzyme Ischemic stroke Fibrinolysis Thrombolysis
24
Stroke Thrombolytic drug Ischemia Blood pressure Hypertension Contraindication Infarction
25
• 26 D is not correct. 14% chose this .
• 27 Ci lostazo l acts as a phosphodiesterase III inhibitor, which leads to inh ibition of platelet aggregation and
vasodilation. Currently its main indication is for treating intermittent claudication (arter ial insufficiency) in
• 28
patients with per ipheral vascu lar disease .
• 29 Cilostazol Intermittent claudication Phosphodiesterase 3 Vasodilation Peripheral artery disease Platelet Claudication Phosphodiesterase Vascular disease

o30 Platelet aggregation


• 31
E is not correct. 15% chose this .
• 32
Clopidogrel works by irreversibly blocking adenosine diphosphate receptors, thus inhibiting platelet aggregation .
• 33 It is used in acute coronary syndrome without ST-segment elevation and is commonly prescr ibed for the
o34 prevention of thrombotic occlusions after coronary stenting.
Clopidogrel Acute coronary syndrome Adenosine diphosphate Platelet Percutaneous coronary intervention Platelet aggregation Thrombosis Adenosine Stent
o35
Coronary stent
• 36
• 37

• 38 Bottom Line:
6
lock Suspend
s 0
End Block
Item: 25 of 38 ~. I • M k <:] t> al ~· ~
QIO: 5776 .l. ar Previous Next lab 'lifllues Notes Calculator

18
• However, it is contra indicated if more than 3-4.5 hou rs have passed since the onset of symptoms, or if the
patient has abnor mal laboratory test va lues, high blood pressure, or a history of recent surgery. Therefore, it is
19
not indicated in this patient.
20 Tissue plasminogen activator Percutaneous coronary intervention Plasmin Myocardial infarction Fibrin Thrombin Enzyme Ischemic stroke Fibrinolysis Thrombolysis

21 Stroke Thrombolytic drug Ischemia Blood pressure Hypertension Contraindication Infarction

22
D is not correct. 14% chose this.
23
Ci lostazo l acts as a phosphodiesterase III inhibitor, which leads to inh ibition of platelet aggregation and
24 vasodilation. Currently its main indication is for treating intermittent claudication (arter ial insufficiency) in
25 patients with per ipheral vascu lar disease.
Cilostazol Intermittent claudication Phosphodiesterase 3 Vasodilation Peripheral artery disease Platelet Claudication Phosphodiesterase Vascular disease
• 26
Platelet aggregation
• 27
• 28 E is not correct. 15% chose this .
• 29 Clopidogrel works by irreversibly blocking adenosine diphosphate receptors, thus inhibiting platelet aggregation .
o30
It is used in acute coronary syndrome without ST-segment elevation and is commonly prescr ibed for the
prevention of thrombotic occlusions after coronary stenting .
• 31 Clopidogrel Acute coronary syndrome Adenosine diphosphate Platelet Percutaneous coronary intervention Platelet aggregation Thrombosis Adenosine Stent
• 32 Coronary stent
• 33
o34
o35
Bottom Line:
• 36 Treatment for ischemic stroke includes thrombo lytics (if given <4.5 hou rs after stro ke onset), antip late let
agents such as aspirin, and anticoagulants such as warfarin .
• 37
Stroke Ischemic stroke Thrombolysis Ischemia Thrombolytic drug
• 38 ~

6
lock Suspend
s 0
End Block
Item: 26 of 38 ~. I • M k <:] t> al ~· ~
QIO: 3475 .l. ar Previous Next lab 'lifllues Notes Calculator


18
A 2 1-yea r-old man awakens one morn ing with su dden onset of fever, stiff neck, and a rash on his lower
19 extremity and abdomen . He is brought to the emergency department (ED) by his col lege dormitory
20 roommate. As he sits on his bed in the ED, he asks for the ceiling light to be turned off. On question ing, he
does not know where he is or what year it is. He has no significant med ical history. He is not an intravenous drug
21
user, nor has he ever engaged in r isky sexua l behaviors. Given the potential ly life-th reatening nature of his illness,
22 he is given an empiric antibiotic to t reat for the most likely causative agent.
23
24 What is the mechan ism of action of the dr ug most likely administered?
25
:
• 26
• 27
A. I nhibition of bacteria l transpeptidase th rough substrate alteration
• 28
• 29 B. I nhibition of dihydropteroate synthase
o30 C. I nhibition of protein synthesis by decreasing the 305 ribosoma l subunit's affinity for aminoacyl-transfer
• 31 RNA
• 32 D. I rreversible inhibition of bacterial transpeptidase
• 33 E. Promotion of the misread ing of messenger RNA
o34
o35
• 36
• 37
• 38 ~

6
lock
s
Suspend
0
End Block
Item: 26 of 3 8 ~ 1 • M k -<:J 1>- Jil ~· !:';-~
QIO: 3475 ..L ar Pre v ious Next Lab fli!ltues Not es Calcula t o r
A A

18
19
The co rrect a n s w e r is D. 47°/o c hose this .
20
As the image shows, this patient has many of the classic Symptoms of
symptoms of bacterial mening itis: photophobia, fever, and stiff
21 neck. He is a young college student living in a dormitory, wh ich Meningitis
22
23
24
makes Neisseria meningitidis the most likely causative agent.
Streptococcus pneumoniae is also a common causative agent in
adults. Therefore, the first-line treatment choice for bacterial
meningitis in adults includes a third-generation cephalosporin such
Central - - - - - -
-Headache
- Altered mental status
I
25 as ceftriaxone. Cephalosporins are bactericidal agents that contain Ears -------'"'1
26 a 13-lactam ring and that inhibit cell-wall synthesis and have good - Phonophobia
• 27
penetration of the centra l nervous system. They are separated into Eyes --------J
four generations, each w ith increasing gram-negative coverage.
• 28 - Photophobia /
Fourth-gene ration and some third-generation cepha losporins are
• 29 antipseudomona l as well.
Neck ---~""-....J
Cephalosporin Neisseria meningitidis Streptococcus pneumoniae Ceftriaxone Photophobia
o30 -Stiffness
• 31 Central nervous system Gram-negative bacteria Meningitis Bactericide Streptococcus

Bacterial meningitis Neisseria Nervous system Fever Neck stiffness Systemic


• 32
-High fever
• 33
0 34 Trunk, ----~-­

0 35
mucu s 0

membranes,
• 36 •
extremities
• 37 (if meningo- •
• •
• 38

coccal

a
Lock
s
Suspend
8
End Bl ock
18 neck. He is a young college student living in a dorm itory, wh ich
19 makes Neisseria meningitidis the most likely causative agent. Central - - - - - -
Streptococcus pneumoniae is also a common causative agent in
20 -Headache
adults. Therefore, the first-l ine treatment choice for bacterial
- Altered mental status
21 mening it is in adults includes a third-generation cephalosporin such
as ceftriaxone. Cephalosporins are bactericidal agents that contain Ears _______,.,.
22
a ~-lactam ring and that inhi bit cell -wall synthesis and have good - Phonophobia
23
penetration of the central nervous system. They are separated into Eyes - - - - - - J
24 four generations, each with increasing gram-negative coverage.
- Photophobia /
25 Fourth-generation and some third-generation cepha losporins are
antipseudomonal as well.
26 Neck ---~~...J
Cephalospo in Neisseria meningitidis Streptococcus pneumoniae Ceftriaxone Photophobia
. 27 -Stiffness
Central nervous system Gram-negative bacteria Meningitis Bactericide Streptococcus
. 28
Bacterial meningitis Neisseria Nervous system Fever Neck stiffness Systemic
. 29 -High fever
o30
Trunk, -----~-­
• 31
mucus •
• 32 membranes,

• 33 extremities
o34 (if meningo- •
coccal •
o35
infection) •
. 36
. 37
- Petechiae
---
Image courtesy of Mikae/ Haggstrom
. 38

a
Lock
s
Suspend
8
End Block
Item: 26 of 38 ~. I • M k <:] t> al ~· ~
QIO: 3475 .l. ar Previous Next lab 'lifllues Notes Calculator


18 A is not correct. 22% chose this .
19 Vancomycin binds to the D-ala D-ala portions of the bacte ria l cel l wa ll precursor preventing their cross-linking
via bacteria l t ranspeptidase. However, vancomycin has minimal activity against gram-negative organisms, and
20
this patient needs coverage against these bugs.
21 Vancomycin 00-transpeptidase Gram-negative bacteria Cell wall Peptidyl transferase

22
B is not correct. 7°/o chose this.
23
5u lfonamides such as sulfamethoxazole and su lfisoxazo le are para-aminobenzoic acid anti-metabol ites that
24 inhibit dihydropteroate synthase . 5u lfonamides are considered bacteriostatic agents. They are frequently
25 combined with trimethoprim and are se ldom used as first-line agents for the life-threaten ing infection suspected
26 in this patient.
Bacteriostatic agent Trimethoprim Oihydropteroate synthase Sulfonamide Sulfonamide (medicine) Sulfamethoxazole Sulfafurazole 4-Aminobenzoic acid
• 27
• 28 C is not correct. 19% chose this .
• 29 Tetracycl ines such as doxycycline are bacteriostatic agents that inhibit protein synthesis by inhibit ing the
binding of aminoacyl-transfer RNA to the 305 r ibosomal subunit. In addition to being bacteriostatic,
o30
tetracyclines also have low central nervous system penetration, making this antibiotic class a poor treatment
• 31 choice in this case .
• 32 Doxycycline Central nervous system Antibiotics Bacteriostatic agent Tetracycline antibiotics 30S Protein Protein synthesis Protein biosynthesis RNA

• 33 E is not correct. 5°/o chose this.


o34 Aminog lycosides such as gentamicin and neomycin bind to the 305 ribosoma l subunit, inhibiting formation of
o35 the init iation comp lex. Aminoglycosides also cause misread ing of messenger RNA and are bactericidal. They are
• 36
often combined with a ~- l actam due to the synergistic effect. Aminoglycosides are usefu l against gram-negative
rods, but do little to curb infections by gram-negative cocci, w hich must be considered in this case .
• 37 Messenger RNA Neomycin Gentamicin Gram-negative bacteria Aminoglycoside 30S Coccus Bactericide RNA Ribosome Synergy
• 38 ~

6
lock
s
Suspend
0
End Block
Item: 26 of 38 ~. I • M k <:] t> al ~· ~
QIO: 3475 .l. ar Previous Next lab 'lifllues Notes Calculator

18 in this patient.
Bacteriostatic agent Trimethoprim Dihydropteroate synthase Sulfonamide Sulfonamide (medicine) Sulfamethoxazole Sulfafurazole 4-Aminobenzoic acid
19
20 C is not correct. 19% chose this.
21 Tetracycl ines such as doxycycline are bacte riostatic agents that inhibit protein synthesis by inhibit ing the
binding of aminoacyl-transfer RNA to the 305 r ibosomal subunit. In addition to being bacteriostatic,
22
tetracyclines also have low central ne rvous system penetration, making this antibiotic class a poor treatment
23 choice in this case.
24 Doxycycline Central nervous system Antibiotics Bacteriostatic agent Tetracycline antibiotics 30S Protein Protein synthesis Protein biosynthesis RNA

25
E is not correct. 5°/o chose this.
26 Aminog lycosides such as gentamicin and neomycin bind to the 305 ribosoma l subunit, inhibiting formation of
• 27 the initiation comp lex. Aminoglycosides also cause misread ing of messenger RNA and are bactericidal. They are
• 28 often combined with a ~- l actam due to the synergistic effect. Aminoglycosides are usefu l against gram-negative
rods, but do little to curb infections by gram-negative cocci, w hich must be considered in this case .
• 29
Messenger RNA Neomycin Gentamicin Gram-negative bacteria Aminoglycoside 30S Coccus Bactericide RNA Ribosome Synergy
o30
• 31
• 32 Bottom Line:
• 33 The most common bacte rial species causing mening it is in young people is N. meningit idis . Thi rd-generation
cephalospor ins are excellent choices for empi ric t reatment of men ingococca l disease. The most common
o34
antibiotic classes, such as aminoglycosides, macrol ides, penici llins, cephalospo rins, fluoroquino lones, and
o35 sulfonamides, have diffe rent mechanisms of action, and it is important to know these differences.
• 36 Meningitis Antibiotics Quinolone Neisseria meningitidis Penicillin Sulfonamide (medicine) Aminoglycoside Macrolide Meningococcal disease Cephalosporin
Empiric therapy
• 37

• 38

6
lock Suspend
s 0
End Block
Item: 26 of 3 8 ~ 1 • M k -<:J 1>- Jil ~· !:';-~
QIO: 3475 ..L ar Pre v ious Next Lab fli!ltues Not es Calcula t o r
A A

18
FA17 p 185.1
19
Cephalosporins (generations 1- V)
20
MECHANISM ~-lactam drugs that inhibit cell wall S) nthesis Organisms typically not co,·ered by 1st-4th
21
but are less susceptible to penicillinases. generation cephalosporins are LA~l E:
22 Bactericidal. Listeria, At~-pica ls (Chlamydia, Mycoplasma),
23 ~ IRSA, and E nterococci.
24 CLINICALUSE 1st generation (cefazolin, cephalexin)-gram Q?) lsi generation- PEcK.
25 cocci, Proteus rnirabilis, E coli, Klebsiella
26 pneumoniae. Cefazolin used prior to surger} to
pre,·ent S aureus wound infections.
• 27
2nd generation (cefaclor, ccfo,ilin, rake fm fur .
• 28
cefuroxime)- gram Q?> cocci, II influenzae, 2nd generation- HEI\S PEcK.
• 29 Enterobacter aerogenes, "\leisserict spp., Serratia
o30 marcescens, Proteus mirabilis, E coli, Klebsiella
• 31 pneumoniae.
• 32
3rd generation (ceftriaxone, cefotaxime, Can cross blood-brain barrier.

• 33
cefpodoxime, ceftazidime)-serious gram e Ccftriaxone-meningitis, gonorrhea,
infections resistant to other ~-l actams. disseminated Lyme disease.
0 34 Ceftazidime- Pseudomonas.
0 35 e
4th generation (cefcpime)-gram organisms,
• 36 with t activity against Pseudomonas and gram
• 37 Q?> organisms.

• 38 5th generation (ceftaroline)-broad gram ® and


a
Lock
s
Suspend
8
End Bl ock
Item: 26 of 3 8 ~ 1 • M k -<:J 1>- Jil ~· !:';-~
QIO: 3475 ..L ar Pre v ious Next Lab fli!ltues Not es Calcula t o r

18
A
2nd generation (ccfaclor, ccfO'I. ilin, Fake fo~ fur. A

cefuroxime)- gram ®cocci, 11 influenzae, 2nd generation- H ENS PEcK.


19
Enterobacter aerogenes, \ieisseria spp., Serratia
20
marcescens, Proteus mirabilis, E coli, Klebsiella
21 pneumoniae.
22 3rd generation (ceftriaxone, cefotaxime, Can cross blood-brain barrier.
23 cefpodoxime, ceftazidim e)-serious gram e Ceft riaxone- meningitis, gonorrhea,
24
infections resistant to other ~-lactams. dissem inated Lyme disease.
Cefta1idime-Pseudomonas.
25
e
4th generation (cefepime)-gram organisms,
26
with f acti\'ity against Pseudomonas and gram
• 27 ®organisms.
• 28 5th generation (ceftaroline)-broad gram ® and
• 29 e
gram organism coverage; unlike 1st-4th
o30 generation cephalosporins, ceftaroline covers
Listeria, MRSA, and Enterococcus faecalis-
• 31
cloes not cover Pseudomonas .
• 32
ADVERSE EFFECTS Hypersensitivity reactions, au toimmune
• 33
hemolyt ic anemia, disulfiram-like reaction,
0 34 vitamin K defi ciency. Low rate of cross-
0 35 reactivity c\·en in penicillin-allergic patients.
• 36 f nephrotoxicity of aminoglycosides.
• 37 MECHANISMOF RESISTANCE Structural change in penicillin-binding proteins
• 38 (Lranspeptidases).

a
Lock
s
Suspend
8
End Bl ock
Item: 26 of 3 8 ~ 1 • M k -<:J 1>- Jil ~· !:';-~
QIO: 3475 ..L ar Pre v ious Next Lab fli!ltues Not es Calcula t o r


18
19
FA17 p 183.1
20
Antimicrobial therapy
21
22
GYRASE
FOUC ACID SYNTHESIS
23 AIID REDUCTION
24 (DNA methy1.lt1011) Metrollldazole Auoroquinolones
Oprofloxaon
25 Levotloxac&n, etc
Sulfonamides
Sulfamethoxazole BACTERIAL CELL Ouinolone
26 Nalidixic aad
Sulfisoxazole
• 27 Sulfadiazine

• 28
Tnmelhoprim
• 29
N
o30 ,olymc a
PROTEIN SYNTHESIS
• 31
• 32 50S SUBUNIT
mRNA
• 33 Chloramphenicol
Clindamyc1n
0 34 '
CELL WALL SYNTHESIS
Linezohd
THF
35 Maao~des
0
PTID' C YC N YI'THESIS
• 36 Glycopeptides
Vancomycin
r. . ------
.-{J /
Cell membrane
Azlthromyc1n
Clarilhromyon
£rythromyc1n
• 37
• 38
.._ Baotracm ft Ce(/ Wall Streptogramins
OUinupnSIJn

a
Lock
s
Suspend
8
End Bl ock
18 Fluoroquinolones
19 Ciprofloxaon
Levofloxadn. etc
Sullonamides
20 Sullamethoxazole BACTERIAL CELL Ouinolone
Sulfisoxazole Nalidixic aCid
21 Sulladwine
22
23 Tnmethopnm

24
,_.,
25 Pfi0T£1HSYNTlfESIS
26 50S SUBUNIT
mRNA
• 27 ChlorampheniCol
• 28 Clmdamyon
CELL WALl SYNTHESIS
Linezobd
• 29 THF
YNTHESIS Macrolides
o30 Az1thromycin
Glycopeptides Celt membrane Clarithromyon
• 31 Vancomyctn Erythromycin
Bac1tracin Celt Wall Streptogramins
• 32

• 33 'EPTIDOG YCAN 'ROSS-LINKING


• Ouinupristin
Dallopristin

o3 4 ~lcilllnase-sensitive penicillins Antipseudomonal Carbapenem1


30S SUBUNIT

o3 5
I r PeniCillin G. v Ticarcillin lm1penem Aminoglycosides Glycyctines
A.mptclihn Piperacillin Meropenem GentamiCin Tigecycline
• 36 Amoxic1llin Cephalosporins U·Vl Ertapenem Neomyon
Doripenem Tetracyclines
Penicillinase-resistant penicillins lst-Cetazolin. etc Amik.lcin Tetracycline
• 37 Oxac1lhn 2nd-Cefoxitin. etc Monobactams Tobramycin Doxycycline
Nafclll.in 3rd-Ceftriaxone. etc Aztreonam Streptomydn Minocydine
• 38 Oorlnv>r1lhn 4th-CPfPOII1lP

a
Lock
s
Suspen d
8
End Bl ock
18 3rd-Ceftriaxone. etc
4th-Cefepime
19 Sth-Ceftaro6ne
20
21
22 FA17 p 187.2

23 Aminoglycosides G entamicin, ~eom) cin, \ mikacin, "\lean" (aminoglycoside) C~\TS ca'\"JOT


24 Tobramycin. Streptomycin. kill anaerobes.
25 MECHANISM Bactericidal; irreversible inhibition of initiation
26 complex through binding of the 30S subunit.
• 27 Can cause misreading of mRi . Also block
translocation. Require 0 2 for uptake; therefore
• 28
ineffective against anaerobes.
• 29
CLINICAL USE Severe gram 8 rod infections. S> nergistic with
o30
~-lactam antibiotics .
• 31 Neomycin for bowel surgery.
• 32 ADVERSE EFfECTS l':cphrotoxicity, cmomuscular blockade,
• 33 O totoxicity (especially when used with loop
o34 diuretics). Teratogen.
o35 MECHANISM OF RESISTANCE Bacterial transferase enzymes inactivate the

I
• 36 drug by acetylation, phosphorylation, or
• 37
adenylation .

• 38

a
Lock
s
Suspend
8
End Block
Item: 28 of 3 8 ~ 1 • M k -<:J 1>- Jil ~· !:';-~
QIO: 1487 ..L ar Pre v ious Next Lab fli!ltues Not es Calcula t o r

IAA]
A A

18
A 32-year-old woman presents to her primary care physician complaining of chronic headaches. She
19 describes the headaches as a throbbing, unilateral pain. She denies any symptoms preceding the onset of
20 her headaches. When these episodes occur at work, she states, it helps to turn off the lights and lie down on
21
a couch. Her headaches last 10 hours on average. She also reports occasional nausea and vomiting with the
headaches.
22
23
What medication can be used in the acute treatment of this patient's condition?
24
:
25
A. Amitriptyline
26
• 27
B. Nadolol
• 28 C. Propranolol
• 29 D. Sumatriptan
o30
E. Verapami l
• 31
• 32
• 33
0 34
0 35
• 36
• 37
• 38

a
Lock
s
Suspend
8
End Bl ock
Item: 28 of 3 8 ~ 1 • M k -<:J 1>- Jil ~· !:';-~
QIO: 1487 ..L ar Pre v ious Next Lab fli!ltues Not es Calcula t o r

IAA]
A A

18
A 32-year-old woman presents to her primary care physician complaining of chronic headaches. She
19 describes the headaches as a throbbing, unilateral pain. She denies any symptoms preceding the onset of
20 her headaches. When these episodes occur at work, she states, it helps to turn off the lights and lie down on
21
a couch. Her headaches last 10 hours on average. She also reports occasional nausea and vomiting with the
headaches.
22
23
What medication can be used in the acute treatment of this patient's condition?
24
:
25
A. Amitriptyline
26
• 27
B. Nadolol
• 28 C. Propranolol
• 29 D. Sumatriptan
o30
E. Verapami l
• 31
• 32
• 33
0 34
0 35
• 36
• 37
• 38

a
Lock
s
Suspend
8
End Bl ock
Item: 28 of 38 ~. I • M k <:] t> al ~· ~
QIO: 1487 .l. ar Previous Next lab 'lifllues Notes Calculator


18
The correct answer is D. 77°/o chose this.
19
Sumatr iptan is a seroton in agonist at 5-HT10 and 5-HT 16 receptors, and is a first-line agent for the acute
20
treatment of migraine headaches. Triptans can abort or reduce the sever ity of migraine headaches and also are
21 used to treat cluster headaches.
22 Sumatriptan Serotonin Migraine Triptan Serotonin receptor agonist Cluster headache Agonist Receptor (biochemistry)

23 A is not correct. 8°/o chose this.


24 Amitr iptyl ine, a tricyclic antidepressant, acts by decreasing reuptake of seroton in and norepinephrine by the
25 presynaptic neuron. It has prophylactic value in the treatment of migra ine and also is used to treat depression,
26
neuropathy, and chronic pain.
Tricyclic antidepressant Amitriptyline Serotonin Antidepressant Migraine Neuron Norepinephrine Chemical synapse Reuptake Chronic pain Peripheral neuropathy
• 27
Major depressive disorder Depression (mood) Preventive healthcare Tricyclic
28
• 29 B is not correct. 3°/o chose this .
o30
Nadolol is a second-l ine ~-blocker that can be used for migraine prophylaxis as wel l as for hypertension and
angina . Potentia l adverse effects include nightmares, paresthesias, bradycard ia, and fatigue .
• 31 Nadolol Bradycardia Migraine Hypertension Angina pectoris Paresthesia Fatigue (medical) Preventive healthcare
• 32
C is not correct. 7°/o chose this .
• 33
Propranolol is a nonse lective ~-b l ocker that is prescribed for migraine prophylaxis, but not in the acute setting .
o34 Propranolol Migraine Preventive healthcare Binding selectivity
o35
E is not correct. 5°/o chose this .
• 36
Verapamil is ca lcium channel blocker that can be used to prevent migraines and cluster headaches, but it is not
• 37
an abortive therapy .
• 38 ~
Calcium channel blocker Verapamil Migraine Calcium channel Cluster headache Calcium

6
lock
s
Suspend
0
End Block
Item: 28 of 38 ~. I • M k <:] t> al ~· ~
QIO: 1487 .l. ar Previous Next lab 'lifllues Notes Calculator


18 Amitr iptyl ine, a tricyclic antidepressant, acts by decreasing reuptake of seroton in and norepinephrine by the
19
presynaptic neuron. It has prophylactic value in the treatment of migra ine and also is used to treat depression,
neuropathy, and chronic pain.
20 Tricyclic antidepressant Amitriptyline Serotonin Antidepressant Migraine Neuron Norepinephrine Chemical synapse Reuptake Chronic pain Peripheral neuropathy
21 Major depressive disorder Depression (mood) Preventive healthcare Tricyclic
22
B is not correct. 3°/o chose this.
23
Nadolol is a second-l ine ~-bloc ker that can be used for migraine prophylaxis as wel l as for hypertension and
24
angina . Potentia l adverse effects include nightmares, paresthesias, bradycard ia, and fatigue.
25 Nadolol Bradycardia Migraine Hypertension Angina pectoris Paresthesia Fatigue (medical) Preventive healthcare

26
C is not correct. 7°/o chose this .
• 27
Propranolol is a nonse lective ~-b l ocker that is prescribed for migraine prophylaxis, but not in the acute setting .
28 Propranolol Migraine Preventive healthcare Binding selectivity

• 29
E is not correct. 5°/o chose this.
o30
Verapamil is ca lcium channel blocker that can be used to prevent migraines and cluster headaches, but it is not
• 31 an abortive therapy .
• 32 Calcium channel blocker Verapamil Migraine Calcium channel Cluster headache Calcium

• 33
o34 Bottom Line:
o35
Sumatr iptan is usefu l for treating migraine headaches in the acute setting. Certain ~-b l ockers, calcium channe l
• 36 blockers, and t r icyclic antidepressants are usefu l in the prophylaxis of migraine headaches.
• 37 Sumatriptan Migraine Tricyclic antidepressant Calcium channel blocker Antidepressant Calcium channel Calcium Preventive healthcare Tricyclic

• 38 ~

6
lock
s
Suspend
0
End Block
Item: 28 of 38 ~. I • M k <:] t> al ~· ~
QIO: 1487 .l. ar Previous Next lab 'lifllues Notes Calculator

6
lock
s
Suspend
0
End Block
Item: 28 of 3 8 ~ 1 • M k -<:J 1>- Jil ~· !:';-~
QIO: 1487 ..L ar Pre v ious Next Lab fli!ltues Not es Calcula t o r
A A

18 FA17 p 488.1

19 Headaches Pain due to irritation of structures such as the dura, cran ial nerves, or extracranial structures. More
20 common in fema les, except cluster headaches.
CLASSIFICATION lOCAliZATION DURATION DESCRIPTION TREATMENT
21
22
Cluste,.a Unilateral 15 min-3 hr; Repetiti'e brief headaches. Acute: sumatriptan, 100% 0 2
repetiti,·e Excruciating periorbital Prophylaxis: ,·erapamil
23
pain "ith lacrimation and
24 rhinorrhea. ~lay present with
25 Horner syndrome.
26 Tension Bilateral > 30 min Steady pain. r-..o photophobia Analgesics, 1'-:S.\ IDs,
• 27 (typically 4-6 or phonophobia. 1 o aura. acetaminophen;
28
hr); constant amitriptyline for chronic
pain
• 29
o30
Migraine Unilateral 4-72 hr Pulsating pain with Acute: 1SA!Ds, triptans,
nausea, photophobia, or dihydroergotamine
• 31
phonophobia. May hm·e Prophylaxis: lifestyle changes
• 32 "aura." Due to irritation of (eg, sleep, exercise, diet},
• 33 CN V, men inges, or blood P-blockers, calcium channel
o3 4 ,·essels (release of substance blockers, amitriptyline,
o3 5
P, ca lcitonin gene-related topiramate, valproate.
peptide, vasoacli\'e peptides). POUi\ 0 - Pulsati le, O ne-dar
• 36
duration, Unilateral, '\ausea,
• 37 Disabling
• 38

a
Lock
s
Suspend
8
End Bl ock
Item: 29 of 3 8 ~ 1 • M k -<:J 1>- Jil ~· !:';-~
QIO: 5195 ..L ar Pre v ious Next Labfli!llues Not es Calcula t o r

IAA]
A A

18
A young man comes to the psychiatric clinic with his parents. Whi le the patient refuses to speak to the
19 doctor, claiming "you are one of t hem, " his parents complain that in the past 6 months their son had dropped
20 out of college, moved back home, and now spends most of the day alone in his room complaining of voices in
21
his head . The doctor starts him on the appropriate med ical t reatment for his cond ition. He has never been on any
psychoactive drugs before. Two days later, the patient presents aga in to the clinic with fever, muscle rig idity,
22 unstable blood pressure, and delirium.
23
24 What type of adverse drug reaction is this patient experiencing?
25
26 A. Hypertensive crisis
. 27
B. Malignant hyperthermia
28
. 29 C. Neuroleptic ma lignant syndrome
o30 D. Serotonin syndrome
• 31
E. Tardive dyskinesia
• 32
• 33
o34
o35
. 36
. 37
. 38

a
Lock
s
Suspend
8
End Bl ock
Item: 29 of 38 ~. I • M k <:] t> al ~· ~
QIO: 5195 .l. ar Previous Next lab 'lifllues Notes Calculator


18
19 The correct answer is C. 68°/o chose this.
20 This patient suffers from sch izophrenia and was treated with antipsychotic drugs, wh ich work by blocking
21 excessive dopamine. Neu roleptic mal ignant syndrome (NMS) is an adverse reaction to antipsychotic dr ugs. Both
atypical and typica l antipsychotics, includ ing ch lorpromazine, haloperidol, and fluphenazine, can cause NMS. The
22
pathophysiology of NMS is blockade of dopamine receptors in the basal gangl ia, hypotha lamus, muscles, and
23 postgangl ion ic sympathetic neurons. It is character ized by muscu lar r igidity, fever, autonomic instability, and
24 elevated creatine kinase, myoglobinu r ia, dyspnea, diaphoresis, confusion, and altered menta l status. I t is also
25 associated with high liver enzymes, leukocytosis, and thrombocytosis. Risk factors include young age, male
gender, intramuscular injection, high dose of antipsychotic, ag itation, dehydration, and concurrent use of
26
lithium. This synd rome can be t reated with dantrolene and dopamine agonists. Mortality of NMS is 20% and is
• 27 usua lly due to acute rena l failure .
28 Fluphenazine Neuroleptic malignant syndrome Chlorpromazine Haloperidol Dantrolene Creatine kinase Basal ganglia Dyspnea Hypothalamus Perspiration

29 Dopamine Schizophrenia leukocytosis Antipsychotic Acute kidney injury Intramuscular injection Dysautonomia Creatine Autonomic nervous system liver

o30 Typical antipsychotic Dopamine agonist Dehydration Altered level of consciousness lithium Pathophysiology Adverse effect Dopamine receptor Neuron

• 31 Receptor (biochemistry) Fever Malignancy Kidney liver function tests Cancer

• 32 A is not correct. 3°/o chose this .


• 33 A hypertensive cr isis may come about in a patient taking a monoamine oxidase (MAO) inhibitor who also takes
o34 ~-agon i sts, or ingests too much tyrosine, an amino acid found in many foods such as cheese and red wine.

o35
Wh ile th is patient does have unstable blood pressu re, he would not be taking MAO inhibitors for his cond ition .
Amino acid Hypertensive crisis Hypertensive emergency Monoamine oxidase Monoamine oxidase inhibitor Monoamine neurotransmitter Tyrosine Blood pressure
• 36
Cheese Hypertension Oxidase Red wine
• 37
• 38 ~
B is not correct. 12% chose this .

6
lock
s
Suspend
0
End Block
Item: 29 of 38 ~. I • M k <:] t> al ~· ~
QIO: 5195 .l. ar Previous Next Lab 'lifllues Notes Calculator


18 B is not correct. 12% chose this.
19 Mal ignant hyperthermia is an adverse reaction to agents commonly used to induce or maintain general
20 anesthesia or pa ralysis, such as isoflurane, sevoflurane, and succinylchol ine. After receiving one of these
21 agents, patients become tachycardic, hypercapnic, and febr ile, with muscu lar rigidity. This reaction can also be
treated with dantrolene. Whi le this patient's symptoms are simi lar to those seen with malignant hyperthermia,
22
there is no reason to believe he is undergoing an operation at this t ime .
23 Isoflurane Dantrolene Malignant hyperthermia Sevoflurane Suxamethonium chloride Hyperthermia Tachycardia General anaesthesia Anesthesia Hypercapnia

24 Fever Paralysis Adverse effect

25
D is not correct. 12% chose this.
26
Seroton in synd rome is an adverse reaction resu lt ing from too much serotonin, often associated with selective
• 27 seroton in reuptake inhibitors (SSRis), opioids, and other central nervous system stimu lants. Onset is often
28 with in minutes, and symptoms include tachycard ia, di lated pupils, myoclonus, hyperthermia, high blood
29
pressure, and occasional ly muscu lar rigidity. Wh ile serotonin syndrome can be very difficu lt to distingu ish from
NMS, this patient is more like ly to be on a neuroleptic drug rather than an SSRI, as they are used to treat
o30 depression and anxiety .
• 31 Serotonin syndrome Central nervous system Myoclonus Selective serotonin reuptake inhibitor Hyperthermia Tachycardia Serotonin Stimulant Reuptake

• 32 Adverse drug reaction Blood pressure Opioid Anxiety Mydriasis Hypertension Antipsychotic Pupillary response Major depressive disorder Adverse effect

• 33 Depression (mood) Nervous system

o34
E is not correct. 5°/o chose this.
o35
Tardive dyskinesia is a common adverse effect of neuro leptic drugs, and is characterized by stereotyped facia l
• 36 or other repetitive movements. These movements usua lly start after longer -ter m use of antipsychotics (a few
• 37 months or longer ), and often cannot be stopped, even when patients are no longer taking these drugs. This
• 38
patient has not yet developed tardive dyskinesia .
~

6
lock Suspend
s 0
End Block
Item: 29 of 38 ~. I • M k <:] t> al ~· ~
QIO: 5195 .l. ar Previous Next Lab 'lifllues Notes Calculator


18
D is not correct. 12% chose this.
19
Se roton in synd rome is an adve rse reaction resu lt ing from too much serotonin, often associated wit h selective
20 seroton in reuptake inhibitors (SSRis), opioids, and other central nervous system stimu lants. Onset is often
21 with in minutes, and symptoms include tachyca rdia, di lated pupils, myoclonus, hyperthe rmia, high blood
22
pressure, and occasional ly muscu lar rigidity. Wh ile serotonin syndrome can be ve ry difficu lt to distingu ish f rom
NMS, this patient is more like ly to be on a neuroleptic drug rathe r than an SSRI, as they are used to treat
23 depression and anxiety .
24 Serotonin syndrome Central nervous system Myoclonus Selective serotonin reuptake inhibitor Hyperthermia Tachycardia Serotonin Stimulant Reuptake

25 Adverse drug reaction Blood pressure Opioid Anxiety Mydriasis Hypertension Antipsychotic Pupillary response Major depressive disorder Adverse effect

26 Depression (mood) Nervous system

• 27
E is not correct. 5°/o chose this.
28
Tardive dyskinesia is a common adverse effect of neuro leptic drugs, and is characterized by ste reotyped facia l
29 or other repetitive movements. These movements usua lly start after longer -ter m use of antipsychotics (a few
o30 months or longer ), and often cannot be stopped, even when patients are no longer taking these drugs. Th is
• 31
patient has not yet developed tardive dyskinesia .
Tardive dyskinesia Antipsychotic Dyskinesia Adverse effect
• 32
• 33
o34 Bottom Line:
o35 Schizoph renia is treated using typica l and atypical antipsychotic dr ugs, which can improve a patient's
• 36 cond ition, but have a number of serious adve rse effects. NMS is a rare but sometimes fata l adve rse effect, and
occu rs mo re often with typica l rather than atypica l antipsychotics .
• 37 Atypical antipsychotic Schizophrenia Antipsychotic Adverse effect
• 38 ~

6
lock Suspend
s 0
End Block
Item: 29 of 3 8 ~ 1 • M k -<:J 1>- Jil ~· !:';-~
QIO: 5195 ..L ar Pre v ious Next Labfli!llues Not es Calcula t o r
A A

18
19 FA17 p543.1
20 Typical antipsychotics Haloperidol, pimozide, triAuopera,inc, Auphen;u ine, thiorida:tine, chlorproma:tin e.
21 MECKANISM Block dopamine 0 2 receptor (t cAMP).
22 CliNICAl USE Schizophrenia (1° positi,·e symptoms), psrchosi~. bipolar disorder, delirium, Tourette syndrome,
23 Huntington disease, OCD.
24 POTENCY H igh potency: TriAuoperazine, fl uphena7ine, llaloperidol (Try to Fly H igh)- neurologic side
25 effects (eg, extrapyramidal S) mploms [EPS)).
26
Lo" potency: C hlorpromazine, Thioridazine (C heating l'hieves are low)-anticholinergie,
antihistamine, <Xrblockade effects.
. 27
ADVERSEEFFECTS Lipid soluble ..... stored in body fat ..... slow to be removed from body.
28
29 Endocrine: dopamine receptor an tagonism ..... hypcrprolactinemia ..... galactorrhea,
oligomenorrhea, gyn ecomastia.
o30
fetabolic: dyslipidemia, weight gain, hyperglycem ia .
• 31
Antimuscarinic: dry mouth, constipation .
• 32 Antihistamine: sedation.
• 33 <Xrblockade: orthostatic hypotension .
o3 4 Cardiac: QT prolongation.
Ophthalmologic: C hlorpromazine- C orneal deposits; T hioridazine- reT inal deposits.
o35
. 36 EPS-ADAPT:
. 37 Hours to days: Acute D ystonia (muscle spasm, stiffness, oculogyric crisis).
Days to months: \ kathisia (restlessness), Parkinsonism (bradykinesia).
. 38
• • \6- - '-L - '-- · · - - - · "T"- -..l! . • - .J . . - 1 :.~--!- '--- f --!- 1 - 1... ----'

a
Lock
s
Suspend
8
End Bl ock
Item: 29 of 3 8 ~ 1 • M k -<:J 1>- Jil ~· !:';-~
QIO: 5195

18
..L
'
ar Pre v ious Next
. .
- .
Labfli!llues
-
Not es Calcula t o r

effects (eg, extrapyramidal S) mptoms [fi:PS]).


. . . .. - .·-
19 Lo'' potency: C h lorpromazine, Thioridazine (Cheat ing T hieves are low)-anticholinergic,
20 antihistamine, a 1-blockade effects.
21 ADVERSE EFFECTS Lipid soluble - stored in body fat - slow to be removed from body.
22 Endocrine: dopamine receptor antagonism - hypcrprolactinemia - galactorrhea,
23 oligomenorrhea, g}11ecomastia.
24 .\ fetabolic: dyslipidemia, weight gain, h)l)crglyccmia.
25 Antimuscarinic: dry mouth, constipation.
Antihistamine: sedation.
26
a 1-blockade: orthostatic h}~otension .
. 27 Cardiac: QT prolongation.
28 Ophthalmologic: C hlorpromazine- C orneal deposits; T hioridazine-reT inal deposits.
29
EPS-ADAPT:
o30 Hours to days: Acute Dystonia (muscle spasm , stiffness, oculogyric crisis).
• 31 Days to months: Akathisia (restlessness), Parkinsonism (bradykinesia).
• 32 Months to years: Tardive dyskinesia (orofacial cho rea).
• 33
Treatment: benztropine (acute dystonia, tardive dyskinesia), benzodiazepines, ~-blockers
(akathisia).
o34
o35 Neuroleptic malignant syndrome (NMS)- \1 alignant FEVER: ~1yoglobinuria, Fever,
E ncephalopathy, unstable Vitals, t Enzymes, muscle Rigidity. Treatment: dantrolene, 0 2 ago nist
. 36
(eg, bromocriptine).
. 37
. 38

Lock
a s
Suspend
8
End Bl ock
Item: 30 of 3 8 ~ 1 • M k -<:J 1>- Jil ~· !:';-~
QIO: 4690 ..L ar Pre v ious Next Lab fli!ltues Notes Calcula t o r

IAA]
A A

18
A 42-year-old obtunded man is brought to the emergency department for evaluation and treatment. Resu lts
19 of an MRI of the head prompt the performance of a lumbar puncture. Laboratory tests show:
20 Opening pressure: 215 mm H20
21 Lymphocytes: 100/ mm3
22 RBCs: SOO/ mm 3
Protein: 30 mg/ dL
23
Glucose: 50 mg/ dL
24
25 No xanthochromia is noted.
26
. 27 What class of drugs should be used as initia l empiric treatment for this patient?
28
:
29 A. Antibiotic (gram-negative coverage)
o30
B. Antibiotic (gram-positive coverage)
• 31
• 32 C. Antifungal agents
• 33 D. Antiretroviral agents
o34
E. Antitubercu lous agent s
o35
F. Antiviral agents
. 36
. 37
. 38

a
Lock
s
Suspend
8
End Bl ock
Item: 30 of 3 8 ~ 1 • M k -<:J 1>- Jil ~· !:';-~
QIO: 4690 ..L ar Pre v ious Next Lab fli!ltues Notes Calcula t o r
A A

18
19 The correct answer is F. 55°/o chose this.
20 This patient has herpes encephalitis, which is infection of the cent ra l nervous
21 system with herpes simplex virus (HSV). Affected patients may develop
characteristic lesions in the temporal and deep frontal lobes (as shown in the
22
image), which result in seizure and other neurologic manifestations. Lumbar
23 puncture will typically yield cerebrospinal fluid (CSF) with RBCs present,
24 which is a consequence of the cerebral hemorrhage caused by necrosis of
25 infected brain tissue. The remainder of the CSF analysis is typical of a viral
meningitis (increased lymphocytes, normal to slightly increased protein, and
26
normal to slightly low glucose). The diagnosis is confirmed with polymerase
. 27 chain reaction (PCR) for HSV. Treatment with acyclovir should begin
28 immediately, even before obtaining CSF PCR results. If treatment is delayed,
permanent neurologic sequelae can occur; if untreated, herpes encephalitis
29
has a mortality rate > 70%.
30 Lumbar puncture Aciclovir Cerebrospinal fluid Central nervous system Herpes simplex virus Meningitis
• 31 Polymerase chain reaction Cerebral hemorrhage Encephalitis Viral meningitis Protein Lymphocyte Sequela Image copyright© Dodd eta/.;
• 32 Neurological disorder Mortality rate Glucose Nervous system Necrosis Herpes simplex Neurology Virus licensee BioMed Central. 2015
• 33
Epileptic seizure Bleeding Human brain Infection Herpesviral encephalitis Frontal lobe Brain Lumbar
-3 4
A is not correct. 10°/o chose this.
-35
Antibiotic therapy for gram- negative organ isms wou ld be warranted if gram- negative bacterial meningitis (most
. 36
common in the newborn period) were suspected; the lumbar puncture findings are inconsistent with a bacterial
. 37 process.
. 38 Lumbar puncture Gram-negative bacteria Antibiotics Meningitis Bacterial meningitis

a
Lock
s
Suspend
8
End Bl ock
Item: 30 of 38 ~. I • M k <:] t> al ~· ~
QIO: 4690 .l. ar Previous Next lab 'lifllues Notes Calculator


18 A is not correct. 10% chose this.
19 Antibiotic th erapy for gram -negative organ isms wou ld be wa r ranted if gram- negative bacterial mening it is (most
20
common in the newborn period) were suspected; the lumba r punct ure find ings are inconsistent wit h a bacter ia l
process.
21 lumbar puncture Gram-negative bacteria Antibiotics Meningitis Bacterial meningitis
22
B is not correct. 10% chose this.
23
Antibiotic therapy for gram -positive organisms would be warrant ed if gram-posit ive bacterial mening it is ( eg,
24
Streptococcus pneumoniae) were suspected; the lumba r puncture find ings are inconsistent wit h a bacter ia l
25 process.
lumbar puncture Streptococcus pneumoniae Meningitis Antibiotics Gram-positive bacteria Bacterial meningitis Streptococcus
26
• 27 C is not correct. 11% chose this .
28 Antifungal therapy wou ld be war ranted if funga l men ingitis were suspected; t he lumbar punctu re findings are
29 inconsistent with a funga l process .
lumbar puncture Meningitis Antifungal New England Compounding Center meningitis outbreak Fungal meningitis Fungus
30
• 31 D is not correct. 8°/o chose this .
• 32 Antiretrovi ral t herapy wou ld be wa r ranted to treat an under lying HIV infect ion (immunocompromised patient s
are more susceptible to deve loping her pes encepha lit is); however, t he init ial t herapy shou ld treat the patient's
• 33
acut e symptoms.
o3 4 Immunodeficiency HIV Encephalitis Herpes simplex Management of HIV/AIDS Herpesviral encephalitis Infection
o35
E is not correct. 6°/o chose this .
• 36
Antitubercu lous therapy would be warrant ed if tubercu lous men ing it is were suspect ed; the lumbar puncture
• 37 findings are inconsist ent with a tubercu lous process .
• 38 lumbar puncture Tuberculous meningitis Meningitis Tuberculosis lumbar
~

6
lock
s
Suspend
0
End Block
Item: 30 of 38 ~. I • M k <:] t> al ~· ~
QIO: 4690 .l. ar Previous Next lab 'lifllues Notes Calculator

• lumbar puncture Streptococcus pneumoniae Meningitis Antibiotics Gram-positive bacteria Bacterial meningitis Streptococcus
18
19 C is not correct. 11% chose this.
20 Ant ifungal therapy wou ld be war ranted if fungal men ingitis were suspected; t he lumbar punctu re findings are
21 inconsistent with a funga l process .
lumbar puncture Meningitis Antifungal New England Compounding Center meningitis outbreak Fungal meningitis Fungus
22
23 D is not correct. 8°/o chose this.
24 Antiretrovi ral t herapy wou ld be wa r ranted to treat an under ly ing HIV infect ion (immunocompromised patient s
are more susceptible to deve loping her pes encepha lit is); however, t he init ial t herapy shou ld treat the patient's
25
acut e symptoms.
26 Immunodeficiency HIV Encephalitis Herpes simplex Management of HIV/AIDS Herpesviral encephalitis Infection
• 27
E is not correct. 6°/o chose this.
28
Antitubercu lous therapy would be warrant ed if tubercu lous men ing it is were suspect ed; the lumbar puncture
29 findings are inconsist ent with a tubercu lous process .
30 lumbar puncture Tuberculous meningitis Meningitis Tuberculosis lumbar

• 31
• 32
Bottom Line:
• 33
Cerebrospina l fluid in herpes encephal it is typical ly shows RBCs, increased lymphocytes, nor ma l t o slightly
o3 4 increased prot ein, and norma l to slightly low glucose. The diagnosis is confi r med with polymerase chain
o35 reaction for the her pes simp lex v irus. Treatment wit h acyclovi r shou ld beg in immediately .
Aciclovir Cerebrospinal fluid Polymerase chain reaction Herpes simplex virus Encephalitis Herpes simplex Protein Herpesviral encephalitis lymphocyte Virus
• 36
Glucose Polymerase
• 37
• 38 ~

6
lock
s
Suspend
0
End Block
Item: 30 of 3 8 ~ 1 • M k -<:J 1>- Jil ~· !:';-~
QIO: 4690 ..L ar Pre v ious Next Lab fli!ltues Notes Calcula t o r
A
FIRST AID FACTS
. . - A

18
19
FA17 p 176.2
20
CSF findings in meningitis
21 OPENING PRESSURE CELL TYPE PROTEIN GLUCOSE
22 Bacterial t tP I s t
23 t t
Fungai/TB t lymphoc} tes
24
Viral l\ormal/t t lymphocytes '\lormal/t 'ormal
25
26
FA17 p 197.3
• 27
Acyclovir, fam ciclovir, valacyclovir
28
MECHANISM Guanosine analogs. Monophosphorylated by li S IVZV thymidine kinase and not phosphor} h1ted
29 in uninfected cells -+ few adverse effects. Triphosphate formed by cellular enzymes. Preferentially
30 inhibit viral DNA polymerase by chain termination.
• 31 (LINICALUSE HSV and ZV. Weak activity against EBV. No activity against CMV. Used for HSV-
• 32 induced mucocutaneous and genital lesions as well as for encephalitis. Prophylaxis in
• 33 immunocompromised patients. No effect on lat·ent forms of I-I SV and VZV. alacyclovir, a
prodrug of acyclovir, has better oral bioava ilabi lity.
0 34
For herpes zoster, use famcic lovir.
0 35
ADVERSE EFFECTS Obstructive crystalline nephropathy and acute renal failure if not adequately hydrated .
• 36
MECHANISM OF RESISTANCE Mutated viral thymidine kinase.
• 37
• 38

a
Lock
s
Suspend
8
End Bl ock
Item: 31 of 3 8 ~ 1 • M k -<:J 1>- Jil ~· !:';-~
QIO: 5194 ..L ar Pre v ious Next Labfli!llues Not es Calcula t o r

IAA]
A A

18
A 32-year-old homeless man w ith a long -standing history of alcohol ism undergoes an elective
19 cholecystectomy in the setting of gal lstone disease. On postoperative day 2, his nurse notes that he has
20 become increasingly agitated. Physical examination revea ls a tremulous, disoriented man. His vital signs are
21
temperatu re of 37.8 °C {100°F), pulse of 120/min, and blood pressure of 175/110 mm Hg .

22
Which of the following medications would treat his most likely condition?
23
24 :
A. Captopril
25
26 B. Chlordiazepoxide
. 27 C. Disulfiram
28
D. Epinephrine
29
30
E. Methadone
• 31 F. Warfarin
• 32
• 33
0 34
0 35
. 36
. 37
. 38

a
Lock
s
Suspend
8
End Bl ock
Item: 31 of 3 8 ~ 1 • M k -<:J 1>- Jil ~· !:';-~
QIO: 5194 ..L ar Pre v ious Next Labfli!llues Not es Calcula t o r

IAA]
A A

18
A 32-year-old homeless man w ith a long -standing history of alcohol ism undergoes an elective
19 cholecystectomy in the setting of gal lstone disease. On postoperative day 2, his nurse notes that he has
20 become increasingly agitated. Physical examination revea ls a tremulous, disoriented man. His vital signs are
21
temperatu re of 37.8 °C {100°F), pulse of 120/min, and blood pressure of 175/110 mm Hg .

22
Which of the following medications would treat his most likely condition?
23
24 :
A. Captopril
25
26 B. Chlordiazepoxide
. 27 C. Disulfiram
28
D. Epinephrine
29
30
E. Methadone
• 31 F. Warfarin
• 32
• 33
0 34
0 35
. 36
. 37
. 38

a
Lock
s
Suspend
8
End Bl ock
Item:31of38 ~. , . M k <:] t> al ~· ~
QIO: 5194 .l. ar Previous Next Lab 'lifllues Notes Calculator


18
19 The correct answer is B. 56°/o chose this.
20 This man has undergone withdrawal from alcohol during his hospital ization and is beginn ing to manifest signs
21 and symptoms of del irium t remens, a life-threatening syndrome character ized by disorientation, agitation,
22
hallucinations, and autonomic instability, peaking 2-5 days after the patient's last drink. It is prevented and
treated with benzod iazepines. Chlordiazepoxide is the most common benzod iazepine used in preventing and
23 treating de lir ium tremens due to its long ha lf-l ife and active metabol ites, leading to decreased peaks/troughs
24 and fewer withdrawal symptoms . In case of liver dysfunction, lorazepam can be used . The management of the
25 de lir ium tremens shou ld also include hospita l admission, intravenous hydration, monitor ing of vita l signs,
benzodiazepines, thiamine/fo lic acid/mu lt ivitamins, and labs (complete blood cel l count, chemistry, liver
26
function tests, and coagu lation profile) . Other important preventive measures to take in treating alcoholism
• 27 include tubercu losis testing as wel l as pneumococcal and influenza vaccination .
28 Benzodiazepine lorazepam Delirium tremens Chlordiazepoxide Tuberculosis liver function tests Complete blood count Half-life Delirium Dysautonomia

29 Hallucination Intravenous therapy Alcoholic beverage liver Alcoholism Influenza Alcohol Coagulation Active metabolite Alcohol withdrawal syndrome

30 Autonomic nervous system Disorientation Benzodiazepine withdrawal syndrome

31
A is not correct. 8°/o chose this .
• 32 Captopril is an angiotensin-converting enzyme inh ibitor. It can be used to decrease blood pressure in patients
• 33 with essential hypertension. The drug reduces levels of angiotensin II, prevents inactivation of bradykinin (a
o3 4 potent vasodi lator ), and results in increased release of ren in. It is common ly used in patients with congestive
heart failure and diabetic renal disease. Although th is patient does have an elevated blood pressure, th is answer
o35
choice is not ideal, because his elevated blood pressu re is like ly part of delirium tremens .
• 36 ACE inhibitor Bradykinin Captopril Angiotensin-converting enzyme Vasodilation Angiotensin Enzyme Enzyme inhibitor Hypertension Angiotensin II

• 37 Essential hypertension Renin Heart failure Congestive heart failure Blood pressure Kidney disease Diabetes mellitus Delirium

• 38 ~ ,... :_ --a.. ------&. 4 ~n/ - L - - - a..L:-

6
lock
s
Suspend
0
End Block
Item:31of38 ~. , . M k <:] t> al ~· ~
QIO: 5194 .l. ar Previous Next Lab 'lifllues Notes Calculator

18 C is not correct . 16% chose this.


19 Disulfiram inhibits acetaldehyde dehyd rogenase and is used to help patients abstain from alcoho l abuse. It
20 wou ld not be the appropriate management for this man with delirium tremens.
Delirium tremens Disulfiram Acetaldehyde Acetaldehyde dehydrogenase Delirium Alcoholic beverage Alcohol Alcohol abuse
21
22 D is not correct. 3 °/o chose this.
23 Epinephrine is an a- and ~ - receptor agon ist. It may be used emergently in t reated patients experiencing
anaphylactic shock or as a treatment for open -ang le glaucoma because of its ability to decrease aqueous humor
24
secretion due to vasoconstriction .
25 Epinephrine Glaucoma Anaphylaxis Vasoconstriction Aqueous humour Anaphylactic shock Agonist Aqueous solution Secretion

26
E is not correct. 16% chose this .
• 27
Methadone is a drug used in the management of opioid addiction . Signs of opioid withdrawal include anxiety,
28 insomn ia, diaphoresis, dilated pupils, rh inor rhea, lacr imation, and piloerection . Although opioid withdrawa l is
29 possible in this patient, alcohol withdrawa l is much more likely given his signs and symptoms.
Goose bumps Rhinorrhea Tears Methadone Perspiration Insomnia Alcohol withdrawal syndrome Opioid Drug withdrawal Anxiety Mydriasis Opioid withdrawal
30
Opioid use disorder Alcoholic beverage Alcohol Addiction Pupillary response Substance dependence
31
• 32 F is not correct. 1 °/o chose this.
• 33 Warfarin is an anticoagu lant ind icated in the long-term management of pu lmona ry embol ism . Postoperative
o34 patients are at increased risk of developing pu lmona ry embol i, but alcoho l withd rawal is more likely in th is
man's presentation . A pulmonary embo lism might be suggested by tachyca rdia, dyspnea, tachypnea, pleurit ic
o35
chest pa in, and evidence of deep vein thrombosis.
• 36 Deep vein thrombosis Anticoagulant Warfarin Dyspnea Pulmonary embolism Tachypnea Tachycardia Pleurisy Thrombosis Embolism Chest pain

• 37 Alcohol withdrawal syndrome Alcohol

• 38

6
lock
s
Suspend
0
End Block
Item:31of38 ~. , . M k <:] t> al ~· ~
QIO: 5194

18
.l. ar Previous
. ~
Next Lab 'lifllues
. .
Notes Calculator

anaphylactic shock or as a treatment for open-ang le glaucoma because of its abi lity to decrease aqueous humor
.
19 secretion due to vasoconstriction.
20 Epinephrine Glaucoma Anaphylaxis Vasoconstriction Aqueous humour Anaphylactic shock Agonist Aqueous solution Secretion

21 E is not correct. 16% chose this.


22 Methadone is a drug used in the management of opioid addiction . Signs of opioid withdrawal include anxiety,
23 insomn ia, diaphoresis, dilated pupi ls, rh inor rhea, lacr imation, and piloerection . Although opioid withdrawa l is
24 possible in this patient, alcohol withdrawa l is much more likely given his signs and symptoms.
Goose bumps Rhinorrhea Tears Methadone Perspiration Insomnia Alcohol withdrawal syndrome Opioid Drug withdrawal Anxiety Mydriasis Opioid withdrawal
25
Opioid use disorder Alcoholic beverage Alcohol Addiction Pupillary response Substance dependence
26
• 27 F is not correct. 1 °/o chose this .
28 Warfarin is an anticoagu lant ind icated in the long -term management of pu lmona ry embol ism . Postoperative
patients are at increased risk of developing pulmona ry embol i, but alcoho l withd rawal is more likely in this
29
man's presentation . A pulmonary embo lism might be suggested by tachyca rdia, dyspnea, tachypnea, pleuritic
30 chest pa in, and evidence of deep vein thrombosis.
31 Deep vein thrombosis Anticoagulant Warfarin Dyspnea Pulmonary embolism Tachypnea Tachycardia Pleurisy Thrombosis Embolism Chest pain

• 32 Alcohol withdrawal syndrome Alcohol

• 33
o34
Bottom Line:
o35
Delirium tremens is a life-th reatening syndrome resu lt ing from alcoho l withd rawal that is manifested by
• 36 agitation, disorientation, hal lucinations, and autonomic instability peaking 2-5 days after a patient's last drink.
• 37 Delirium tremens Alcohol withdrawal syndrome Dysautonomia Delirium Autonomic nervous system Hallucination Alcohol Alcoholic beverage Disorientation

• 38 ~

6
lock
s
Suspend
0
End Block
Item: 31 of 3 8 ~ 1 • M k -<:J 1>- Jil ~· !:';-~
QIO: 5194 ..L ar Pre v ious Next Labfli!llues Not es Calcula t o r
A

18 A lal;fii!11•J for year:


FIRST AID FAC T S
2017 •
19
20
FA17 p 542.1
21
Delirium tremens Life-threatening alcohol'' ithdra\\ al S) ndrome that peaks 2- 4 days after last drink.
22
C haracterized by autonomic hyperactivity (eg, tachycardia, tremors, anxiety, seizures), electrolyte
23
disturbances, respiratory alkalosis. Classically occurs in hospital setting (eg, 2-4 days postsurgery)
24 in alcoholics not able to drinl.. as inpatients. Treatment: benzodiazepines (eg, chlordiazepoxide,
25 lorazepam, diazepam).
26
• 27 FA17 p 541 .2
28 Alcoholism Physiologic tolerance and dependence on alcohol with symptoms of withdrawal when intake is
29 interrupted.
30 Complications: alcoholic cirrhosis, hepatitis, pancreatitis, peripheral neuropathy, testicular atrophy.
31 Treatment: disulfi ram (to condition the patient to abstai n from alcohol use), acamprosate,
naltrexone, supportive care. Support gro11ps such as Alcoholics Anonymous are helpful in
• 32
sustaining abstinence and supporting patient and fami ly.
• 33
Wernicke-Korsakoff Caused by vitamin B1 deficiency. Triad of conrusion, ophthalmoplegia, ataxia (Wernicke
0 34
syndrome encephalopathy). May progress to irreversible memory loss, confabulation, personality change
0 35 (Korsakoff syndrome). Symptoms may be precipitated by giving dextrose before administeri ng
• 36 ,·itamin B1 to a patient with thiamine deficiency. Associated with peri,·entricular hemorrhage/
• 37 necrosis of mammillary bodies. Treatment : I vitamin B1•
• 38

a
Lock
s
Suspend
8
End Bl ock
Item: 32 of 3 8 ~ 1 • M k -<:J 1>- Jil ~· !:';-~
QIO: 3549 ..L ar Pre v ious Next Lab fli!ltues Not es Calcula t o r

IAA]
A A

18
A 75-year-old man is brought to the emergency department by ambulance. He has weakness and diminished
19 sensation in his right leg that he says began 1 hour ago. The on-call neurolog ist determines the patient had a
20 stroke. CT of the head shows no bleeding. The patient's fam ily arrives and asks if the patient can receive the
"clot buster" that they heard about on television.
21
22 The patient's vitals are:
23 Temp: 37.5°C
24 Pulse: 70/ min
BP: 190/ 110 mm Hg
25
RR: 14 breaths/ m in
26
The labs include:
• 27 Hg: 12.5
28 WBC: 8,000
29 Platelets: 150,000
INR: 1.1
30
31
• 32 Which of the fol lowing is a contra indication for the patient receiving thrombo lytic therapy for his stroke?
• 33 :
0 34 A . A history of a stroke 2 years ago
0 35 B. A platelet count of 150,000/m m 3
• 36
C. A systol ic blood pressure of 190 mm Hg
• 37
• 38
D . An age of 75 years

a
Lock
s
Suspend
8
End Bl ock
Item: 32 of 38 ~. I • M k <:] t> al ~· ~
QIO: 3549 .l. ar Previous Next lab 'lifllues Notes Calculator


18
19 The correct answer is C. 53°/o chose this.
20 A systol ic blood pressure of 190 mm Hg is a contraind ication for tissue plasminogen activator (tPA) therapy for
21 an acute stroke. The current gu idel ines preclude tPA therapy for a systo lic blood pressure > 185 mm Hg or a
diastol ic blood pressure of > 110 mm Hg. Patients with BP > 185 mm Hg should be t reated with
22
antihypertensives such as labeto lol or nicardipine. If intravenous t reatment does not bring the blood pressu re
23 into the acceptable range, the patient shou ld not be treated with tPA because the r isk of intracerebral
24 hemorr hage with th rombo lytic the rapy may be increased.
Tissue plasminogen activator Cerebral hemorrhage Thrombolysis Blood pressure Systole Intravenous therapy Stroke Bleeding Plasmin Contraindication
25
Antihypertensive drug Thrombolytic drug Diastole
26
• 27 A is not correct. 12% chose this .
28 A history of a stroke 2 years ago will not prevent th is patient from receiving tissue plasminogen activator (tPA).
29 However, a history of stroke or head trauma within the last 3 months befo re the stroke is a contraindication for
tPA the rapy.
30 Tissue plasminogen activator Stroke Plasmin Contraindication Head injury
31
B is not correct. 14% chose this.
32
A platelet count of 150,000/mm3 is not a contraind ication fo r tissue plasminogen activator therapy for acute
• 33
stroke and is actua lly within nor mal limits (barely). However, a platelet count of< 100,000/mm3 is a
o34 contra indication for thrombolytic the rapy.
o35 Tissue plasminogen activator Thrombolysis Thrombolytic drug Contraindication Platelet Plasmin Stroke

• 36 D is not correct. 9°/o chose this.


• 37 The patient's age of 75 yea rs is not a contraind ication for tissue plasminogen activator (tPA) therapy. The on ly
• 38 ~
age restriction fo r tPA to treat acute stroke is an age < 18 years .

6
lock Suspend
s 0
End Block
Item: 32 of 38 ~. I • M k <:] t> al ~· ~
QIO: 3549 .l. ar Previous Next lab 'lifllues Notes Calculator

18 •
contra indication for thrombolytic the rapy.
• • •••••
19 Tissue plasminogen activator Thrombolysis Thrombolytic drug Contraindication Platelet Plasmin Stroke
20
D is not correct. 9°/o chose this.
21
The patient's age of 75 yea rs is not a contraind ication for tissue plasminogen activator (tPA) therapy. The on ly
22 age restriction fo r tPA to treat acute stroke is an age < 18 years.
23 Tissue plasminogen activator Plasmin Contraindication Stroke Plasminogen activator

24 E is not correct. 12% chose this.


25 An I nternational No rmal ized Ratio (I NR) of 1.1 is not a contraindication for t issue plasminogen activator (tPA)
26 the rapy according to current guidelines, and is actual ly in the norma l range (0.8-1.2) . However, anticoagu lation
• 27 at the time of the stroke, an INR > 1. 7, and a prolonged activated partial thromboplastin t ime are
contra indications to tPA the rapy.
28 Tissue plasminogen activator Partial thromboplastin time Prothrombin time Anticoagulant Contraindication Plasmin Thromboplastin Stroke
29
30
31 Bottom Line:
32 Contraindications to thrombolytic therapy include uncontrolled hypertension with a systol ic blood pressure
> 185 mm Hg and diastolic blood pressu re > 110 mm Hg, histo ry of stroke or head t rauma with in the last 3
• 33
months, International Normal ized Ratio > 1. 7 if on oral anticoagulant, and platelet count < 100,000/mm3 . The
o34 most common contraind ication is t ime since onset of symptoms; many institutions are now using 4.5 hours as
o35 the cutoff for giving agents like tPA, but much of the teaching is sti ll 3 hours.
Anticoagulant Thrombolysis Prothrombin time Thrombolytic drug Contraindication Blood pressure Hypertension Platelet Tissue plasminogen activator Systole
• 36
Diastole Stroke Head injury
• 37
• 38 ~

6
lock
s
Suspend
0
End Block
Item: 32 of 3 8 ~ 1 • M k -<:J 1>- Jil ~· !:';-~
QIO: 3549 ..L ar Pre v ious Next Lab fli!ltues Not es Calcula t o r
A A

18
FA17 p 415.2
19
Thrombolytics Alteplase (tPA), reteplase (rP ), streptokinase, tenecleplase (T ' K-tPA).
20
MECHANISM Directly or indirectly aid c01wersion of plasminogen to plasmin, which clea\·es thrombin and fibrin
21
clots. t PT, t P'IT, no change in platelet count.
22
CliNICAL USE Early .\II, early ischemic stroke, direct thrombolysis of severe PE.
23
ADVERSE Eff£CTS Bleeding. Contraindicated in patients with acti\·e bleeding, history of intracranial bleeding,
24
recent surgery, known bleeding diatheses, or SC\ Cre h~ pertension. Nonspecific reversal with
25
antifibrinolr tics (eg, aminocaproic acid, trane, amic acid), platelet transfusions, and factor
26 corrections (eg, cryoprecipitate, F'F'P, PCC).
• 27
28 FA17 p 391 .2
29
Coagulation and kinin pathways
30
Collagen, ................................... HMWK
31
32
basement membrane,
activat~ platelets •••• ••••
•••• K 11·k ·
a 1 rem ----- - ~
1 ..........
j v sod"l
a
·
I ahon

• 33 Contact
act1vat on
.
:
I

... •"
.,"
;
/ Bradykinin •f: ..... . . .,. i Permeabmty
....
....
........
0 34 mtnns1c XII - Xlla ... i Pain

35 pathway XI J:....xla
0

• 36 IX _l.IXa

• 37
nssue factor
~
1• -
VIII
withvWF ANTICOAGULANTS: lla (thrombin)
Ttssue factc VII- - -.- - - + VIIa ___::::::......
*- - -,
• 38 - heparin (greatest efficacy)
lextnns1c) • I u wu , ... ~........~... A~...- ..,;...\
--~----------~

a
Lock
s
Suspend
8
End Bl ock
Item: 32 of 3 8 ~ 1 • Ma rk -<:J I> ~ £!1}>'
• !!":-~
QIO: 3549 ..L Prev ious Next Lab lues Not es Cal culat o r

A A

18 FA17 p 391 .2

19 Coagulation and kinin pathways


20
21
Collagen,
basement membrane,
activated platelets •••••
.
,/
••••••••••••••••••••••••••••••••••· HMWK

...
Kaltik
rein ·-····•
1 , ..••·'"
j Vasod'laf
1 1on
22
·E::: .....• i Permeabilrty
23
a
.
:
I

~ ~
~
#
./ BrclAul.inm
~R ~~
' ~.
•• i Pa•n
xu - xua
24
XI ...!... XIa
25
26
• 27
Tissue factor
"-......_
IX _!.IXa
1
• -
VIII
with vWF ANTICOAGULANTS: lla (thrombin)
TISSUe factc VII - - -., ----+ VIla _.....:::::::...- - -,
(extnns1c) • heparin (greatest ethcacyl
28
X,.--.... Xa • LMWH (dalteparin. eno.xaparinl
pathway
• d1rect thrombin inhibitors (argatroban.
29
ANTICOAGULANTS: factor Xa
- j• - v bivalirudin. dabigatran)
30
• LMWH (greatest efficacy) II ,.--.... lla " / '
31 · heparin Prothrombin Thrombin
Plasminogen
· direct Xa inhibitors (apixaban. rivaroxaban) ~
32 • fondaparinux
,.--.... •.THROMBOLYTICS:
s · alteplase, reteplase.
_j Ia
• 33 tPA ~ streptokinase. tenecteplase

~Aminocaproic aad
Fibrinogen Fibrin monomers
-34
-35
HemophiliaA: deficiency of factor VIII (XRJ
Hemophilia 8: dehciency of factor IX(XR) Combmed
? Plasmin
••
Hemophilia C: dehciency of factor XI (AR) ••
• 36 path ay ? XIIIa - -XIII •
'------- •••
• 37 Note. Kallikrein aCtivates bradyk1nin; ACE inactivates bradylonin •
••
• reqUire Ca1• • phospholipid _ _ _ _ __;~:....__ _ _ _ Fibnn degradation
• 38 orodurt~

a
Lock
s
Suspend
8
End Bl ock
..·..
18 activated platelets
,., i Vasodilation
••
Bradykinin ·•(·····~» i Permeabitity
19 Contact
actrvaton
''
''
....
20
nsc)
xu ....!......xua ·· ... i Pain
21 path y xt ...!...xta
22
23
nssuef~t
Tissue factor
"-
VII --~--+ VIla -""'---.....:::::.._ _
IX _l.IXa

_
1 • -
VIII
withvWF ANTICOAGULANTS: lla (thrombin)
24 (extr nSic) ~ * ~ • hepann (greatest efhc.acyl
pathway X ~ Xa • l.MWH (daltepann. enoxaparinl
25
- 1•- v • d~rect thrombin inhibitors (argatroban.
bivabrudin. dabigatran)

l ~lla ~
26
ANTICOAGULANTS: factor xa
. 27 - lMWH (greatest efficacy)
• hepann Prothrombin Thrombin
Plasm1nogen
28 • direct xa inhibitors (apixaban. rivaroxaban) ~ ••THROMBOLYTICS:
29
• fondapannux
~-
Ia tPA 10!
+ • alteplase. reteplase.
streptokinase. tenecteplase
F1bnnogen Fibrin monomers
30
31 HemophiliaA: deficiency of factor VIII (XR)
Hemophilia 8: deficiency of factor IX (XR) Combined
? Plasmin

Aminocaproic aod

1L m.1l) ~ .ystem
32 ''
Hemophilia C: deficiency of factor XI (AR) pathway ?XIIIa - -XIII '
----- ••'
• 33 •
Note. Kallikrein activates bradykinin; ACE inactivates bradykinin ••

o34 • require Ca". phospholipid _ _ _ _ ___:.T_ ____ Fibrin degradation
= 1nhibited by vitamin Kantagonist warfarin products
o35
• =cofactor Fibrin mesh stabilizes
. 36 ..... = activates but not part or coagulation cascade platelet plug
. 37 l.MWH. low-molecular-weight heparin

. 38

a
Lock
s
Suspend
8
End Block
Item: 32 of 3 8 ~ 1 • Ma rk -<:J I> ~ £!1}>'
• !!":-~
QIO: 3549 ..L Prev ious Next Lab lues Not es Cal culat o r
I . t t y • • g t t t I
18 • = cofactOI' Fibrin mesh stabilizes
19 ···-- =activates but not part of coagulation cascade platelet plug
l.MWH. low-molecular-weight heparin
20
21
22 FA17p392.1

23
Coagulation cascade components

24
25
26
Procoagulation

epoode
OXIdized r~UCl.J~
vitamin K - - --
reduced
vitamin K - - --
(acts as
cofactor)
c: inactive II, VII, IX. X. C. S
~Umyiatl>o•Y' ..
Warfarin inhibits the enz,·me vitamin K
epoxide reductase.
'eonates lack enteric bacteria, which produce
vitamin K.
. 27 mature (active) II, VII, IX. X. C. S Vitamin K deficiency: l S)11thesis of factors II ,
28 VI I, IX, X, protein C, proteinS.
29 '\\ F carries/protects \'Ill (, oJks\\'agen
Factories make gr8 (great) cars.
30
31
Anticoagulation Antithrombin inhibits activated forms of factors
thrombin thrombomodulln complex II, VI l, IX, X, XI, XII.
32 (endothelial cells) proteinS Heparin enhances the activity of antithrombin.
• 33 Protein C - - - -- -- activated protein C - cleaves and inactivates Va. VIlla
Principal targets of antithrombin: thrombin and
-34 factor , a.
tPA
-35 Plasminogen - - plasmin - - fibrinolysis: Factor Leiden mutation produces a factor V
1. cleavage of fibrin mesh resistant to inhibition by acti,·ated protein C.
. 36 2. destruction of coagulation factoo
tPA is used clinicallr as a thrombolytic.
. 37
. 38

a
Lock
s
Suspend
8
End Bl ock
Item: 33 of 38 ~ 1 • M k -<:J 1>- Jil ~· !:';-~
QIO: 2488 ..L ar Pre v ious Next Lab fli!ltues Notes Calcula t o r

IAA]
A A

18
A patient is being prepared for a surg ical procedure. He is confused, tachycardic, tachypneic, and
19 hypertensive. His skin is dry, flushed, and warm to the touch, and his pupi ls are dilated and unresponsive to
20 light. The anesthesiologist immediately administers a dose of physostigmine, and the symptoms quickly
resolve.
21
22
What is the mechanism of the preoperative drug that most likely caused this patient's symptoms?
23
24 4 2 8835 :
A. a 1 -Receptor agonist
25
26 B. 132 -Receptor antagonist
. 27 C. Muscle-type nicotinic receptor antagonist
28
D. Nonselective muscarinic receptor antagonist
29
30
E. Selective M2 receptor antagonist

31
32
• 33
0 34
0 35
. 36
. 37
. 38

a
Lock
s
Suspend
8
End Bl ock
Item: 33 of 38 ~. I • M k <:] t> al ~· ~
QIO: 2488 .l. ar Previous Next lab 'lifllues Notes Calculator


18
19 The correct answer is D. 68°/o chose this.
20 This patient is experiencing atropine overdose. Atropine is a nonselective muscarinic antagonist used in surgica l
procedures to suppress bronchiolar secretions, and postsurgica lly to prevent the muscarinic effects of
21
acetylchol inesterase inhibitors used to reverse muscle paralysis at the end of a procedure . Physostigmine
22 antagon izes the action of anticholinergics, which block the postsynaptic receptor sites of acetylchol ine.
23 Physostigmine reversibly inhibits the destruction of acetylchol ine by acetylchol inesterase, thereby increasing the
24
concentration of acetylcho line at sites of cholinergic transmission and counteracting the effects of receptor
antagon ists.
25 Physostigmine Atropine Muscarinic antagonist Acetylcholinesterase Acetylcholine Cholinergic Anticholinergic Receptor antagonist Muscarinic acetylcholine receptor
26 Acetylcholinesterase inhibitor Chemical synapse Drug overdose Paralysis Neurotransmitter receptor Receptor (biochemistry)
• 27
A is not correct. 8°/o chose this.
28
An overdose of an a 1 -receptor agonist such as phenyleph r ine wi ll cause severe hypertension, and can provoke
29
seizures in epileptics.
30 Phenylephrine Epilepsy Hypertension Epileptic seizure Agonist Drug overdose

31
B is not correct. 4°/o chose this.
32
An overdose of a ~rrecepto r agonist such as albuterol can cause tremor, palpitations, and hypo kalemia,
33 resu lt ing in arrhythmias.
o34 Hypokalemia Palpitations Tremor Agonist Salbutamol Cardiac arrhythmia Drug overdose

o35 C is not correct. 13% chose this.


• 36 An overdose of a muscle-type nicotinic receptor antagonist such as atracurium wi ll cause progressive paralysis
• 37 resu lt ing in hemodynamic instabi lity .
Nicotinic antagonist Atracurium besilate Nicotinic acetylcholine receptor Receptor antagonist Receptor (biochemistry) Paralysis Drug overdose
• 38 ~

6
lock
s
Suspend
0
End Block
Item: 33 of 38 ~. I • M k <:] t> al ~· ~
QIO: 2488 .l. ar Previous Next lab 'lifllues Notes Calculator

• An overdose of an a 1-receptor agonist such as phenyleph r ine wi ll cause severe hypertension, and can provoke •
18
seizures in epileptics.
19
Phenylephrine Epilepsy Hypertension Epileptic seizure Agonist Drug overdose
20
21
B is not correct. 4°/o chose this.
An overdose of a ~rrecepto r agonist such as albuterol can cause tremor, palpitations, and hypoka lemia,
22
resu lting in arrhythmias.
23 Hypokalemia Palpitations Tremor Agonist Salbutamol Cardiac arrhythmia Drug overdose
24
C is not correct. 13% chose this.
25
An overdose of a muscle-type nicotinic receptor antagonist such as atracurium wi ll cause progressive paralysis
26 resu lting in hemodynamic instability. t
• 27 Nicotinic antagonist Atracurium besilate Nicotinic acetylcholine receptor Receptor antagonist Receptor (biochemistry) Paralysis Drug overdose

28 E is not correct. 7°/o chose this.


29 Atropine is nonselective muscarinic antagonist and does not inh ibit M2 alone.
30 Atropine Muscarinic antagonist Receptor antagonist Muscarinic acetylcholine receptor Antagonist

31
32
Bottom Line:
33
An overdose of a muscarinic receptor antagonist such as atropine can cause adverse effects such as dry
• 34
mouth, mental status changes, pupillary dilation, and tachycard ia. An acetylcholinesterase inhibitor such as
o35 physostigmine can reverse these effects .
Acetylcholinesterase inhibitor Physostigmine Atropine Muscarinic acetylcholine receptor Tachycardia Muscarinic antagonist Acetylcholinesterase Pupillary response
• 36
Mydriasis Xerostomia Receptor antagonist Drug overdose Receptor (biochemistry) Enzyme inhibitor
• 37
• 38 • •

6
lock
s
Suspend
0
End Block
Item: 33 of 38 ~. I • M k <:] t> al ~· ~
QIO: 2488 .l. ar Previous Next lab 'lifllues Notes Calculator

6
lock
s
Suspend
0
End Block
Item: 33 of 38 ~ 1 • M k -<:J 1>- Jil ~· !:';-~
QIO: 2488 ..L ar Pre v ious Next Lab fli!ltues Notes Calcula t o r
A A

18
FA17 p232.1
19
Cholinomimetic agents
20 DRUG ACTION APPLICATIONS
21 Direct agonists
22 Bethanechol Activates bowel and bladder smooth mu~cle; Postoperative ileus, neurogenic ileus, urinar)
23 resistant to AChE, no nicotinic acti,·ity. retention
24 "Bethany, call (bethanechol) m e to acti\'atc
25 rour bowels and bladder."

26 Carbachol Carbon copy of acetylcholine (but resistant to Constricts pupil and relie,•es intraocular
AChE). pressure in open-angle glaucoma
• 27
Methacholine Stimulates m uscarinic receptors in airway when Challenge test for diagnosis of asthma
28
inhaled.
29
Pilocarpine Contracts ci liary muscle of eye (open-:mgle Potent stimulator of sweat, tears, and saliva
30
glaucoma), pupillary sphincter (closed-a ngle Open-angle and closed-angle glaucoma,
31
glaucoma); resistant to AChE, can cross blood- xerostomia (Sjogren syndrome)
32 brain barrier (tertiary amine). "You cry, drool,
33 an d swea t on your ' p•'Iow. '"
0 34 Indirect agonists (anticholinesterases)
0 35 Galantamine, t ACh. Al:rhei mer disease (AI7heimer patients gallantly
• 36 donepezil, swim down the ri,er)
• 37 rivastigmine
• 38 Edrophonium t ACh. Historically used to diagnose myasthenia gra,·is;

a
Lock
s
Suspend
8
End Bl ock
Item: 33 of 38 ~ 1 • M k -<:J 1>- Jil ~· !:';-~
QIO: 2488 ..L ar Pre v ious Next Lab fli!ltues Notes Calcula t o r
A A

18 Indirect agonists (anticholinesterases)


19 Galant amine, t ACh. Al:rhei mer disease (AI7heimer patients gallantly
20 d onep ezil, swim down the ri' er)
rivastigmine
21
22
Edrophon ium t ACh. Historically used to diagnose myasthenia gra,·is;
replaced by anti-.\ ChR Ab (anti-acetylcholine
23
receptor antibody) test.
24
Neostigmi ne t ACh. Postoperative and neurogenic ileus and
25
r\eo C ' S = :\o Cl'\S penetration (quaternarr urinary retention, myasthenia gravis,
26 amine). reversal of neuromuscular junction blockade
• 27 (postoperative).
28 Physostigmine t Ch. Ph~ sos tigmine "ph) \es" atropine Antidote for anticholinergic toxicity; phrcely
29 overdose. (freely) crosses blood-brain barrier - C lS
30
(tertiary amine).
31
Pyridostigmine t ACh; t muscle strength. Pyridostigm inc gets Myasthenia gravis (long acting); does not
rid of myasthenia gravis. penetrate C S (quaternary ami ne).
32
ole: With all cholinom imetic agents, watch for exacerbat ion of COPD, asthma, and peptic ulcers when giving to susceptible
33
patients.
0 34
0 35
FA17 p 232.2
• 36
• 37
Cholinesterase Often due to organophosphates, such as DU \ 1BBELSS.
inhibitor poisoning parathion, that irreversibly inhibit ACh F:. Organophosphates are often components of
• 38
r--··--- n=---•--- r '-=··-•=-.. '•=--=- : ............... : .... :.J ........ .._ .... : ....... .... :._,. .......... 11. ............ ., : ... ( ........... , .. __

a
Lock Suspend
s 8
End Bl ock
Item: 33 of 38 ~ 1 • M k -<:J 1>- Jil ~· !:';-~
QIO: 2488 ..L ar Pre v ious Next Lab fli!ltues Notes Calcula t o r
I • " I
18
receptor antibody) test.
19
Neostigmine t ACh. Postoperative and neurogenic ileus and
20 !\eo C ·s
= No CNS penetration (quaternary urinary retention, myasthenia gravis,
21 amine). re\ersal of neuromuscular junction blockade
22 (postoperative).
23 Physostigmine t ACh. Ph~ sostigmine "ph) \es" atropine Antidote for anticholinergic toxicity; plu eely
24 overdose. (freely) crosses blood-brain barrier - C1'\S
(tertiary amine).
25
Pyridostigmine t ACh; t muscle strength. Pyridostigmine gets 1\ lyasthenia gra' is (long acting); does not
26
rid of myasthenia gra\'iS . penetrate CNS (quaternary amine).
• 27
1 ole: With all cholinomimetic agents, watch for exacerbation of COPD, asthma, and peptic ulcers when giving to susceptible
28
patients.
29
30
FA17 p 232.2
31
Cholinesterase Often clue to organophosphates, such as DUMBB ELSS.
32
inhibitor poisoning parathion, that irreversibly inhibit AChF.. Organophosphates are often components of
33 Causes Diarrhea, Urinat ion, \ll iosis, insecticides; poisoning usually seen in farmers.
0 34 Bronchospasm, Bradrcardia, Excitation Antidote-atropine (competitive inhibitor) +
0 35 of skeletal muscle and C1 S, Lacrimal ion, pralidoxime (regenerates AChE if given early).
• 36 Sweating, and SaJi,·ation. ~11 ay lead to
respiratory failure if untreated .
• 37
• 38

a
Lock
s
Suspend
8
End Bl ock
Item: 34 of 3 8 ~ 1 • M k -<:J 1>- Jil ~· !:';-~
QIO: 1453 ..L ar Pre v ious Next Lab fli!ltues Notes Calcula t o r

IAA]
A A

18
A 46-year-old woman has not achieved adequate relief from depression fol lowing a 6-w eek trial of sertraline.
19 A subsequent 6-week tria l of f luoxetine was likewise unsuccessfu l. Her physician discontinues this medication
20 and starts her on a new medication 1 week later. A few hours after taking the fi rst dose, she beg ins to have
21
muscle spasms, high fever, and changes in mental status. During the physical examination, brisk, passive
dorsiflex ion of each ankle produces sustained clonus.
22
23
Wh ich of the following agents was she most likely prescribed after discontinuing fluoxetine?
24
:
25
A . Diazepam
26
. 27
B. Mirtazapine
28 C. Phenobarbital
29 D. Propranolol
30
E. Tranylcypromine
31
32
33
• 34
• 35
. 36
. 37
. 38

a
Lock
s
Suspend
8
End Bl ock
Item: 34 of 38 ~. I • M k <:] t> al ~· ~
QIO: 1453 .l. ar Previous Next lab 'lifllues Notes Calculator


18
19 The correct answer is E. 51 °/o chose this.
20 This patient is experiencing seroton in synd rome, a conste llation of symptoms that includes hyperpyrexia,
21 muscle rigidity, ca rdiovascu lar collapse, and menta l status changes . Th is syndrome is a wel l-known complication
22
of combining se lective serotonin reuptake inh ibitors (SSRis) with monoamine oxidase inhibitors (MAOis) . MAOis
act by inhibiting the enzyme monoamine oxidase, which is responsbile for breaking down neurotransmitters .
23 There are two different types of monoamine oxidases: (1) MAO-A, wh ich breaks down serotonin,
24 norepinephrine, epineph rine, and melatonin, and (2) MAO-B, which breaks down amines. I t is extremely
25 important to discontinue SSRi s such as fluoxetine at least 2 weeks before starting an MAOI such as
tranylcypromine (th is is known as a "washout" period) to avoid reaching dangerously high concentrations of
26
seroton in from the combination of the medications. MAOis are sometimes used in cases of refractory
• 27 depression, but note that their use is now extremely rare due to side effects and medication interactions.
28 Serotonin syndrome Tranylcypromine Fluoxetine Epinephrine Melatonin Hyperpyrexia Serotonin Monoamine oxidase B Norepinephrine Monoamine oxidase inhibitor

29 Fever Enzyme Monoamine oxidase Selective serotonin reuptake inhibitor Neurotransmitter Reuptake Monoamine oxidase A Monoamine neurotransmitter

30 Treatment-resistant depression Circulatory system Major depressive disorder Shock (circulatory) Hypertonia Side effect Adverse drug reaction Depression (mood)

31 Pharmaceutical drug Circulatory collapse Muscle Amine

32
A is not correct. 9°/o chose this.
33 Diazepam is a benzodiazepine that augments the activity of inhibitory chloride channels and is used to treat
34 anxiety and status epilepticus. It does not affect serotonin levels in the brain and thus wou ld not lead to
o35 seroton in syndrome when combined with a se lective serotonin reuptake inh ibitor (SSRI) .
Benzodiazepine Selective serotonin reuptake inhibitor Serotonin syndrome Diazepam Serotonin Status epilepticus Serotonin reuptake inhibitor Anxiety Reuptake
• 36
Chloride channel Reuptake inhibitor Brain Chloride Enzyme inhibitor
• 37
• 38 ~
B is not correct. 26% chose this .

6
lock
s
Suspend
0
End Block
Item: 34 of 38 ~. I • M k <:] t> al ~· ~
QIO: 1453 .l. ar Previous Next lab 'lifllues Notes Calculator


18 B is not correct. 26% chose this.
19 Mirtazapine is an atypical antidepressant that can be used to t reat patients with depression, particu larly those
20 with symptoms of weight loss or sleeplessness. Mirtazapine is unlikely to cause serotonin syndrome when
21
combined with a selective seroton in reuptake inhibitor (SSRI ).
Selective serotonin reuptake inhibitor Serotonin syndrome Mirtazapine Antidepressant Serotonin Serotonin reuptake inhibitor Reuptake Insomnia Weight loss
22
Major depressive disorder Depression (mood) Reuptake inhibitor Enzyme inhibitor
23
24 C is not correct. 9°/o chose this.
25
Phenobar bita l is a barbiturate prima r ily used to treat seizures by augmenting GABA chlor ide channel function. It
does not affect serotonin levels in the brain and thus wou ld not cause serotonin syndrome when combined with
26 a selective seroton in reuptake inhibitor (SSRI ) .
• 27 Barbiturate Selective serotonin reuptake inhibitor Serotonin syndrome Phenobarbital Serotonin Gamma-Aminobutyric acid Serotonin reuptake inhibitor

28 Chloride channel Epileptic seizure Reuptake Reuptake inhibitor Enzyme inhibitor Chloride Brain

29
D is not correct. 5°/o chose this.
30
Propranolol is a nonse lective ~-b l ocker that can be taken as needed before publ ic speaking or other anxiety-
31 provoking circumstances in nongeneral ized socia l anxiety disorder. I t is not used for the treatment of
32 depression. Additional ly, propranolol antagonizes ~ receptors and has no effect on serotonin levels, so is not
associated with seroton in syndrome.
33
Serotonin syndrome Propranolol Social anxiety disorder Serotonin Anxiety disorder Anxiety Major depressive disorder Depression (mood) Social anxiety
34
Receptor (biochemistry) Binding selectivity
o35
• 36
• 37 Bottom Line:
• 38 ~
MAOis used in combination with SSRis or TCAs can lead to a serious adverse effect called seroton in syndrome. ~

6
lock
s
Suspend
0
End Block
Item: 34 of 38 ~. I • M k <:] t> al ~· ~
QIO: 1453 .l. ar Previous Next lab 'lifllues Notes Calculator

• Major depressive disorder Depression (mood) Reuptake inhibitor Enzyme inhibitor


18
19 C is not correct. 9°/o chose this.
20 Phenobar bita l is a barbiturate prima r ily used to treat seizures by augmenting GABA chlor ide channel function . It
21 does not affect serotonin levels in the brain and thus wou ld not cause serotonin syndrome when combined with
22
a selective serotonin reuptake inhibitor (SSRI ).
Barbiturate Selective serotonin reuptake inhibitor Serotonin syndrome Phenobarbital Serotonin Gamma-Aminobutyric acid Serotonin reuptake inhibitor
23
Chloride channel Epileptic seizure Reuptake Reuptake inhibitor Enzyme inhibitor Chloride Brain
24
25 D is not correct. 5°/o chose this.
26
Propranolol is a nonselective ~ - b l ocker that can be taken as needed before publ ic speaking or other anxiety-
provoking circumstances in nongeneral ized socia l anxiety disorder. I t is not used for the treatment of
• 27
depression. Additionally, propranolol antagonizes ~ receptors and has no effect on serotonin levels, so is not
28 associated with seroton in syndrome.
Serotonin syndrome Propranolol Social anxiety disorder Serotonin Anxiety disorder Anxiety Major depressive disorder Depression (mood) Social anxiety
29
30 Receptor (biochemistry) Binding selectivity

31
32 Bottom Line:
33
MAOis used in combination with SSRi s or TCAs can lead to a serious adverse effect called seroton in syndrome.
34 It is characterized by t remor, hyperreflexia, clonus, hyperthermia, muscle rigidity, and mental status changes.
o35 An SSRI or TCA should be discontinued at least 14 days prior to starting an MAOI and vice versa .
Serotonin syndrome Clonus Hyperreflexia Selective serotonin reuptake inhibitor Serotonin Hyperthermia Monoamine oxidase inhibitor Tremor Hypertonia
• 36
Adverse effect Tricyclic antidepressant Muscle
• 37
• 38 ~

6
lock
s
Suspend
0
End Block
Item: 34 of 3 8 ~ 1 • M k -<:J 1>- Jil ~· !:';-~
QIO: 1453

18
A
..L
. . . ar Pre v ious Next Lab fli!ltues Notes Calcula t o r
A

19 FA17 p 545.3
20
Serotonin syndrome Can occur with any drug that f 5-IIT (eg, 'v1 O ls, SSRis, St Rls, TCAs, tramadol, ondansetron,
21 triptans, linezolid, .\I DJ\ lA, dext romethorphan). Characterized by 3 A's: neuromuscular
22 h) perActi\'ity (clonus, hyperreflexia, h) pertonia, tremor. seizure), Autonomic stimulation
23 (hyperthermia, diaphoresis, diarrhea), and \ gitation. Treatment: cyproheptadine (5-HT1 receptor
antagonist).
24
25
26 FA17 p 546.1

. 27 Monoamine oxidase Tranylcypromine, P henelzine, lsocarboxazid, Sclegilinc (selccti\·e ~1.\0-B inhibitor).


inhibitors (.\l AO Takes P ride In Shanghai).
28
29 MECHANISM t'\onselecti\·e .\tfAO inhibition f le\'els of amine neurotransmitters (norepinephrine, 5-HT,
dopamine).
30
31
CLINICAL USE Atypical depression, an xiety. Parkinson disease (sclegiline).

32 ADVERSE EFFECTS C 1 S stimulation; hypertensive crisis, most notably with ingestion of tyramine, which is found in
many foods such as aged cheese and wine. Tyramine displaces other neurotransmitters (eg, E)
33
into the synaptic cleft- f sympathetic stimulation. Contraindicated with SSRis, TCAs, St. John's
34
wort, meperidine, dextromethorphan (to prevent serotonin srndrome).
o35 Wait 2 weeks after stopping .\tli\0 inhibitors before starting serotonergic drugs or stopping dietary
. 36 restrictions.
. 37
. 38
• CA-47- CA A -,

a
Lock Suspend
s 8
End Bl ock
Item: 34 of 3 8 ~ 1 • Ma r k -<:J I> ~ £!1}>'
• !!":-~
QIO: 1453 ..L Prev ious Next Lab lues Not es Cal culat o r

A A

18 FA17 p 544.3
19
Antidepressants
20
NORADRENERGIC SEROTONERGIC
21
22 AXON AXON
23 ---- --~
--0-
r-----__r-,...---
MAO lnhlbotors
~ "i MAO - - • Metabolites
24 Metaboli tes - - - MAO

25
I Buproplon \
26 NE - -0 00
• 27 (/0
28 0
29 I
30 TCAs, SNRis -o-. ~} Mlrtazaplne TCAs, SSRis,

31 1 NEreupta~ M SNRis, trazodone

32 '--o o:0
33
34
o35
( "'~""·
5-HT receptor

• 36
• 37
• 38 POSTSYNAPTIC NEURON

a
Lock
s
Suspend
8
End Bl ock
Item: 35 of 3 8 ~ 1 • M k -<:J 1>- Jil ~· !:';-~
QIO: 4196 ..L ar Pre v ious Next Labfli!llues Notes Calcula t o r

IAA]
A A

18
A 35-year-old man undergoes a surg ical procedure in which the anesthesiolog ist uses ha lothane for
19 anesthesia and a depolarizing agent for neuromuscular blockade. During th is procedure, the patient develops
20 muscular rigid ity and pyrexia .
21
22 The anesthesiologist injects an antidote that acts through which of the following mechanisms?
23 :
24 A. Acting as a glutathione substitute
25 B. Antagonizing y-aminobutyric acid receptors
26
C. Blocking calcium release
. 27
D. Catalyzing y-carboxylation of glutamic acid residues
28
29 E. I nhibiting acetylcholinesterase
30
31
32
33
34
o35
. 36
. 37
. 38

a
Lock
s
Suspend
8
End Bl ock
Item: 35 of 38 ~. I • M k <:] t> al ~· ~
QIO: 4196 .l. ar Previous Next Lab 'lifllues Notes Calculator


18
19 The correct answer is C. 67°/o chose this.
20 This question refers to a patient who was placed under anesthesia using halothane and succinylcholine, a
depolarizing agent used for neuromuscular blockade. Succinylchol ine acts by mimicking acetylchol ine (ACh) and
21
binds to the nicotinic receptors at the neuromuscular junction. Unlike ACh, wh ich is quickly destroyed,
22 succinylchol ine rema ins bound to the nicotinic receptors and produces small fasciculations, followed by
23 paralysis. This question requires you to recognize that halothane and succinylcholine-when used together or
even alone-increase a patient's likelihood of developing mal ignant hyperthermia . This risk is especial ly
24
increased in those that have a predisposing genetic mutation in the ryanodine receptor gene. Finally, you must
25 understand that dantrolene sodium is the treatment for malignant hyperthermia. I t acts by blocking the release
26 of Ca 2+ from the sarcoplasmic reticulum, ultimately reducing heat production and decreasing muscle tone .
Ryanodine receptor Halothane Dantrolene Malignant hyperthermia Suxamethonium chloride Neuromuscular junction Neuromuscular-blocking drug
• 27
Sarcoplasmic reticulum Acetylcholine Gene Hyperthermia Anesthesia Depolarization Nicotinic acetylcholine receptor Fasciculation Paralysis Muscle tone Ryanodine
28
Mutation Receptor (biochemistry) Cancer Muscle
29
30 A is not correct. 4°/o chose this.
31 N-Acetylcysteine acts as a glutathione substitute and is used to t reat acetaminophen toxicity.
Glutathione Paracetamol Paracetamol toxicity Hepatotoxicity Toxicity
32
33 B is not correct. 8°/o chose this.
34 Flumazenil is a GABA receptor antagonist that is common ly used to reverse the effects of benzodiazepines.
Flumazenil GABA receptor Gamma-Aminobutyric acid GABA receptor antagonist Benzodiazepine Receptor antagonist Antagonist Receptor (biochemistry)
35
• 36 D is not correct. 6°/o chose this .
• 37 Vitamin K is used to reverse the effects of warfarin by catalyzing the y-carboxylation of glutamic acid residues in
• 38 ~
clotting factors II, VII, I X, X, C, and S.

6
lock
s
Suspend
0
End Block
Item: 35 of 38 ~. I • M k <:] t> al ~· ~
QIO: 4196 .l. ar Previous Next Lab 'lifllues Notes Calculator


18 N-Acetylcysteine acts as a glutathione substitute and is used to t reat acetaminophen toxicity .
Glutathione Paracetamol Paracetamol toxicity Hepatotoxicity Toxicity
19
20 B is not correct. 8°/o chose this.
21 Flumazenil is a GABA receptor antagonist that is common ly used to reverse the effects of benzodiazepines.
Flumazenil GABA receptor Gamma-Aminobutyric acid GABA receptor antagonist Benzodiazepine Receptor antagonist Antagonist Receptor (biochemistry)
22
23 D is not correct. 6°/o chose this.
24 Vitamin K is used to reverse the effects of warfarin by catalyzing the y-carboxylation of glutamic acid residues in
25 clotting factors II, VII, IX, X, C, and S.
Warfarin Vitamin K Glutamic acid Vitamin Coagulation
26
• 27 E is not correct. 15% chose this .
28
Neostigmine is a cholinesterase inh ibitor, which wi ll increase the acetylcholine concentration at the
neuromuscu lar junction . This can potentiate the effects of the depolarizing blockade. Cho linesterase inhibitors
29 such as neostigmine can be used to reverse the action of nondepo larizing agents but cannot reverse the action
30 of succinylcholine or malignant hyperther mia .
Malignant hyperthermia Acetylcholinesterase inhibitor Neostigmine Suxamethonium chloride Neuromuscular junction Acetylcholine Hyperthermia Cholinesterase
31
Depolarization Enzyme inhibitor
32
33
34 Bottom Line:
35
Dantrolene is a drug that inh ibits the release of calcium from the sa rcoplasmic reticu lum of skeletal muscle and
• 36 can be used to treat ma lignant hyperthermia or neuroleptic ma lignant synd rome.
• 37 Neuroleptic malignant syndrome Oantrolene Malignant hyperthermia Hyperthermia Sarcoplasmic reticulum Skeletal muscle Antipsychotic Calcium Cancer Muscle

• 38 ~

6
lock
s
Suspend
0
End Block
Item: 35 of 3 8 ~ 1 • M k -<:J 1>- Jil ~· !:';-~
QIO: 4196 ..L ar Pre v ious Next Labfli!llues Notes Calcula t o r
A A

18
FA17 p520.1
19
Neuromuscular !uscle paralysis in surgery or mechanical \'Cntilation. Sclccti,·c for 'm nicotinic receptors at
20
blocking drugs neuromuscular junction but not autonomic 'n receptors.
21
Depolarizing Succinylcholine-strong Ch receptor agonist; produces sustained depolarization and pre,·ents
22
muscle contraction.
23 Reversal of blockade:
24 Phase I (prolonged depolarization)- no antidote. Block potentiated br cholinesterase inhibitors.
25 Phase II (repolarizcd but blocked; ,\ Ch receptors arc available, but desensitized)- may be
26
re,·ersed \\'ith cholinesterase inhibitors.
Complications include hrpercalcemia, hyperkalemia, malignant hyperthermia.
. 27
Nondepolarizing Tubocurarine, atracurium, mi,·acurium, p:mcuron ium, 'ecuronium, rocuron ium-compet it ive
28
antagonists-compete " ith ACh for receptors.
29
Reversal of blockade-neostigmine (must be given with atropine to pre\'ent muscarinic effects such
30 as bradycardia), edrophon ium, and other cholinesterase inhibitors.
31
32 FA17 p 520.2
33 Dantrolene
34 MECHANISM Prevents release of Ca 2+ from the sarcoplasmic reticulum of skeletal muscle by binding to the
35 ryanodine receptor.
. 36 CLINICALUSE Malignant hyperthermia and ncurolept ic malignant syndrome (a toxicity of antipsychotic drugs).
. 37
. 38 FA17o519.1

a
Lock
s
Suspend
8
End Bl ock
Item: 35 of 3 8 ~ 1 • M k -<:J 1>- Jil ~· !:';-~
QIO: 4196 ..L ar Pre v ious Next Labfli!llues Notes Calcula t o r
A
. A

18 Re\'ersal of blockade-neostigmine (must be given wit·h atropine to pre\·ent muscarin ic effects such
19 as bradycardia), edrophonium, and other cholinesterase inhibitors.
20
21 FA17 p 520.2
Dantrolene
22
23
MECHANISM Pre\·ents release of Ca2• from the sarcoplasmic reticulum of skeletal muscle by binding to the
rranodine receptor.
24
CLINICAL USE lalignant hr perthermia and neuroleptic malignant S} ndrome (a toxicity of antipsychotic drugs).
25
26
• 27 FA17 p 519.1

28
Inhaled anesthetics Desflurane, halothane, enfluranc, i oAuranc. scvoAuranc, methoxyflurane, f 20 .
29 MECHANISM Mechanism unknO\\ n.
30 EFFECTS Myocardial depression, respiratory depression, nausea/emesis, f cerebral blood fl ow (~ cerebral
31
metabolic demand).
32 ADVERSE EFFECTS Hepatotoxicity (halothane), nephrotoxicity (methoxyflurane), proconvulsant (enAurane,
epileptogenic), expansion of tmppcd g:~ s in a body cavity (N20 ).
33
Malignant hyperthermia- rare, life-threatening cond ition in which inhaled anesthetics or
34
succinylcholine induce fever and SC\'Cre muscle contractions. Susceptibility is often inherited as
35 autosomal dominant with variable penetrance. ~ lutations in voltage-sensitive ryanodine receptor
• 36 cause f Ca 2+ release from sarcoplasmic reticulum. Treatment: dantrolene (a ryanod inc receptor
• 37 antagonist).
• 38

a
Lock
s
Suspend
8
End Bl ock
Item: 36 of 3 8 ~ 1 • M k -<:J 1>- Jil ~· !:';-~
QIO: 1445 ..L ar Pre v ious Next Lab fli!ltues Notes Calcula t o r

IAA]
A A

18
A concerned mother brings her 7-year-old son to the ped iatrician after his teacher reports episodes during
19 wh ich the chi ld becomes inattentive. The teacher states that the ch ild frequently ceases his activities and
20 "stares blankly into space." He resumes his normal activities shortly thereafter.
21
22 What is the mechanism of the agent that will most effectively treat this child's disorder?
23 :
24 A. Activating GABA receptors with a y-Aminobutyric acid analogue
25 B. Blocking T-type calcium channels
26
C. Increased firing frequency of chloride channels
. 27
D. Increasing efflux/decrease influx of sodium channels
28
29 E. Limiting influx of sodium channels
30
31
32
33
34
35
. 36
. 37
. 38

a
Lock
s
Suspend
8
End Bl ock
Item: 36 of 38 ~. I • M k <:] t> al ~· ~
QIO: 1445 .l. ar Previous Next lab 'lifllues Notes Calculator


18
The correct answer is B. 65°/o chose this.
19
This child is suffering from absence (petit ma l) seizures. The typical age range 2 seconds
20
for this disorder is 3-7 years, although it may continue into ado lescence r)r v v a y~
21 before subsiding in adulthood. Absence seizu res usual ly occur several t imes
22 per day. During these seizures, the patient experiences an abr upt but _ __
momentary episode of lost consciousness fol lowed by resumption of previous
23 Image copyright © 2012
activities . Ethosuximide is the drug of choice for this type of seizu re disorder. Benjamin eta/; licensee
24 Its mechanism is to block the T-type calcium channel. Springer.
25
The EEG shows the spike and wave pattern characteristic of absence seizures.
26
Ethosuximide T-type calcium channel Electroencephalography Calcium channel Absence seizure Epilepsy Spike-and-wave Epileptic seizure Calcium
• 27
28
A is not correct. 9°/o chose this.
29
Gabapent in, an analogue of y-aminobutyr ic acid, has been approved for the treatment of al l seizu re types
except pr imary generalized seizures (ie, ton ic-clon ic and absence) . Gabapentin can be used to t reat partial
30 seizures that subsequently genera lize, but is not approved for primary general ized seizures . It is normally used
31 as adjunct therapy for seizures that respond poorly to sing le-agent therapy. Gabapentin functions to inh ibit
32 high-voltage-activated Ca 2+ channels.
Gabapentin Generalised tonic-clonic seizure Partial seizure Epileptic seizure Seizure types Generalised epilepsy Structural analog
33
34 C is not correct. 8°/o chose this.
35 Diazepam, a long-acting benzodiazepine, is the first-l ine agent for treatment of status epilepticus, which is
defined as >5 minutes of continuous seizu res or >2 discrete seizures between wh ich there is incomplete
36
recovery of consciousness . It is not indicated for the treatment of absence seizu res. It increases the fi r ing
• 37 frequency of ch loride channe ls, resu lting in y-aminobutyric acid agonist activity .
• 38 Benzodiazepine Diazepam Status epilepticus Absence seizure Agonist Epileptic seizure Chloride channel Chloride
~

6
lock
s
Suspend
0
End Block
Item: 36 of 38 ~. I • M k <:] t> al ~· ~
QIO: 1445 .l. ar Previous Next lab 'lifllues Notes Calculator

18
g g •
Gabapentin Generalised tonic-clonic seizure Partial seizure Epileptic seizure Seizure types Generalised epilepsy Structural analog
19
20 C is not correct. 8°/o chose this.
21
Diazepam, a long-acting benzodiazepine, is the fi rst-l ine agent fo r treatment of status epilepticus, w hich is
defined as >5 minutes of continuous seizu res or >2 discrete seizures between wh ich the re is incomplete
22 recovery of consciousness. It is not indicated for the t reatment of absence seizu res. It increases the fi r ing
23 frequency of ch loride channe ls, resu lting in y-aminobutyric acid agonist activity.
Benzodiazepine Diazepam Status epilepticus Absence seizure Agonist Epileptic seizure Chloride channel Chloride
24
25 D is not correct. 8°/o chose this.
26 Phenytoin is effective in all seizure types except absence seizu res. It is the drug of choice for initial therapy,
• 27 particular ly in adu lts, but has sign ificant adverse effects, including sedation, ataxia, nystagmus, hirsutism, and
gingival hyperplasia. I ts mechan ism is to increase sodium channel inactivation by increasing the efflux and
28
decreasing the influx of the channels.
29 Hirsutism Phenytoin Nystagmus Gingival enlargement Ataxia Sodium channel Absence seizure Epileptic seizure Sedation Sodium Seizure types Hyperplasia

30
E is not correct. 10% chose this.
31
Carbamazepine is an antiepileptic agent that is not indicated for the treatment of absence seizures. It is used
32 instead in the t reatment of tonic-clonic (grand mal) and partial (foca l) seizures. It increases sodium channe l
33 inactivation in part by limiting sodium channe l influx. Valproic acid also inactivates sodium channels and can
34
treat absence seizu res. Ethosuximide and valproic acid are equa lly effective in treating absence seizures, but
ethosuximide is prefer red because valproic acid has a significantly worse side effect profile, which includes
35 headache, somnolence, and GI sy mptoms.
36 Ethosuximide Carbamazepine Valproate Somnolence Sodium channel Anticonvulsant Headache Side effect Seizure types Absence seizure Epileptic seizure

• 37 Generalised tonic-clonic seizure Sodium Adverse effect

• 38 ~

6
lock
s
Suspend
0
End Block
Item: 36 of 38 ~. I • M k <:] t> al ~· ~
QIO: 1445 .l. ar Previous Next lab 'lifllues Notes Calculator

• Benzodiazepine Diazepam Status epilepticus Absence seizure Agonist Epileptic seizure Chloride channel Chloride
18
19 D is not correct. 8°/o chose this.
20 Phenytoin is effective in all seizure types except absence seizu res. It is the drug of choice for initial therapy,
21 particular ly in adu lts, but has sign ificant adverse effects, including sedation, ataxia, nystagmus, hirsutism, and
gingival hyperplasia. I ts mechan ism is to increase sodium channel inactivation by increasing the efflux and
22
decreasing the influx of the channels.
23 Hirsutism Phenytoin Nystagmus Gingival enlargement Ataxia Sodium channel Absence seizure Epileptic seizure Sedation Sodium Seizure types Hyperplasia
24
E is not correct. 10% chose this.
25
Carbamazepine is an antiepileptic agent that is not indicated fo r the treatment of absence seizu res. It is used
26 instead in the t reatment of tonic-clonic (grand mal) and partial (foca l) seizu res. It increases sodium channe l
• 27 inactivation in part by limiting sodium channe l influx. Valproic acid also inactivates sodium channels and can
28 treat absence seizu res. Ethosuximide and valproic acid are equa lly effective in t reating absence seizures, but
ethosuximide is prefer red because valproic acid has a significantly wo rse side effect profile, which includes
29
headache, somnolence, and GI symptoms.
30 Ethosuximide Carbamazepine Valproate Somnolence Sodium channel Anticonvulsant Headache Side effect Seizure types Absence seizure Epileptic seizure

31 Generalised tonic-clonic seizure Sodium Adverse effect

32
33
Bottom Line:
34
Absence seizu res are a type of genera lized seizure that affects chi ldren around the ages of 3-7 years.
35
Cl inical ly, the seizu re episodes look like "sta ring spells." Electroencepha log raphy shows a cha racte ristic 3-Hz
36 spike-and-wave pattern . Treatment is with ethosuximide (first line) or valproic acid.
• 37 Ethosuximide Valproate Electroencephalography Generalised epilepsy Spike-and-wave Absence seizure Epileptic seizure

• 38 ~

6
lock
s
Suspend
0
End Block
Item: 36 of 3 8 ~ 1 • M k -<:J 1>- Jil ~· !:';-~
QIO: 1445 ..L ar Pre v ious Next Lab fli!ltues Notes Calcula t o r
- ---- - ---- • 1' - • •

18 FIRST AID FACTS

19
FA17p514.1
20
Epilepsy drugs
21 GENERALIZED
22 ~
<
'-J ~ .,.,
23 C>
~
:z
C>
~ ... :::>
'-J

...... ..:=
.,., ~
~
u
24
<
~ v
'-J
z
.,., ........
:::> ...

..."" .,., ...


........
:z
< .....
C>
MECHANISM NOTES
< SIDE EFFECTS
25
Ethosuximide • Blocks thalamic T-type Cah El GI IIJ- Ethosuximide Sucks to ha,·e Silent
26 ~ channels causes Fatigue. C l distress. (absence) Seizures
. 27 lleadache, Itching (and
urticaria), and Stevens-
28 Johnson syndrome
29 Benzodiazepines •• f CABt\, action Sedation, tolerance, Also for eclampsia seizures (1st
(eg, diazepam, ~ dependence, respiratory line is MgS04)
30
lorazepam, dcprc~~ion
31 midazolam)
32 Phenobarbital ~ ~ f CABA" action Sedation, tolerance, Ist line in neonates
dependence, induction
33 of cytochrome P--!50,
34
35 Phenytoin, ~ •
~
...
~
Blocks Na+ channels; 1cro-
order kinetics
cardiorespiratory depression
\Jeurologic: nystagmus, diplopia, ataxia, sedation, peripheml
neuropathy. Dcrmatologic: hirsutism, Stc,·em-)ohnson
fosphenytoin
36 S) ndromc. gingival hyperplasia, DRESS syndrome.
.t-.lusculoskcletal: osteopenia, SLE-like syndrome. llematologic:
. 37
megaloblastic anemia. Reproducti,·c: teratogenesis (fetal
. 38 hydantoin S) ndromc). Other: C)tochrome P-450 induction

a
Lock
s
Suspend
8
End Bl ock
Item: 36 of 3 8 ~ 1 • M k -<:J 1>- Jil ~· !:';-~
QIO: 1445 ..L ar Pre v ious Next Lab fli!ltues Notes Calcula t o r

18
y I g , g ' I p ' ' ,p p
fosphenytoin order kinetics neuropathy. Oermatologic: hi rsutism, Stc\'cns-)ohnson
19 S) ndrome. gingival hyperplasia, DRESS syndrome.
l\lmculoskeletal: osteopenia, SLE-Iike syndrome. llematologic:
20 megaloblastic anemia. Reproducti\'e: teratogenesis (fetal
21 hydantoin S) ndrome). Other: C)tochrome P-4;Qinduction
22 Carbamazepine ./ Blocks a+ channels Diplopia, ata\ia, blood 1st line for trigeminal neuralgia
./ d) M:r.uia~ (agranulocytosis,
23 aplastic anemia), ]j,er
to.xicit). teratogenesis.
24
induction of C\ tochrome
25 P-450, SIADI i, Ste,ens-
Johnson ~) ndrome
26
Valproic acid ./ ./ I i\a+ channel inacti\'ation, C l distress. rare but fatal Also used for m\·oclonic seizures,
• 27 ./ I CABA concentration hepatotoxicity (measure bipolar disord~r, migraine
28 by inhibiting CAB.\ LFTs), pancreatitis, neural prophylaxis
transaminase h1be defects, tremor, weight
29 gain, contraindicated in
30 pregnancy
Vigabatrin I CABA by irrcvcrsibl) Permanent visual loss (black
31 inhibiting CABA box warning)
32 trans:uninase
33 Gabapentin Primarily inh ibits high-voltage- Sedation, ataxia Also used for periphera I
activa ted Ca 2+channels; neuropathy, postherpetic
34 designed as CABA analog neuralgia
35 Topiramate ./ ./ Blocks 'a+ channels, I C \ BA Sedntion, mental dulling, Also used for migraine
action kidney ~I ones, weight loss, pre,·ention
36
glaucoma
• 37 Lamotrigine ./ ./ ./ Blocks ' oltage-gated !a+ Ste\ens-Johnson S) ndrome
• 38 channels, inhibits the release (must be titrated slo\\ly•)
• r ' •

a
Lock
s
Suspend
8
End Bl ock
Item: 36 of 3 8 ~ 1 • M k -<:J 1>- Jil ~· !:';-~
QIO: 1445 ..L ar Pre v ious Next Lab fli!ltues Notes Calcula t o r
A
induction of C) tochrome A

18
P-450, Slt\011, Ste\ens-
19 Johnson syndrome
20 Valproic acid ./ • ./ I 'a+ channel inactivation, C l distress. rare but fatal Also used for myoclonic seizures,
./ I GA BA concentration hepatotoxicity (measure bipolar disord~r, migraine
21 by inhibiting GABA LFTs), pancreatitis, neural proph}iaxis
22 transaminase tube defects, tremor, weight
gain, contraindicated in
23 pregnancy
24 Vigabatrin I GA BA b} irrc\'crsibl) Perm.menl 'isuallos~ (black
inhibiting CAB.\ bo' "arning)
25 transaminase
26 Gabapentin ./ Primaril} inhi~its high-,oltage- Sedation. ata,ia Also used for peripheral
. 27 acti\'ated Ca-+ channels; neuropathy, postherpetic
designed as CABA analog neuralgia
28
Topiramate ./ ./ Blocks •a+ channels, I C \B!\ Sedation, mental dulling . Also used for migraine
29 action kidney ~ Iones, weight loss, prevention
gbucoma
30
Lamotrigine ./ ./ ./ Blocks voltage-gated 1 a' Stevens-Johnson syndrome
31 channels, inhibits the release (must be titra ted slowly)
32 of glutamate
Levetiracetam ./ ./ Unknown; may modulate ~;11 igue,
drowsiness,
33
CABA and glutamate release headache, neuropsychiatric
34 svmptoms (eg, personality
changes)
35
Tiagabine ./ I CABA by inhibiting rcuptake
36
= 1st line; = lst line for acute; ••• = lst line for prophylaxis.
. 37
. 38

a
Lock
s
Suspend
8
End Bl ock
Item: 27 of 3 8 ~ 1 • M k -<:J 1>- Jil ~· !:';-~
QIO: 4835 ..L ar Pre v ious Next Lab fli!ltues Not es Calcula t o r

IAA]
A A

18
A 70-year-old woman presents to her physician w ith a history of memory loss and occasiona lly becoming lost
19 when in publ ic. Brain biopsy of another patient suffering from the same disease process is shown in the
20 1mage.
21
22
23
24
25
26
• 27
28
29
30
31
32
What is the most appropriate therapy?
33
34 :
A . Bromocriptine
35
36 B. Diazepam
• 37 C. Donepezil
• 38 ft I -' · -..J---1---L.. :..J---

a
Lock
s
Suspend
8
End Bl ock
18
19
20
21
22
23
24
25
26
. 27
28
29 What is the most appropriate therapy?
30
:
31 A. Bromocriptine
32
B. Diazepam
33
34 C. Donepezi l
35 D. Levodopa/carbidopa
36
E. Seleg iline
. 37
. 38

a
Lock
s
Suspend
8
End Block
Item: 27 of 38 ~. I • M k <:] t> al ~· ~
QIO: 4835 .l. ar Previous Next lab 'lifllues Notes Calculator


18
19 The correct answer is C. 65°/o chose this.
20 This patient has symptoms of dementia, an d the brain biopsy shows neurofibrillary tangles and senile plaques,
21 wh ich are character istic of Alzheimer's disease. People suff ering from Alzheimer's disease often have >90% loss
22
of acetylcholine in their brains. I nitial treatment of th is disease is acetylcholinesterase inhibitors, wh ich increase
the amount of acetylcholine in the presynaptic space. Donepezil is an acetylchol inesterase inhibitor that is often
23 used for this purpose.
24 Acetylcholinesterase inhibitor Donepezil Alzheimer' s disease Acetylcholine Senile plaques Dementia Acetylcholinesterase Neurofibrillary tangle Chemical synapse

25 Biopsy Brain Enzyme inhibitor Brain biopsy

26
A is not correct. 9°/o chose this.
27
Bromocriptine is a dopamine receptor agonist used to treat Pa r kinson's disease.
28 Bromocriptine Parkinson' s disease Dopamine receptor Dopamine Agonist Dopamine agonist Receptor (biochemistry)

29
B is not correct. 5°/o chose this.
30
Diazepam is a benzodiazepine. It increases the frequency of y-amino butyric acid A channel opening and is used
31 to treat anxiety, status epilepticus, and alcohol withdrawa l.
Benzodiazepine Diazepam Status epilepticus Alcohol withdrawal syndrome Butyric acid Anxiety Alcohol Alcoholic beverage
32
33 Dis not correct. 11% chose this.
34 Levodopa/carbidopa is a combination that increases levels of dopamine in the brain . It is used to t reat
35 Parkinson's disease.
Parkinson' s disease Dopamine Carbidopa/levodopa Brain
36
• 37 E is not correct. 10% chose this .
• 38 ~
Seleg iline is an inh ibitor of monoamine oxidase type 8, wh ich increases dopamine availability and is used to

6
lock
s
Suspend
0
End Block
Item: 27 of 38 ~. I • M k <:] t> al ~· ~
QIO: 4835 .l. ar Previous Next lab 'lifllues Notes Calculator
• Biopsy Brain Enzyme inhibitor Brain biopsy
18
19 A is not correct. 9°/o chose this.
20 Bromocriptine is a dopamine receptor agonist used to t reat Pa r kinson's disease.
Bromocriptine Parkinson's disease Dopamine receptor Dopamine Agonist Dopamine agonist Receptor (biochemistry)
21
22 B is not correct. 5°/o chose this.
23 Diazepam is a benzodiazepine. It increases the frequency of y-amino butyric acid A channel opening and is used
24
to treat anxiety, status epilepticus, and alcohol withdrawa l.
Benzodiazepine Diazepam Status epilepticus Alcohol withdrawal syndrome Butyric acid Anxiety Alcohol Alcoholic beverage
25
26
Dis not correct. 11% chose this.
27
Levodopa/ca rbidopa is a combination that increases levels of dopamine in the brain . It is used to t reat
Parkinson's disease.
28 Parkinson's disease Dopamine Carbidopa/levodopa Brain
29
E is not correct. 10% chose this.
30
Se leg iline is an inh ibitor of monoamine oxidase type B, w hich increases dopamine avai lability and is used to
31 treat Parkinson 's disease.
32 Selegiline Parkinson's disease Dopamine Monoamine oxidase Monoamine neurotransmitter Enzyme inhibitor

33
34
Bottom Line:
35
Alzheime r's disease is characterized by neu rofib rillary tangles and senile plaques on brain biopsy . Treatment of
36
this disease is acetylcholinesterase inhibitors .
• 37 Alzheimer's disease Acetylcholinesterase Senile plaques Acetylcholinesterase inhibitor Neurofibrillary tangle Dementia Human brain Brain Biopsy

• 38

6
lock
s
Suspend
0
End Block
Item: 27 of 3 8 ~ 1 • M k -<:J 1>- Jil ~· !:';-~
QIO: 4835 ..L ar Pre v ious Next Lab fli!ltues Not es Calcula t o r
A A

18
19 FA17 p490.1
20 Neurodegenerative l in cognitive ability, memory, or function with intact consciousness.
21 disorders
22 DISEASE DESCRIPTION HISTOLOGIC/GROSS FINDINGS

23 Parkinson disease Parkinson TRAPS your body: Loss of dopaminergic neurons (ie,
24 Tremor (pill-rolling tremor at rest) depigmentation) of substantia nigra pars
Rigidity (cogwheel) compacta.
25
Akinesia (or bradykinesia) Lc''} bodies: composed of a-synuclein
26 Postural instabilit) (intracellular eosinophilic inclusions ).
27 Shuffiing gait
28 ~ IPTP, a contaminant in illegal drugs, is

29 metabolized to MPP+, which can cause


parkinsonian symptoms.
30
Huntington disease Autosomal dominant trinucleotide (CAG)n Atrophy of caudate and putamen with ex vacuo
31
repeat disorder on chromosome 4. Symptoms ven tricu lomega Iy.
32
manifest between ages 20 and 50: chorea, f dopamine, l GABA, l ACh in bra in. 1 euronal
33 athetosis, aggression, depression, dementia death via 1 MDA-R bind ing and glutamate
34 (sometimes initially mistaken for substance excitotoxicity.
35 abuse).
36
Anticipation results from expansion of CAC
repeats. Caudate loses ACh and GABA.
• 37
Alzheimer disease l\ lost common cause of dementia in elder!). Widespread cortical atrophy (normal cortex
• 38 ~ -- - - ____ ..J _ _ _ _ _ - '-: __ .. _ L -- · - • _: • _r _ _ __._ ___ :__ \1 _ L _ : _ __ ..J : _____ ~\ -----=- •'-·

a
Lock
s
Suspend
8
End Bl ock
Item: 27 of 3 8 ~ 1 • M k -<:J 1>- Jil ~· !:';-~
QIO: 4835 ..L ar Pre v ious Next Lab fli!ltues Not es Calcula t o r
A A

18 Alzheimer disease ~ lost


common cause of dementia in elderly. Widespread cortical atrophy (normal cortex I];
19 Down syndrome patients have t risk of cortex in Alzheimer disease [!t), especially
20 developing Alzheimer disease, as APP is hippocampus (arrows in : and ). arrowmg
21 located on chromosome 21. of g) ri and widening of sulci.
Associated with the folio'' ing altered proteins: Senile plaques [!) in gray matter: extracellular
22
• poE2: l risk of sporadic form ~ -amyloid core; mar cause amyloid angiopathy
23
• poE4: t risk of sporadic form - intracranial hemorrhage; A~ (amyl oid-~)
24 APP, presenilin-1, prcscnilin-2: familial S) nthcsizcd by cleaving amyloid precursor
25 forms (10%) with earlier onset protein (APP).
26 l Ch 1'\eurofibrillary tangles 0 : intracellular,
hrperphosphorylated tau protein = insoluble
27
cytoskeletal elements; number of tangles
28
correlates with degree with dementia.
29
Frontotemporal Early changes in personalit) and behavior Frontotemporal lobe degeneration
30 dementia (Pick (behavioral nuiant), or aphasia (primary Inclusions of hyperphosphorylated tau (round
31 disease) progressi,·c aphasia). Pick bodies [!}) or ubiquitinatcd TDP-•+3.
32 May have associated movement disorders
33
(eg, parkinsonism, ALS-Iikc UM /L I I
degeneration).
34
Lewy body dementia Dementia and visual hallucinations Intracellular Lewy bodies rJ primarily in cortex.
35
("haLewycinations") - parkinsonian features
36
Vascular dementia Result of multiple arterial infarcts and/or ~ IRI
or CT shows multiple cortical and/or
• 37
chronic ischemia. subcortical infarcts.
• 38 StPn-wi~l" rll"rlinP in rnPniti\l' :1hililv wilh hiiP-

a
Lock Suspend
s 8
End Bl ock
Item: 27 of 3 8 ~ 1 • M k -<:J 1>- Jil ~· !:';-~
QIO: 4835 ..L ar Pre v ious Next Lab fli!ltues Not es Calcula t o r
A A

18 Vascular dementia Result of multiple arterial infarcts and/or lRl or CT shows multiple cortical and/or
19 chronic ischemia. subcortical infarcts.
20 Step-wise decline in cognitive ability wit h late-
onset memory impairment. 2nd most common
21
cause of dementia in elderly.
22
Creutzfeldt-Jakob Rapidly progressive (weeks to months) dementia Spongiform cortex.
23
disease with myoclonus (""startle myoclonus"). Prions (Prpc ..... Prpsc sheet [J3-pleated sheet
24 Commonly see periodic sharp waves on EEC resistant to proleases]) C).
25 and t 14-3-3 protein in CSF.
26
27
28
29
30
31
32
33
34
35
36
• 37
• 38

a
Lock
s
Suspend
8
End Bl ock
18
19
20
21
22
23
24
25
26
27
28
29
FA1 7 p 51 8.5
30
Alzheimer disease drugs
31
Memanti ne
32
MECHANISM MDA receptor antagonist; helps prevent cxcitotoxicity (mediated by Ca 2+).
33
ADVERSEEFFECTS Dizziness, confusion, hallucinations.
34
Donepezil, galantamine, rivast igmine, tacrine
35
MECHANISM AChE inhibitors.
36
• 37
ADVERSE EFFECTS 1\ausea, dizziness, insomnia.
• 38

a
Lock Suspend
s 8
End Block
Item: 37 of 3 8 ~ 1 • M k -<:J 1>- Jil ~· !:';-~
QIO: 250 1 ..L ar Pre v ious Next Lab fli!ltues Notes Calcula t o r
A A

18
A 6-year-old boy was brought to a pediatrician 2 weeks ago because his teacher noticed he was "blanking
19 out" in the classroom, staring into space, failing to respond to his name, and occasiona lly drooling. The
20 pediatrician prescribed a drug for these episodes 1 week ago. The boy's parents bring him to the emergency
21
department today because he has worsening gastrointestinal upset, pain on swallowing, and blistering around his
nose and mouth.
22
23
Wh ich drug most likely caused this boy's symptoms?
24
:
25
A. Benzodiazapines
26
27
B. Carbamazepine
28 C. Ethosuximide
29 D. Gabapentin
30
E. Phenytoin
31
32
33
34
35
36
. 37
. 38

a
Lock
s
Suspend
8
End Bl ock
Item: 37 of 38 ~. I • M k <:] t> al ~· ~
QIO: 2501 .l. ar Previous Next lab 'lifllues Notes Calculator


18

I
The correct answer is C. 75°/o chose this.
19
Based on the description of this boy's "episodes, " he is suffer ing from absence seizures. Ethosuximide is the
20
first-line t reatment for absence seizu res, and works by blocking T-type ca lcium channels. It is associated with
21 Stevens-Johnson syndrome (SJS). This is character ized by extensive shedding of the epidermis and blister ing of
22 the nasal, oral, and genital mucosa as wel l as the conjunctivae, which can resu lt in complaints of severe pain on
swallowing and dehydration. Erythema, palpable pu r pura, and epider mal necro lysis also may ensue. SJS is 1
23
treated by stopping the offending agent, managing fluid balance, preventing secondary infections of the skin,
24 and possibly administering corticosteroids, although this last measure is controversia l. More common adverse
25 effects of ethosuximide are gastrointestina l distu r bances, fatigue, and dizziness. I n rare cases, ethosuximide
26
can cause blood dyscrasias. Lamotrigine is an alternative medication that can be used to treat absence seizures
and is also associated with SJS .
27 Ethosuximide lamotrigine Stevens-Johnson syndrome Erythema Absence seizure Corticosteroid Epileptic seizure Dizziness Epidermis Calcium Fatigue (medical)
28 Mucous membrane Calcium channel Dehydration Pharmaceutical drug
29
A is not correct. 3°/o chose this.
30
Benzodiazapines are the first-line treatment for status epilepticus, but are not used for absence seizures . I n
31
addition, benzodiazepines are not associated with Stevens-Johnson synd rome. Benzodiazapine toxicity causes
32 sedation and respiratory depression.
Stevens-Johnson syndrome Hypoventilation Benzodiazepine Status epilepticus Absence seizure Epileptic seizure Sedation Major depressive disorder
33
34 Depression (mood) First-line treatment Therapy Toxicity

35 B is not correct. 8°/o chose this.


36 Carbamazepine is used to t reat partial and tonic-clonic seizures, but not absence seizures . It acts by increasing
37 sodium channel inactivation. Carbamazepine can cause dip lopia, induction of the cytoch rome P-450 system,
• 38
blood dyscrasias, liver toxicity, and Stevens-Johnson syndrome .
~

6
lock
s
Suspend
0
End Block
Item: 37 of 38 ~. I • M k <:] t> al ~· ~
QIO: 2501 .l. ar Previous Next lab 'lifllues Notes Calculator


18 B is not correct. 8°/o chose this.
19 Carbamazepine is used to t reat partial and tonic-clonic seizures, but not absence seizures . It acts by increasing
20
sodium channel inactivation. Carbamazepine can cause dip lopia, induction of the cytoch rome P-450 system,
blood dyscrasias, liver toxicity, and Stevens-Johnson syndrome .
21 Stevens-Johnson syndrome Carbamazepine Diplopia Absence seizure Sodium channel Cytochrome P450 Epileptic seizure Hepatotoxicity liver Sodium
22 Generalised tonic-clonic seizure Toxicity Seizure types
23
D is not correct. 5°/o chose this.
24
Gabapentin is common ly prescribed for foca l seizu res and partial siezu res. The mechanism of action of
25
gabapentin is not fu lly understood, but it is structural ly similar to the neurotransmitter GABA; however, it does
26 not bind to GABA receptors . Common side effects of gabapentin include tremor, nystagmus, and fatigue.
27 Gabapentin Nystagmus Neurotransmitter Gamma-Aminobutyric acid Partial seizure Tremor Epileptic seizure Fatigue (medical) Mechanism of action GABA receptor

28 Adverse drug reaction Receptor (biochemistry) Side effect

29 E is not correct. 9°/o chose this.


30 Phenytoin is used to treat partia l and tonic-clonic seizures, but not absence seizu res. It acts by increasing
31 sodium channel inactivation. Phenytoin toxicity causes nystagmus, diplopia, ataxia, ging ival hyperplasia, and
hi rsutism . Like ethosuximide, it is associated with Stevens-Johnson syndrome .
32
Ethosuximide Hirsutism Stevens-Johnson syndrome Phenytoin Diplopia Nystagmus Gingival enlargement Absence seizure Sodium channel Ataxia Epileptic seizure
33
Generalised tonic-clonic seizure Hyperplasia Sodium Seizure types
34
35
36 Bottom Line:
37 Drug-induced Stevens-Johnson synd rome (SJS ) is characterized by severe bl istering of the mucosal surfaces,
• 38 and potentially also leads to purpura, erythema, and epidermal necrosis. Many antiseizure medications are
~

6
lock
s
Suspend
0
End Block
Item: 37 of 38 ~. I • M k <:] t> al ~· ~
QIO: 2501 .l. ar Previous Next lab 'lifllues Notes Calculator

• blood dyscrasias, liver toxicity, and Stevens-Johnson syndrome .


18
Stevens-Johnson syndrome Carbamazepine Diplopia Absence seizure Sodium channel Cytochrome P450 Epileptic seizure Hepatotoxicity liver Sodium
19
Generalised tonic-clonic seizure Toxicity Seizure types
20
21 D is not correct. 5°/o chose this.
22 Gabapentin is common ly prescribed for foca l seizu res and partial siezu res. The mechanism of action of
gabapentin is not fu lly understood, but it is structural ly similar to the neurotransmitter GABA; however, it does
23
not bind to GABA receptors . Common side effects of gabapentin include tremor, nystagmus, and fatigue.
24 Gabapentin Nystagmus Neurotransmitter Gamma-Aminobutyric acid Partial seizure Tremor Epileptic seizure Fatigue (medical) Mechanism of action GABA receptor
25 Adverse drug reaction Receptor (biochemistry) Side effect
26
E is not correct. 9°/o chose this.
27
Phenytoin is used to treat partia l and tonic-clonic seizures, but not absence seizu res. It acts by increasing
28 sodium channel inactivation. Phenytoin toxicity causes nystagmus, diplopia, ataxia, ging ival hyperplasia, and
29 hi rsutism . Like ethosuximide, it is associated with Stevens-Johnson syndrome .
Ethosuximide Hirsutism Stevens-Johnson syndrome Phenytoin Diplopia Nystagmus Gingival enlargement Absence seizure Sodium channel Ataxia Epileptic seizure
30
Generalised tonic-clonic seizure Hyperplasia Sodium Seizure types
31
32
33 Bottom Line:
34
Drug-induced Stevens-Johnson synd rome (SJS) is characte rized by seve re bl istering of the mucosal surfaces,
35 and potentially also leads to purpura, erythema, and epidermal necrosis. Many antiseizure medications are
36 associated with SJS, including ethosuximide, commonly used to treat absence seizures .
Ethosuximide Stevens-Johnson syndrome Erythema Absence seizure Necrosis Epileptic seizure Epidermis Mucous membrane
37
• 38 ~

6
lock
s
Suspend
0
End Block
Item: 37 of 3 8 ~ 1 • M k -<:J 1>- Jil ~· !:';-~
QIO: 250 1 ..L ar Pre v ious Next Lab fli!ltues Notes Calcula t o r
A A

18
19 FA17 p241.2

20
Drug reactions- musculoskeletal/skin/connective tissue
DRUG REACTION CAUSAL AGENTS NOTES
21
Fat redistribution P rotease inhibitors, G lucocorticoids Fat PiG
22
Gingival hyperplasia Phenytoin, Ca 2'" channel blockers, cyclosporine
23
24
Hyperuricemia (gout) Pvrazinamide, T hia1ides, Furosemide, "\ iacin, Painful Tophi and Feet eed C are
'
C~closporine
25
Myopathy Statins, fibrates, niacin, colchicine, daptomycin,
26
hydrox\·chloroquine, interferon-a,
27
penicillamine, glucocorticoids
28
Osteoporosis Corticosteroids, depot medroxyprogcstcronc
29 acetate, GnRH agonists, aromalasc inhibitors,
30 anticonvulsants, heparin
31 Photosensitivity Sulfonamidcs, Amiodaronc, 'lclracyclincs, 5-FU SAT For Photo
32 Rash (Stevens- Anti-epileptic drugs (especia lly lamotriginc), Steven Johmon has epileptic allergy to mlfa
33 Johnson syndrome) allopurinol, sulfa drugs, penicillin drugs and penicillin
34 SLE-Iike syndrome Sulfa drugs, Hyd ralazine, Isoniazid, I laving lupus is "SIIIPP-E "
35 P rocainamide, Phenytoin, Etancrcept
36 Teeth discoloration Tctracycl ines Tecthracyclines
37 Tendonitis, tendon F'luoroquinolones
• 38 rupture, and

a
Lock
s
Suspend
8
End Bl ock
Item: 37 of 3 8 ~ 1 • M k -<:J 1>- Jil ~· !:';-~
QIO: 250 1 ..L ar Pre v ious Next Lab fli!ltues Notes Calcula t o r
A A

18 Tendonitis, tendon F'luoroqu inolones


19 rupture, and
cartilage damage
20
21
22 FA17 p 514.1

23 Epilepsy drugs
GENERALIZED
24
~

25 ....< !::!
"'
....v .......z
0 :z ::::>
~ 0
~ !::!
26
...
~ .,.. ~
<

27
;::
a:
~
:z
0
~
....,.. "' .......
<
=>"-
~ ...
...::::!
~
MECHANISM SIOEEFFE(IS NOTES
28 Ethosuximide • Blocks thalamic T-type Ca2• Jo:FC IIIJ- Ethosm:imide Sucks to have Silent
./ channels causes Fatigue, C l distress, (absence) Seizures
29
lleadache, Itching (and
30 uri icmia). <lnd Stevens-
Johnson syndrome
31
Benzodiazepines •• t CAB/\,\ action Sedation, tolcr;mcc, Also for eclampsia seizures (lsi
32 (eg, diazepam, ./ dependence, respiratory line is MgS04)
lorazepam, deprc~~i on
33
midazolam)
34
Phenobarbital ./ ./ t CABA~ action Sedation, tolerance, 1st line in neonates
35 dependence. induction
of crlochrome P--!50,
36 cardiorespiratory depression
37 Phenytoin, ./ • ••• Blocks 'a+ channels; 1ero- \Jeurologic: nystagmus, diplopia, ataxia, sedation, peripheral
. 38 fosphenytoin ./ ./ order kinetics neuropathy. Dcrmatologic: hirsutism, Ste,·ens-)ohnson
• 1 • • 1 1 1 .--..n
• r~ro 1

a
Lock Suspend
s 8
End Bl ock
Item: 37 of 3 8 ~ 1 • M k -<:J 1>- Jil ~· !:';-~
QIO: 250 1 ..L ar Pre v ious Next Lab fli!ltues Notes Calcula t o r
A A

18 Phenytoin, ./ • *** Blocks 1a+ channels; 1ero- \Jeurologic: nystagmus, diplopia, ataxia, sedation, peripheral
19 fosphenytoin ./ ./ order ki nctics neuropathy. Oermatologic: hirsutism, Stc\'ens-)ohnson
S) ndrome. gingival hyperplasia, DRESS syndrome.
20 ~ lnsculoskeletal: osleopenia, SLE-Iike syndrome. llematologic:
megalobl.istic anemia. Reproducti\'e: teratogenesis (fetal
21
hyda ntoin S) ndrome). Other: C)tochrome P-4;Q induction
22 Carbamazepine ./ Blocks a• channels Diplopia, ala\ia, blood 1st line for trigeminal neuralgia
23 ./ d) scrasia} (agranulocytosis.
aplastic anemia), Ji,er
24 toJo.icit). teratogenesis.
25 induction of C\ tochrome
P-450, SIADI i, Ste\ens-
26 Johnson ~) ndrome
27 Valproic acid ./ • ./ t 'a+ channel inacti\'ation, C l distress. rare but fatal Also used for myoclonic seizures,
./ t CABA concentration hepatotoxicity (measure bipolar disorder, migraine
28 by inhibiting CABA LFTs), pancreatitis, neural prophylaxis
29 transaminase h1be defects, tremor, weight
gain, contraindicated in
30 pregnancy
31 Vigabatrin t CABA by irrcvcrsibl) Permanent visual loss (black
32
inhibiting CABA bo" warning)
transaminase
33 Primarily inhibits high-voltage- Sedation, ataxia Also used for periphera I
Gabapentin
34 activa ted Ca 2+ channels; neuropathy, postherpetic
designed as CABA analog neuralgia
35
Topiramate ./ ./ Blocks 'a+ channels, t C \ Br\ Sedation, mental dulling, Also used for migraine
36 action kidney ~I ones, weight loss, pre,·ention
37
glaucoma
lamotrigine ./ ./ ./ Blocks , oltagc-gated la+ Ste,ens-Johnson S) ndromc
. 38 l"'h'llnnPic tnhil,i t~ ti-l~ ,.,..)~""' .... cP fnH•<t h~ tit Mt ~¥1 <lnu h·\

a
Lock
s
Suspend
8
End Bl ock
Item: 37 of 3 8 ~ 1 • M k -<:J 1>- Jil ~· !:';-~
QIO: 250 1 ..L ar Pre v ious Next Lab fli!ltues Notes Calcula t o r
A A

18 loxicil), teratogenesis,
induction of C) loch rome
19 P-450, SIADI I, Ste1ens-
20 Johnson~) ndrome

Valproic acid ./ • ./ t t\a+ channel inacti1·ation, Cl distl'ess, rare but fatal Also used for mroclonic seizures,
21
./ t CABA concentration hepatotoxicity (measure bipolar disord~r. migraine
22 by inhibiting CABA LFTs), pancreatitis, neural prophylaxis
transaminase h1be defects, tremor, weight
23
gain, contraindicated in
24 pregnancy
25 Vigabatrin t C.\BA b) irrcvcrsibl) Perm.111ent 'isualloss (black
inhibiting C.\B.\ bo' 11arning)
26 transaminase
27 Gabapentin ./ Primaril} inhi~its high-1oltage- Sedation. ala,ia Also used for peripheral
aclil'ated Ca-• channels; neuropathy, postherpetic
28
designed as CABA analog neuralgia
29 Topiramate ./ ./ Blocks 'a+ channels, t C \ BA Sedation, mental dulling. Also used for migraine
30 action kidney stones, weight loss, prevention
glaucoma
31
Lamotrigine ./ ./ ./ Blocks voltage-gated a' Stevens-Johnson syndrome
32 channels, inhibits the release (mmt be titra ted ~lowly)
of glutamate
33
Levetiracetam ./ ./ Unknown; may modulate J•:1t igue, drowsiness.
34 CABA and glutamate relea~e hcndache, neuropsychiatric
35 svmptoms (cg, personality
changes)
36
Tiagabine t CABA by inhibit ing rcuptakc
37
= 1st line; = 1st line for acute; ••• = 1st line for prophylaxis.
• 38

a
Lock
s
Suspend
8
End Bl ock
Item: 37 of 3 8 ~ 1 • M k -<:J 1>- Jil ~· !:';-~
QIO: 250 1 ..L ar Pre v ious Next Lab fli!ltues Notes Calcula t o r
A A

18
FA17 p 452.1
19 Blistering skin disorders
20
Pemphigus vulgaris Potentially fatal autoimmune skin disorder with lgC antibody against desmoglein (component of
21 desmosomes, which connect keratinocytes in the stratum spinosum).
22 Flaccid intraepidermal bullae caused b) acantholysis (separation ofkeratinocytes, resembling a
23 "row of tombstones"); oral mucosa is also in,olved. T) pe II hypersensitivity reaction.
24
lmmunoAuorescencc rc,·cals antibodies around epidermal cells in a reticular (net-like) pattern
' ikolskr sign<±> (separation of epidermis upon manual stroking of skin).
25
Bullous pemphigoid Less severe than pemphigus vulgaris. Involves lgC antibody against hemidesmosomes (epidermal
26
basement membrane; antibodies are " bullow" the epidermis).
27 Tense blisters containing eosinophils affect skin but spare oral mucosa.
28 lmmunoAuorcscence reveals linear pattern at epidermal-dermal junction [!J.
29 ~ikolsky sign e.

30 Dermatitis Pruritic papules, vesicles, and bullae (oft en found on elbows) D. Deposits of IgA at tips of dermal
31 herpetiformis papillae. Associated with celi<1c dise;~se. Treatment: dapsone, gluten-free diet.
32 Erythema multi forme Associated with infections (cg, Mycoplasma pneumoniae, 1-1$\1}, drugs (eg, sulfa dmgs, ~-l aetams,
33
phenytoin), cancers, autoimmune disease. Presents with multiple types of lesions-maculcs,
papules, vesicles, target lesions (look like targets with multiple rings and dusky center showing

I
34
epithelial disruption)
35
Stevens-Johnson Characterized by fever, bullae formation and necrosis, sloughing of skin at dermal-epidermal
36 syndrome junction, high mortality rate. Typically 2 mucous membranes are im·olved [!I rn.
and targetoid
37 skin lesions may appear, as seen in erythema multifonne. Usually associated with ad,·ersc drug
• 38 reaction. A more severe form of Stc\ ens-johnson S) ndrome (SJS) with> 30% of the body surface

a
Lock
s
Suspend
8
End Bl ock
Item: 37 of 3 8 ~ 1 • M k -<:J 1>- Jil ~· !:';-~
QIO: 250 1 ..L ar Pre v ious Next Lab fli!ltues Notes Calcula t o r

18
A - . . -
phenytoin), cancers, autoimmune disease. Presents with multiple types of lesions-maculcs,
A

19 papules, \'esicles, target· lesions (look like targets with multiple rings and dusky center showing
20 epithelial disruption)
21 Stevens-Johnson Characterized by fe,·er, bullae formation and necrosis, sloughing of skin at dermal-epidermal
syndrome junction, high mortal it)' rate. ' J)·picall) 2 mucous membranes are im·oh-ed , and targetoid
22
skin lesions may appear, as seen in Cr) thema multi forme. Usually associated with achcrsc drug
23
reaction. A more severe form of Stc' ens-johnson S} ndrome (SJS) with > 30% of the body surface
24 area im·olved is toxic epidermal necrolysis (TEN). 10-30% im·oh-ement denotes SJS-TE '.
25
26
27
28
29
30
31
32
33
34
35

I
36
37
. 38

a
Lock
s
Suspend
8
End Bl ock
Item: 38 of 38 ~. I • M k <:] t> al ~· ~
QIO: 1477 .l. ar Previous Next lab 'lifllues Notes Calculator


18
A 60-yea r-old man presents to his primary care physician complaining of difficu lty with activities of daily
19 living. He reports having particu lar difficu lty button ing his clothes and writing. On physica l examination, he
20 has bilateral resting tremor in his hands and cogwhee ling with passive movement. When asked to wa lk, the
patient's gait is slow and shuffl ing, and he turns en bloc. He also complains of a runny nose, cough, and achy
21
muscles. The patient is started on a medication that al leviates his motor symptoms and was orig inally marketed
22 for flu prophylaxis.
23
24 What is the most like ly mechanism of action of this drug in the context of the patient's neurologic disease?
25
:
26 A. Acts as an agonist at dopamine receptors
27
B. Functions as a metabolic precursor to dopamine
28
29
C. I ncreases the bioavailability of dopamine by inh ibition of dopamine decarboxylase
30 D. I nhibits catecho l-0-methyltransferase
31 E. I nhibits degradation of dopamine by monoamine oxidase type B
32
F. Stimulates endogenous dopamine release
33
34
35
36
37
• 38 ~

6
lock
s
Suspend
0
End Block
Item: 38 of 38 ~. I • M k <:] t> al ~· ~
QIO: 1477 .l. ar Previous Next lab 'lifllues Notes Calculator


18
19 The correct answer is F. 39°/o chose this.
20 This patient's symptoms are consistent with Parkinson disease. Amantadine was developed as an antiviral agent
used as prophylaxis and treatment of influenza; however, due to high rates of resistance and low efficacy, it is
21
no longer recommended for this use in the Un ited States. Its antiparkinsonian activity is postulated to occur
22 secondary to its ability to enhance the synthesis and release of endogenous dopamine stores from surviving
23 neurons in the substantia nigra and decrease dopamine reupta ke.
Amantadine Substantia nigra Dopamine Antiparkinson medication Parkinson' s disease Neuron Reuptake Influenza Antiviral drug Endogeny (biology)
24
Preventive healthcare
25
26 A is not correct. 16% chose this.
27 Bromocriptine is a dopaminergic receptor agonist that usual ly is used in combination with levodopa . It is used to
28
treat Parkinson disease as wel l as hyperprolactinemia and acromegaly.
Bromocriptine Acromegaly Hyperprolactinaemia l-DOPA Parkinson' s disease Agonist Dopaminergic Receptor (biochemistry)
29
30 B is not correct. 13% chose this.
31
By acting as a metabol ic precursor to dopamine, levodopa repletes dopamine levels that are reduced in the
substantia nig ra. Levodopa can improve many of the symptoms of Parkinson disease.
32 Substantia nigra Dopamine l-DOPA Parkinson' s disease Metabolism
33
C is not correct. 12% chose this.
34
Carbidopa works by inhibiting dopa decarboxylase and preventing the conversion of levodopa to dopamine in
35 the periphery . Because carbidopa cannot cross the blood-brain barrier, it does not affect conversion of levodopa
36 in the brain and thus increases the availability of dopamine in the central nervous system .
Carbidopa Central nervous system Dopamine l-DOPA Blood-brain barrier Aromatic l-amina acid decarboxylase Nervous system Brain
37
38 ~ D is not correct. 9°/o chose this.
6
lock
s
Suspend
0
End Block
Item: 38 of 38 ~. I • M k <:] t> al ~· ~
QIO: 1477 .l. ar Previous Next lab 'lifllues Notes Calculator


18 C is not correct. 12% chose this .
19 Carbidopa works by inhibiting dopa decarboxylase and preventing the conversion of levodopa to dopamine in
20
the periphery . Because carbidopa cannot cross the blood-brain barrier, it does not affect conversion of levodopa
in the brain and thus increases the availability of dopamine in the central nervous system .
21 Carbidopa Central nervous system Dopamine l-DOPA Blood-brain barrier Aromatic l-amina acid decarboxylase Nervous system Brain
22
D is not correct. 9°/o chose this.
23
Tolcapone is a potent, reversible inhibitor of the enzyme catechol-0-methyltransferase (COMT) . COMT is found
24 both peripheral ly and in the CNS, and inhibition increases availability of L-DOPA.
25 Tolcapone Enzyme Catechol-0-methyl transferase Central nervous system

26
E is not correct. 11% chose this.
27
Seleg iline inh ibits monoamine oxidase B (MAO-B), which preferential ly metabolizes dopamine. By inhibiting
28 MAO-B, selegil ine increases dopamine concentrations in the brain and therefore is useful as a treatment for
29 Parkinson disease.
Selegiline Monoamine oxidase B Dopamine Monoamine oxidase Parkinson' s disease Monoamine neurotransmitter Metabolism Brain
30
31
32 Bottom Line:
33 Amantadine enhances the synthesis and release of endogenous dopamine from surviving neurons in the
34 substantia nig ra by an incompletely understood mechan ism, making it a useful treatment for Parkinson
35
disease. It was developed as an antiviral agent used for the prophylaxis and treatment of influenza; however,
its use as a influenza treatment is no longer recommended in the United States due to widespread resistance
36 and lack of efficacy.
37 Amantadine Substantia nigra Dopamine Parkinson' s disease Influenza Neuron Antiviral drug Endogeny (biology) Preventive healthcare

38 ~

6
lock
s
Suspend
0
End Block
Item: 38 of 3 8 ~ 1 • M k -<:J 1>- Jil ~· !:';-~
QIO: 1477 ..L ar Pre v ious Next Lab fli!ltues Notes Calcula t o r
A A

18
19 FA17p517.1
20 Parkinson disease Parkinsonism is due to loss of dopaminergic neurons and excess cholinergic activity.
21 drugs Bromocriptine, Amantadine, Levodopa (with carbidopa), Selegiline (and COMT inhibitors),
22 Antimuscarinics (B.\LS \ ).
STRATEGY AGENTS
23
24
Dopamine agonists Ergot- Bromocriptine
1
on-ergot (preferred)-pramipexole, ropinirole
25
t dopamine availability Amantadine (t dopamine release and l dopamine reuptake): toxicity= ataxia, livedo reticularis.
26
27
f L-DOPA availability Agents prevent peripheral (prc-BBB) L-DOPA degradation .... t L-OOPA entering CNS .... t central
L-DOPA available for com·ersion to dopamine.
28
Lc,·odopa (L-DOPA)/carbidopa-carbidopa blocks peripheral conversion of L-OOP to
29 dopamine by inhibiting DOP decarboxylase. Also reduces side effects of peripheral L-DOPr\
30 conversion into dopam ine (eg, nausea, vomiting).
31 Entacapone, tolcapone-prcvent periphera l L-OOP degradation to 3-0-methyldopa (3-0MD)
32
by inhibiting COMT.

33 Prevent dopamine Agents act centrally (post-8813) to inhibit breakdown of dopamine.


breakdown • Selegiline-blocks conversion of dopamine into DOPAC by selectively inhibiting MAO-B.
34
• Tolcapone-bloeks conversion of dopamine to 3-methoxytyramine (3-MT) by inhibiting central
35
COMT
36
Curb excess cholinergic Benztropine, trihexyphenidyl (Antimuscarinic; impro,·es tremor and rigidity but has little effect on
37 activity .
brad\-kinesia .
in Parkinson disease). Park rour 'vlercedes-Benz.
38

a
Lock
s
Suspend
8
End Bl ock
Item: 38 of 3 8 ~ 1 • Ma rk -<:J I> ~ £!1}>'
• !!":-~
QIO: 1477 ..L Pre v ious Next Lab lues Notes Calcula t o r
A A

18
CIRCULATION
19
L·OOPA
20 DOC
21
22
23
BlOOD-
BRAIN
BARRIER
--{
r En~aeapone
Toopone )
24
L·OOPA
25
26
PRESYNAPTlC
27 TERMINAl FROM niE Dopamine
SUBSTANTIA NIGRA Too pone
28
29
30
31
32 Setegiline
Rasag1~ne
33 0
34 0 0
0

36
0 0
37
38 on<TCV.... AOTV" r
lock
a s
Suspend
8
End Block
Item: 38 of 3 8 ~ 1 • M k -<:J 1>- Jil ~· !:';-~
QIO: 1477 ..L ar Pre v ious Next Lab fli!ltues Notes Calcula t o r
. ..,,, ~

18
19
20 L-OOPA

21 ooc)
22 PRESYNAPTIC
TBIMINAl FROM THE Dopamine
23 SUBSTANTIA NIGRA
24 J
~
25
26
27 ~ Reuptake

28 r 0
29 0
0
30 0
0
31 Amantadine 0
32
33
POSTSYNAPTIC
34 TERMINAL IN
THE STRIATUM
35
36
37
38

a
Lock
s
Suspend
8
End Bl ock
Item: 38 of 3 8 ~ 1 • M k -<:J 1>- Jil ~· !:';-~
QIO: 1477 ..L ar Pre v ious Next Lab fli!ltues Notes Calcula t o r
A A

18
FA17p518.1
19
Levodopa/ carbidopa
20
MECHANISM t b ·el of dopamine in brain. Unlike dopamine, L-DOPA can cross blood-brain barrier and is
21
com·erted by dopa decarboxylase in the CNS to dopamine. Carbidopa, a peripheral DOPA
22 decarbox·ylase inhibitor, is given with 1.-DOPA to t the bioa,ailability of L-DOPA in the brain and
23 to limit peripheral side effects.
24 CliNICAl USE Parkinson disease.
25 ADVERSE EFFECTS Arrhythmias from t peripheral formation of catecholamines. Long-term use can lead to dyskinesia
26 following administration ("on-off" phenomenon), akinesia between doses.
27
28 FA17 p 470.1
29
Basal ganglia Important in ,·oluntary movements and making postural
30 adjustments.
31 Recei,·es cortical input, prO\·ides negative feedback to cortex to
32 modulate movement.
Striatum =putamen (motor) + caudate (cognitive). D 1-Receptor = DIRect
33
Lentiform= putamen+ globus pallidus. pathway.
34
Indi rect = Inhibitory.
35
Input Irom SNc
36
• Stimulatory
37 Dopamme
• Inhibitory
38 •

a
Lock
s
Suspend
8
End Bl ock
18
19 SNc Substlntia nigra pars compacta
20 GPe Globus pallidus extemus
GPi Globus pallidus anternus
21 .
Direct lndarect'
STN Subthalamic nucleus
22 Motor cortex pathway pathway o, Dopamine 01 receptor
23 faditates anh1bits 02 Dopamine 0 2 receptor
movement
24
25
26
27
Thalamus
28
29
30
31
32
33
34 Pedunculo·

35
36
37 Spinal
cord
38 •

a
Lock
s
Suspend
8
End Block

You might also like